Download DUET Master 2018 DU PhD in Chemistry Question Paper With Answer Key

Download DUET (Delhi University Entrance Test conducted by the NTA) 2018 DU PhD in Chemistry Question Paper With Solution Key

1)
2)
3)
4)
5)
DU PhD in Chemistry
Topic:- DU_J18_PHD_CHEM
Which of the following statements about sulfur dioxide is true?
[Question ID = 677]
1. It forms a S-S dimer in condensed phase [Option ID = 2707]
2. Its anhydride of sulfuric acid [Option ID = 2706]
3. Its O-S-O angle is 180
0
[Option ID = 2708]
4. It is a product of the combustion of fossil fuels that contain sulfur [Option ID = 2705]
Correct Answer :-
It is a product of the combustion of fossil fuels that contain sulfur [Option ID = 2705]
Which of the following is a strong acid in pure liquid HF
[Question ID = 683]
1. H
2
O [Option ID = 2731]
2. NaF [Option ID = 2729]
3. CH
3
COOH [Option ID = 2730]
4. SbF
5
[Option ID = 2732]
Correct Answer :-
SbF
5
[Option ID = 2732]
Each of the following molecules can act as a chelating agent EXCEPT [Question ID = 679]
1. [Option ID = 2716]
2. [Option ID = 2714]
3. [Option ID = 2713]
4. [Option ID = 2715]
Correct Answer :-
[Option ID = 2714]
What is correct about h-index?
[Question ID = 758]
1. Alternative of impact factor [Option ID = 3031]
2. Based on most quoted papers [Option ID = 3030]
3. Quantify scientific productivity [Option ID = 3029]
4. All of these [Option ID = 3032]
Correct Answer :-
All of these [Option ID = 3032]
The hyperfine electron spin resonance (e.s.r.) spectrum of the benzene radical has how many lines? [Question ID = 748]
1. 12 [Option ID = 2992]
2. 7 [Option ID = 2990]
3. 1 [Option ID = 2991]
4. 6 [Option ID = 2989]
Correct Answer :-
7 [Option ID = 2990]
FirstRanker.com - FirstRanker's Choice
1)
2)
3)
4)
5)
DU PhD in Chemistry
Topic:- DU_J18_PHD_CHEM
Which of the following statements about sulfur dioxide is true?
[Question ID = 677]
1. It forms a S-S dimer in condensed phase [Option ID = 2707]
2. Its anhydride of sulfuric acid [Option ID = 2706]
3. Its O-S-O angle is 180
0
[Option ID = 2708]
4. It is a product of the combustion of fossil fuels that contain sulfur [Option ID = 2705]
Correct Answer :-
It is a product of the combustion of fossil fuels that contain sulfur [Option ID = 2705]
Which of the following is a strong acid in pure liquid HF
[Question ID = 683]
1. H
2
O [Option ID = 2731]
2. NaF [Option ID = 2729]
3. CH
3
COOH [Option ID = 2730]
4. SbF
5
[Option ID = 2732]
Correct Answer :-
SbF
5
[Option ID = 2732]
Each of the following molecules can act as a chelating agent EXCEPT [Question ID = 679]
1. [Option ID = 2716]
2. [Option ID = 2714]
3. [Option ID = 2713]
4. [Option ID = 2715]
Correct Answer :-
[Option ID = 2714]
What is correct about h-index?
[Question ID = 758]
1. Alternative of impact factor [Option ID = 3031]
2. Based on most quoted papers [Option ID = 3030]
3. Quantify scientific productivity [Option ID = 3029]
4. All of these [Option ID = 3032]
Correct Answer :-
All of these [Option ID = 3032]
The hyperfine electron spin resonance (e.s.r.) spectrum of the benzene radical has how many lines? [Question ID = 748]
1. 12 [Option ID = 2992]
2. 7 [Option ID = 2990]
3. 1 [Option ID = 2991]
4. 6 [Option ID = 2989]
Correct Answer :-
7 [Option ID = 2990]
6)
7)
8)
9)
10)
11)
12)
The energy changes involving the core electrons of an atom or molecule are expressed in which region of the electromagnetic
spectrum? [Question ID = 742]
1. Ultraviolet and Visible region [Option ID = 2967]
2. X-ray region [Option ID = 2968]
3. Radiofrequency region [Option ID = 2966]
4. Infra-red region [Option ID = 2965]
Correct Answer :-
X-ray region [Option ID = 2968]
Find out the expected intensity ratio of M and M+1 signal for the Naphthalene molecular ion [Question ID = 726]
1. 99:1.1 [Option ID = 2903]
2. 1.1:99 [Option ID = 2904]
3. 9:01 [Option ID = 2901]
4. 1:9 [Option ID = 2902]
Correct Answer :-
9:01 [Option ID = 2901]
Cobalt-60 is used in radiation therapy of cancer and can be produced by the bombardment of Cobalt-59 with [Question ID = 692]
1. Alpha particles [Option ID = 2765]
2. Beta particles [Option ID = 2767]
3. Neutrons [Option ID = 2766]
4. Gamma rays [Option ID = 2768]
Correct Answer :-
Neutrons [Option ID = 2766]
The standard emf of galvanic cell involving 3 moles of electrons in its redox reaction is 0.59 V. The equilibrium constant for the
reaction of the cell is- [Question ID = 763]
1. [Option ID = 3051]
2. [Option ID = 3052]
3. [Option ID = 3049]
4. [Option ID = 3050]
Correct Answer :-
[Option ID = 3052]
A characteristic common to polymers that can be made to conduct electricity such as polyacetylene, polypyrrole is: [Question ID =
685]
1. Conjugation throughout the polymeric chain. [Option ID = 2740]
2. A high degree of cross linking [Option ID = 2738]
3. A very low glass transition temperature [Option ID = 2737]
4. Presence of stereogenic centers of the same configuration [Option ID = 2739]
Correct Answer :-
Conjugation throughout the polymeric chain. [Option ID = 2740]
Impact factor is [Question ID = 768]
1. Ratio between citations and recent citable items publish [Option ID = 3071]
2. All of these [Option ID = 3072]
3. Addition of citations and recent citable items publish [Option ID = 3069]
4. Ratio between recent citable items publish and citations [Option ID = 3070]
Correct Answer :-
Ratio between recent citable items publish and citations [Option ID = 3070]
On the basis of oxidation-reduction potential, which of the following is most likely to occur? [Question ID = 693]
FirstRanker.com - FirstRanker's Choice
1)
2)
3)
4)
5)
DU PhD in Chemistry
Topic:- DU_J18_PHD_CHEM
Which of the following statements about sulfur dioxide is true?
[Question ID = 677]
1. It forms a S-S dimer in condensed phase [Option ID = 2707]
2. Its anhydride of sulfuric acid [Option ID = 2706]
3. Its O-S-O angle is 180
0
[Option ID = 2708]
4. It is a product of the combustion of fossil fuels that contain sulfur [Option ID = 2705]
Correct Answer :-
It is a product of the combustion of fossil fuels that contain sulfur [Option ID = 2705]
Which of the following is a strong acid in pure liquid HF
[Question ID = 683]
1. H
2
O [Option ID = 2731]
2. NaF [Option ID = 2729]
3. CH
3
COOH [Option ID = 2730]
4. SbF
5
[Option ID = 2732]
Correct Answer :-
SbF
5
[Option ID = 2732]
Each of the following molecules can act as a chelating agent EXCEPT [Question ID = 679]
1. [Option ID = 2716]
2. [Option ID = 2714]
3. [Option ID = 2713]
4. [Option ID = 2715]
Correct Answer :-
[Option ID = 2714]
What is correct about h-index?
[Question ID = 758]
1. Alternative of impact factor [Option ID = 3031]
2. Based on most quoted papers [Option ID = 3030]
3. Quantify scientific productivity [Option ID = 3029]
4. All of these [Option ID = 3032]
Correct Answer :-
All of these [Option ID = 3032]
The hyperfine electron spin resonance (e.s.r.) spectrum of the benzene radical has how many lines? [Question ID = 748]
1. 12 [Option ID = 2992]
2. 7 [Option ID = 2990]
3. 1 [Option ID = 2991]
4. 6 [Option ID = 2989]
Correct Answer :-
7 [Option ID = 2990]
6)
7)
8)
9)
10)
11)
12)
The energy changes involving the core electrons of an atom or molecule are expressed in which region of the electromagnetic
spectrum? [Question ID = 742]
1. Ultraviolet and Visible region [Option ID = 2967]
2. X-ray region [Option ID = 2968]
3. Radiofrequency region [Option ID = 2966]
4. Infra-red region [Option ID = 2965]
Correct Answer :-
X-ray region [Option ID = 2968]
Find out the expected intensity ratio of M and M+1 signal for the Naphthalene molecular ion [Question ID = 726]
1. 99:1.1 [Option ID = 2903]
2. 1.1:99 [Option ID = 2904]
3. 9:01 [Option ID = 2901]
4. 1:9 [Option ID = 2902]
Correct Answer :-
9:01 [Option ID = 2901]
Cobalt-60 is used in radiation therapy of cancer and can be produced by the bombardment of Cobalt-59 with [Question ID = 692]
1. Alpha particles [Option ID = 2765]
2. Beta particles [Option ID = 2767]
3. Neutrons [Option ID = 2766]
4. Gamma rays [Option ID = 2768]
Correct Answer :-
Neutrons [Option ID = 2766]
The standard emf of galvanic cell involving 3 moles of electrons in its redox reaction is 0.59 V. The equilibrium constant for the
reaction of the cell is- [Question ID = 763]
1. [Option ID = 3051]
2. [Option ID = 3052]
3. [Option ID = 3049]
4. [Option ID = 3050]
Correct Answer :-
[Option ID = 3052]
A characteristic common to polymers that can be made to conduct electricity such as polyacetylene, polypyrrole is: [Question ID =
685]
1. Conjugation throughout the polymeric chain. [Option ID = 2740]
2. A high degree of cross linking [Option ID = 2738]
3. A very low glass transition temperature [Option ID = 2737]
4. Presence of stereogenic centers of the same configuration [Option ID = 2739]
Correct Answer :-
Conjugation throughout the polymeric chain. [Option ID = 2740]
Impact factor is [Question ID = 768]
1. Ratio between citations and recent citable items publish [Option ID = 3071]
2. All of these [Option ID = 3072]
3. Addition of citations and recent citable items publish [Option ID = 3069]
4. Ratio between recent citable items publish and citations [Option ID = 3070]
Correct Answer :-
Ratio between recent citable items publish and citations [Option ID = 3070]
On the basis of oxidation-reduction potential, which of the following is most likely to occur? [Question ID = 693]
13)
14)
15)
16)
17)
1. [Option ID = 2770]
2. [Option ID = 2772]
3. [Option ID = 2771]
4. [Option ID = 2769]
Correct Answer :-
[Option ID = 2769]
How many diastereoisomers are possible for the compound 2, 4 ?diphenylcyclobutane-1, 3 di carboxylic acids. [Question ID = 725]
1. 6 [Option ID = 2899]
2. 5 [Option ID = 2898]
3. 8 [Option ID = 2900]
4. 4 [Option ID = 2897]
Correct Answer :-
5 [Option ID = 2898]
An increase in equivalent conductance of a strong electrolyte with dilution is mainly due to- [Question ID = 764]
1. increase in ionic mobility of ions [Option ID = 3055]
2. increase in number of ions [Option ID = 3054]
3. 100% ionization of electrolyte at normal dilution [Option ID = 3056]
4. increase in both i.e. number of ions and ionic mobility of ions. [Option ID = 3053]
Correct Answer :-
increase in ionic mobility of ions [Option ID = 3055]
The solid state structures of the principal allotropes of elemental boron are made up of which of the following structural units
[Question ID = 699]
1. [Option ID = 2796]
2. [Option ID = 2795]
3. [Option ID = 2794]
4. [Option ID = 2793]
Correct Answer :-
[Option ID = 2793]
The molecular geometry of thionyl chloride is best described as [Question ID = 688]
1. T-shaped [Option ID = 2752]
2. Tetrahedral [Option ID = 2751]
3. Trigonal pyramidal [Option ID = 2749]
4. Trigonal planar [Option ID = 2750]
Correct Answer :-
Trigonal pyramidal [Option ID = 2749]
In a face-center cubic (FCC) type of crystal lattice, the number of atoms belonging exclusively to each unit cell within the lattice
is/are: [Question ID = 754]
1. 2 [Option ID = 3014]
2. 1 [Option ID = 3013]
3. 3 [Option ID = 3015]
4. 4 [Option ID = 3016]
Correct Answer :-
4 [Option ID = 3016]
FirstRanker.com - FirstRanker's Choice
1)
2)
3)
4)
5)
DU PhD in Chemistry
Topic:- DU_J18_PHD_CHEM
Which of the following statements about sulfur dioxide is true?
[Question ID = 677]
1. It forms a S-S dimer in condensed phase [Option ID = 2707]
2. Its anhydride of sulfuric acid [Option ID = 2706]
3. Its O-S-O angle is 180
0
[Option ID = 2708]
4. It is a product of the combustion of fossil fuels that contain sulfur [Option ID = 2705]
Correct Answer :-
It is a product of the combustion of fossil fuels that contain sulfur [Option ID = 2705]
Which of the following is a strong acid in pure liquid HF
[Question ID = 683]
1. H
2
O [Option ID = 2731]
2. NaF [Option ID = 2729]
3. CH
3
COOH [Option ID = 2730]
4. SbF
5
[Option ID = 2732]
Correct Answer :-
SbF
5
[Option ID = 2732]
Each of the following molecules can act as a chelating agent EXCEPT [Question ID = 679]
1. [Option ID = 2716]
2. [Option ID = 2714]
3. [Option ID = 2713]
4. [Option ID = 2715]
Correct Answer :-
[Option ID = 2714]
What is correct about h-index?
[Question ID = 758]
1. Alternative of impact factor [Option ID = 3031]
2. Based on most quoted papers [Option ID = 3030]
3. Quantify scientific productivity [Option ID = 3029]
4. All of these [Option ID = 3032]
Correct Answer :-
All of these [Option ID = 3032]
The hyperfine electron spin resonance (e.s.r.) spectrum of the benzene radical has how many lines? [Question ID = 748]
1. 12 [Option ID = 2992]
2. 7 [Option ID = 2990]
3. 1 [Option ID = 2991]
4. 6 [Option ID = 2989]
Correct Answer :-
7 [Option ID = 2990]
6)
7)
8)
9)
10)
11)
12)
The energy changes involving the core electrons of an atom or molecule are expressed in which region of the electromagnetic
spectrum? [Question ID = 742]
1. Ultraviolet and Visible region [Option ID = 2967]
2. X-ray region [Option ID = 2968]
3. Radiofrequency region [Option ID = 2966]
4. Infra-red region [Option ID = 2965]
Correct Answer :-
X-ray region [Option ID = 2968]
Find out the expected intensity ratio of M and M+1 signal for the Naphthalene molecular ion [Question ID = 726]
1. 99:1.1 [Option ID = 2903]
2. 1.1:99 [Option ID = 2904]
3. 9:01 [Option ID = 2901]
4. 1:9 [Option ID = 2902]
Correct Answer :-
9:01 [Option ID = 2901]
Cobalt-60 is used in radiation therapy of cancer and can be produced by the bombardment of Cobalt-59 with [Question ID = 692]
1. Alpha particles [Option ID = 2765]
2. Beta particles [Option ID = 2767]
3. Neutrons [Option ID = 2766]
4. Gamma rays [Option ID = 2768]
Correct Answer :-
Neutrons [Option ID = 2766]
The standard emf of galvanic cell involving 3 moles of electrons in its redox reaction is 0.59 V. The equilibrium constant for the
reaction of the cell is- [Question ID = 763]
1. [Option ID = 3051]
2. [Option ID = 3052]
3. [Option ID = 3049]
4. [Option ID = 3050]
Correct Answer :-
[Option ID = 3052]
A characteristic common to polymers that can be made to conduct electricity such as polyacetylene, polypyrrole is: [Question ID =
685]
1. Conjugation throughout the polymeric chain. [Option ID = 2740]
2. A high degree of cross linking [Option ID = 2738]
3. A very low glass transition temperature [Option ID = 2737]
4. Presence of stereogenic centers of the same configuration [Option ID = 2739]
Correct Answer :-
Conjugation throughout the polymeric chain. [Option ID = 2740]
Impact factor is [Question ID = 768]
1. Ratio between citations and recent citable items publish [Option ID = 3071]
2. All of these [Option ID = 3072]
3. Addition of citations and recent citable items publish [Option ID = 3069]
4. Ratio between recent citable items publish and citations [Option ID = 3070]
Correct Answer :-
Ratio between recent citable items publish and citations [Option ID = 3070]
On the basis of oxidation-reduction potential, which of the following is most likely to occur? [Question ID = 693]
13)
14)
15)
16)
17)
1. [Option ID = 2770]
2. [Option ID = 2772]
3. [Option ID = 2771]
4. [Option ID = 2769]
Correct Answer :-
[Option ID = 2769]
How many diastereoisomers are possible for the compound 2, 4 ?diphenylcyclobutane-1, 3 di carboxylic acids. [Question ID = 725]
1. 6 [Option ID = 2899]
2. 5 [Option ID = 2898]
3. 8 [Option ID = 2900]
4. 4 [Option ID = 2897]
Correct Answer :-
5 [Option ID = 2898]
An increase in equivalent conductance of a strong electrolyte with dilution is mainly due to- [Question ID = 764]
1. increase in ionic mobility of ions [Option ID = 3055]
2. increase in number of ions [Option ID = 3054]
3. 100% ionization of electrolyte at normal dilution [Option ID = 3056]
4. increase in both i.e. number of ions and ionic mobility of ions. [Option ID = 3053]
Correct Answer :-
increase in ionic mobility of ions [Option ID = 3055]
The solid state structures of the principal allotropes of elemental boron are made up of which of the following structural units
[Question ID = 699]
1. [Option ID = 2796]
2. [Option ID = 2795]
3. [Option ID = 2794]
4. [Option ID = 2793]
Correct Answer :-
[Option ID = 2793]
The molecular geometry of thionyl chloride is best described as [Question ID = 688]
1. T-shaped [Option ID = 2752]
2. Tetrahedral [Option ID = 2751]
3. Trigonal pyramidal [Option ID = 2749]
4. Trigonal planar [Option ID = 2750]
Correct Answer :-
Trigonal pyramidal [Option ID = 2749]
In a face-center cubic (FCC) type of crystal lattice, the number of atoms belonging exclusively to each unit cell within the lattice
is/are: [Question ID = 754]
1. 2 [Option ID = 3014]
2. 1 [Option ID = 3013]
3. 3 [Option ID = 3015]
4. 4 [Option ID = 3016]
Correct Answer :-
4 [Option ID = 3016]
18)
19)
20)
21)
22)
23)
24)
Among the following, the weakest oxidizing agent is [Question ID = 675]
1. Mg (s) [Option ID = 2698]
2. [Option ID = 2699]
3. [Option ID = 2700]
4. [Option ID = 2697]
Correct Answer :-
Mg (s) [Option ID = 2698]
For a polymer, which of the following statement/s is/are true? [Question ID = 759]
1. Weight average molecular weight is almost always higher than the number average molecular weight [Option ID = 3035]
2. Formation of a polypeptide from its monomers (amino acids) is an example of addition polymerization [Option ID = 3034]
3. All of these [Option ID = 3036]
4. Vinyl polymerization is an example of condensation polymerization. [Option ID = 3033]
Correct Answer :-
Weight average molecular weight is almost always higher than the number average molecular weight [Option ID = 3035]
Quantum dots are [Question ID = 762]
1. Three dimensional [Option ID = 3048]
2. One dimensional [Option ID = 3046]
3. Two dimensional [Option ID = 3047]
4. Zero dimensional [Option ID = 3045]
Correct Answer :-
Zero dimensional [Option ID = 3045]
The unit of rate constant for a third order reaction is: [Question ID = 749]
1. [Option ID = 2993]
2. [Option ID = 2995]
3. [Option ID = 2996]
4. [Option ID = 2994]
Correct Answer :-
[Option ID = 2996]
All the following elements have at least one isotope that is not radioactive EXCEPT [Question ID = 673]
1. Pb [Option ID = 2690]
2. O [Option ID = 2689]
3. Sn [Option ID = 2691]
4. No [Option ID = 2692]
Correct Answer :-
No [Option ID = 2692]
The conditions for a species to follow Bose-Einstein statistics are; [Question ID = 736]
1. Particles are indistinguishable, with no restriction on filling up of energy levels [Option ID = 2944]
2. Particles are indistinguishable, with a restriction on filling up of energy levels [Option ID = 2943]
3. Particles are distinguishable, with a restriction on filling up of energy levels [Option ID = 2941]
4. Particles are distinguishable, with no restriction on filling up of energy levels [Option ID = 2942]
Correct Answer :-
Particles are indistinguishable, with no restriction on filling up of energy levels [Option ID = 2944]
In the kinetic theory of collisions, the SI unit of collision number, in terms of m (meter) and s (second), is:
[Question ID = 761]
FirstRanker.com - FirstRanker's Choice
1)
2)
3)
4)
5)
DU PhD in Chemistry
Topic:- DU_J18_PHD_CHEM
Which of the following statements about sulfur dioxide is true?
[Question ID = 677]
1. It forms a S-S dimer in condensed phase [Option ID = 2707]
2. Its anhydride of sulfuric acid [Option ID = 2706]
3. Its O-S-O angle is 180
0
[Option ID = 2708]
4. It is a product of the combustion of fossil fuels that contain sulfur [Option ID = 2705]
Correct Answer :-
It is a product of the combustion of fossil fuels that contain sulfur [Option ID = 2705]
Which of the following is a strong acid in pure liquid HF
[Question ID = 683]
1. H
2
O [Option ID = 2731]
2. NaF [Option ID = 2729]
3. CH
3
COOH [Option ID = 2730]
4. SbF
5
[Option ID = 2732]
Correct Answer :-
SbF
5
[Option ID = 2732]
Each of the following molecules can act as a chelating agent EXCEPT [Question ID = 679]
1. [Option ID = 2716]
2. [Option ID = 2714]
3. [Option ID = 2713]
4. [Option ID = 2715]
Correct Answer :-
[Option ID = 2714]
What is correct about h-index?
[Question ID = 758]
1. Alternative of impact factor [Option ID = 3031]
2. Based on most quoted papers [Option ID = 3030]
3. Quantify scientific productivity [Option ID = 3029]
4. All of these [Option ID = 3032]
Correct Answer :-
All of these [Option ID = 3032]
The hyperfine electron spin resonance (e.s.r.) spectrum of the benzene radical has how many lines? [Question ID = 748]
1. 12 [Option ID = 2992]
2. 7 [Option ID = 2990]
3. 1 [Option ID = 2991]
4. 6 [Option ID = 2989]
Correct Answer :-
7 [Option ID = 2990]
6)
7)
8)
9)
10)
11)
12)
The energy changes involving the core electrons of an atom or molecule are expressed in which region of the electromagnetic
spectrum? [Question ID = 742]
1. Ultraviolet and Visible region [Option ID = 2967]
2. X-ray region [Option ID = 2968]
3. Radiofrequency region [Option ID = 2966]
4. Infra-red region [Option ID = 2965]
Correct Answer :-
X-ray region [Option ID = 2968]
Find out the expected intensity ratio of M and M+1 signal for the Naphthalene molecular ion [Question ID = 726]
1. 99:1.1 [Option ID = 2903]
2. 1.1:99 [Option ID = 2904]
3. 9:01 [Option ID = 2901]
4. 1:9 [Option ID = 2902]
Correct Answer :-
9:01 [Option ID = 2901]
Cobalt-60 is used in radiation therapy of cancer and can be produced by the bombardment of Cobalt-59 with [Question ID = 692]
1. Alpha particles [Option ID = 2765]
2. Beta particles [Option ID = 2767]
3. Neutrons [Option ID = 2766]
4. Gamma rays [Option ID = 2768]
Correct Answer :-
Neutrons [Option ID = 2766]
The standard emf of galvanic cell involving 3 moles of electrons in its redox reaction is 0.59 V. The equilibrium constant for the
reaction of the cell is- [Question ID = 763]
1. [Option ID = 3051]
2. [Option ID = 3052]
3. [Option ID = 3049]
4. [Option ID = 3050]
Correct Answer :-
[Option ID = 3052]
A characteristic common to polymers that can be made to conduct electricity such as polyacetylene, polypyrrole is: [Question ID =
685]
1. Conjugation throughout the polymeric chain. [Option ID = 2740]
2. A high degree of cross linking [Option ID = 2738]
3. A very low glass transition temperature [Option ID = 2737]
4. Presence of stereogenic centers of the same configuration [Option ID = 2739]
Correct Answer :-
Conjugation throughout the polymeric chain. [Option ID = 2740]
Impact factor is [Question ID = 768]
1. Ratio between citations and recent citable items publish [Option ID = 3071]
2. All of these [Option ID = 3072]
3. Addition of citations and recent citable items publish [Option ID = 3069]
4. Ratio between recent citable items publish and citations [Option ID = 3070]
Correct Answer :-
Ratio between recent citable items publish and citations [Option ID = 3070]
On the basis of oxidation-reduction potential, which of the following is most likely to occur? [Question ID = 693]
13)
14)
15)
16)
17)
1. [Option ID = 2770]
2. [Option ID = 2772]
3. [Option ID = 2771]
4. [Option ID = 2769]
Correct Answer :-
[Option ID = 2769]
How many diastereoisomers are possible for the compound 2, 4 ?diphenylcyclobutane-1, 3 di carboxylic acids. [Question ID = 725]
1. 6 [Option ID = 2899]
2. 5 [Option ID = 2898]
3. 8 [Option ID = 2900]
4. 4 [Option ID = 2897]
Correct Answer :-
5 [Option ID = 2898]
An increase in equivalent conductance of a strong electrolyte with dilution is mainly due to- [Question ID = 764]
1. increase in ionic mobility of ions [Option ID = 3055]
2. increase in number of ions [Option ID = 3054]
3. 100% ionization of electrolyte at normal dilution [Option ID = 3056]
4. increase in both i.e. number of ions and ionic mobility of ions. [Option ID = 3053]
Correct Answer :-
increase in ionic mobility of ions [Option ID = 3055]
The solid state structures of the principal allotropes of elemental boron are made up of which of the following structural units
[Question ID = 699]
1. [Option ID = 2796]
2. [Option ID = 2795]
3. [Option ID = 2794]
4. [Option ID = 2793]
Correct Answer :-
[Option ID = 2793]
The molecular geometry of thionyl chloride is best described as [Question ID = 688]
1. T-shaped [Option ID = 2752]
2. Tetrahedral [Option ID = 2751]
3. Trigonal pyramidal [Option ID = 2749]
4. Trigonal planar [Option ID = 2750]
Correct Answer :-
Trigonal pyramidal [Option ID = 2749]
In a face-center cubic (FCC) type of crystal lattice, the number of atoms belonging exclusively to each unit cell within the lattice
is/are: [Question ID = 754]
1. 2 [Option ID = 3014]
2. 1 [Option ID = 3013]
3. 3 [Option ID = 3015]
4. 4 [Option ID = 3016]
Correct Answer :-
4 [Option ID = 3016]
18)
19)
20)
21)
22)
23)
24)
Among the following, the weakest oxidizing agent is [Question ID = 675]
1. Mg (s) [Option ID = 2698]
2. [Option ID = 2699]
3. [Option ID = 2700]
4. [Option ID = 2697]
Correct Answer :-
Mg (s) [Option ID = 2698]
For a polymer, which of the following statement/s is/are true? [Question ID = 759]
1. Weight average molecular weight is almost always higher than the number average molecular weight [Option ID = 3035]
2. Formation of a polypeptide from its monomers (amino acids) is an example of addition polymerization [Option ID = 3034]
3. All of these [Option ID = 3036]
4. Vinyl polymerization is an example of condensation polymerization. [Option ID = 3033]
Correct Answer :-
Weight average molecular weight is almost always higher than the number average molecular weight [Option ID = 3035]
Quantum dots are [Question ID = 762]
1. Three dimensional [Option ID = 3048]
2. One dimensional [Option ID = 3046]
3. Two dimensional [Option ID = 3047]
4. Zero dimensional [Option ID = 3045]
Correct Answer :-
Zero dimensional [Option ID = 3045]
The unit of rate constant for a third order reaction is: [Question ID = 749]
1. [Option ID = 2993]
2. [Option ID = 2995]
3. [Option ID = 2996]
4. [Option ID = 2994]
Correct Answer :-
[Option ID = 2996]
All the following elements have at least one isotope that is not radioactive EXCEPT [Question ID = 673]
1. Pb [Option ID = 2690]
2. O [Option ID = 2689]
3. Sn [Option ID = 2691]
4. No [Option ID = 2692]
Correct Answer :-
No [Option ID = 2692]
The conditions for a species to follow Bose-Einstein statistics are; [Question ID = 736]
1. Particles are indistinguishable, with no restriction on filling up of energy levels [Option ID = 2944]
2. Particles are indistinguishable, with a restriction on filling up of energy levels [Option ID = 2943]
3. Particles are distinguishable, with a restriction on filling up of energy levels [Option ID = 2941]
4. Particles are distinguishable, with no restriction on filling up of energy levels [Option ID = 2942]
Correct Answer :-
Particles are indistinguishable, with no restriction on filling up of energy levels [Option ID = 2944]
In the kinetic theory of collisions, the SI unit of collision number, in terms of m (meter) and s (second), is:
[Question ID = 761]
25)
26)
27)
28)
29)
1. m-
2
s
-1
[Option ID = 3042]
2. m
4
s
-1
[Option ID = 3041]
3. m
2
s
-1
[Option ID = 3043]
4. None of these [Option ID = 3044]
Correct Answer :-
None of these [Option ID = 3044]
Correct characteristics of the functional groups of adenine in DNA base pair are [Question ID = 706]
1. [Option ID = 2824]
2. [Option ID = 2821]
3. [Option ID = 2823]
4. [Option ID = 2822]
Correct Answer :-
[Option ID = 2822]
The carbon monoxide molecule has an internuclear distance of 1.13 Angstroms. What is the moment of Inertia of this molecule?
[Question ID = 740]
1. [Option ID = 2960]
2. [Option ID = 2957]
3. [Option ID = 2958]
4. [Option ID = 2959]
Correct Answer :-
[Option ID = 2957]
Which of the following represent/s non-linear optical technique? [Question ID = 744]
1. Second Harmonic generation [Option ID = 2974]
2. Two-photon photoluminescence [Option ID = 2975]
3. Four-wave mixing [Option ID = 2973]
4. All of these [Option ID = 2976]
Correct Answer :-
All of these [Option ID = 2976]
. Which of the following does not affect the intensity of spectral lines of a sample? [Question ID = 743]
1. Path length of a sample [Option ID = 2972]
2. Population of energy states [Option ID = 2970]
3. Heisenberg?s Uncertainty principle [Option ID = 2971]
4. Concentration of a sample [Option ID = 2969]
Correct Answer :-
Heisenberg?s Uncertainty principle [Option ID = 2971]
[Question ID = 716]
FirstRanker.com - FirstRanker's Choice
1)
2)
3)
4)
5)
DU PhD in Chemistry
Topic:- DU_J18_PHD_CHEM
Which of the following statements about sulfur dioxide is true?
[Question ID = 677]
1. It forms a S-S dimer in condensed phase [Option ID = 2707]
2. Its anhydride of sulfuric acid [Option ID = 2706]
3. Its O-S-O angle is 180
0
[Option ID = 2708]
4. It is a product of the combustion of fossil fuels that contain sulfur [Option ID = 2705]
Correct Answer :-
It is a product of the combustion of fossil fuels that contain sulfur [Option ID = 2705]
Which of the following is a strong acid in pure liquid HF
[Question ID = 683]
1. H
2
O [Option ID = 2731]
2. NaF [Option ID = 2729]
3. CH
3
COOH [Option ID = 2730]
4. SbF
5
[Option ID = 2732]
Correct Answer :-
SbF
5
[Option ID = 2732]
Each of the following molecules can act as a chelating agent EXCEPT [Question ID = 679]
1. [Option ID = 2716]
2. [Option ID = 2714]
3. [Option ID = 2713]
4. [Option ID = 2715]
Correct Answer :-
[Option ID = 2714]
What is correct about h-index?
[Question ID = 758]
1. Alternative of impact factor [Option ID = 3031]
2. Based on most quoted papers [Option ID = 3030]
3. Quantify scientific productivity [Option ID = 3029]
4. All of these [Option ID = 3032]
Correct Answer :-
All of these [Option ID = 3032]
The hyperfine electron spin resonance (e.s.r.) spectrum of the benzene radical has how many lines? [Question ID = 748]
1. 12 [Option ID = 2992]
2. 7 [Option ID = 2990]
3. 1 [Option ID = 2991]
4. 6 [Option ID = 2989]
Correct Answer :-
7 [Option ID = 2990]
6)
7)
8)
9)
10)
11)
12)
The energy changes involving the core electrons of an atom or molecule are expressed in which region of the electromagnetic
spectrum? [Question ID = 742]
1. Ultraviolet and Visible region [Option ID = 2967]
2. X-ray region [Option ID = 2968]
3. Radiofrequency region [Option ID = 2966]
4. Infra-red region [Option ID = 2965]
Correct Answer :-
X-ray region [Option ID = 2968]
Find out the expected intensity ratio of M and M+1 signal for the Naphthalene molecular ion [Question ID = 726]
1. 99:1.1 [Option ID = 2903]
2. 1.1:99 [Option ID = 2904]
3. 9:01 [Option ID = 2901]
4. 1:9 [Option ID = 2902]
Correct Answer :-
9:01 [Option ID = 2901]
Cobalt-60 is used in radiation therapy of cancer and can be produced by the bombardment of Cobalt-59 with [Question ID = 692]
1. Alpha particles [Option ID = 2765]
2. Beta particles [Option ID = 2767]
3. Neutrons [Option ID = 2766]
4. Gamma rays [Option ID = 2768]
Correct Answer :-
Neutrons [Option ID = 2766]
The standard emf of galvanic cell involving 3 moles of electrons in its redox reaction is 0.59 V. The equilibrium constant for the
reaction of the cell is- [Question ID = 763]
1. [Option ID = 3051]
2. [Option ID = 3052]
3. [Option ID = 3049]
4. [Option ID = 3050]
Correct Answer :-
[Option ID = 3052]
A characteristic common to polymers that can be made to conduct electricity such as polyacetylene, polypyrrole is: [Question ID =
685]
1. Conjugation throughout the polymeric chain. [Option ID = 2740]
2. A high degree of cross linking [Option ID = 2738]
3. A very low glass transition temperature [Option ID = 2737]
4. Presence of stereogenic centers of the same configuration [Option ID = 2739]
Correct Answer :-
Conjugation throughout the polymeric chain. [Option ID = 2740]
Impact factor is [Question ID = 768]
1. Ratio between citations and recent citable items publish [Option ID = 3071]
2. All of these [Option ID = 3072]
3. Addition of citations and recent citable items publish [Option ID = 3069]
4. Ratio between recent citable items publish and citations [Option ID = 3070]
Correct Answer :-
Ratio between recent citable items publish and citations [Option ID = 3070]
On the basis of oxidation-reduction potential, which of the following is most likely to occur? [Question ID = 693]
13)
14)
15)
16)
17)
1. [Option ID = 2770]
2. [Option ID = 2772]
3. [Option ID = 2771]
4. [Option ID = 2769]
Correct Answer :-
[Option ID = 2769]
How many diastereoisomers are possible for the compound 2, 4 ?diphenylcyclobutane-1, 3 di carboxylic acids. [Question ID = 725]
1. 6 [Option ID = 2899]
2. 5 [Option ID = 2898]
3. 8 [Option ID = 2900]
4. 4 [Option ID = 2897]
Correct Answer :-
5 [Option ID = 2898]
An increase in equivalent conductance of a strong electrolyte with dilution is mainly due to- [Question ID = 764]
1. increase in ionic mobility of ions [Option ID = 3055]
2. increase in number of ions [Option ID = 3054]
3. 100% ionization of electrolyte at normal dilution [Option ID = 3056]
4. increase in both i.e. number of ions and ionic mobility of ions. [Option ID = 3053]
Correct Answer :-
increase in ionic mobility of ions [Option ID = 3055]
The solid state structures of the principal allotropes of elemental boron are made up of which of the following structural units
[Question ID = 699]
1. [Option ID = 2796]
2. [Option ID = 2795]
3. [Option ID = 2794]
4. [Option ID = 2793]
Correct Answer :-
[Option ID = 2793]
The molecular geometry of thionyl chloride is best described as [Question ID = 688]
1. T-shaped [Option ID = 2752]
2. Tetrahedral [Option ID = 2751]
3. Trigonal pyramidal [Option ID = 2749]
4. Trigonal planar [Option ID = 2750]
Correct Answer :-
Trigonal pyramidal [Option ID = 2749]
In a face-center cubic (FCC) type of crystal lattice, the number of atoms belonging exclusively to each unit cell within the lattice
is/are: [Question ID = 754]
1. 2 [Option ID = 3014]
2. 1 [Option ID = 3013]
3. 3 [Option ID = 3015]
4. 4 [Option ID = 3016]
Correct Answer :-
4 [Option ID = 3016]
18)
19)
20)
21)
22)
23)
24)
Among the following, the weakest oxidizing agent is [Question ID = 675]
1. Mg (s) [Option ID = 2698]
2. [Option ID = 2699]
3. [Option ID = 2700]
4. [Option ID = 2697]
Correct Answer :-
Mg (s) [Option ID = 2698]
For a polymer, which of the following statement/s is/are true? [Question ID = 759]
1. Weight average molecular weight is almost always higher than the number average molecular weight [Option ID = 3035]
2. Formation of a polypeptide from its monomers (amino acids) is an example of addition polymerization [Option ID = 3034]
3. All of these [Option ID = 3036]
4. Vinyl polymerization is an example of condensation polymerization. [Option ID = 3033]
Correct Answer :-
Weight average molecular weight is almost always higher than the number average molecular weight [Option ID = 3035]
Quantum dots are [Question ID = 762]
1. Three dimensional [Option ID = 3048]
2. One dimensional [Option ID = 3046]
3. Two dimensional [Option ID = 3047]
4. Zero dimensional [Option ID = 3045]
Correct Answer :-
Zero dimensional [Option ID = 3045]
The unit of rate constant for a third order reaction is: [Question ID = 749]
1. [Option ID = 2993]
2. [Option ID = 2995]
3. [Option ID = 2996]
4. [Option ID = 2994]
Correct Answer :-
[Option ID = 2996]
All the following elements have at least one isotope that is not radioactive EXCEPT [Question ID = 673]
1. Pb [Option ID = 2690]
2. O [Option ID = 2689]
3. Sn [Option ID = 2691]
4. No [Option ID = 2692]
Correct Answer :-
No [Option ID = 2692]
The conditions for a species to follow Bose-Einstein statistics are; [Question ID = 736]
1. Particles are indistinguishable, with no restriction on filling up of energy levels [Option ID = 2944]
2. Particles are indistinguishable, with a restriction on filling up of energy levels [Option ID = 2943]
3. Particles are distinguishable, with a restriction on filling up of energy levels [Option ID = 2941]
4. Particles are distinguishable, with no restriction on filling up of energy levels [Option ID = 2942]
Correct Answer :-
Particles are indistinguishable, with no restriction on filling up of energy levels [Option ID = 2944]
In the kinetic theory of collisions, the SI unit of collision number, in terms of m (meter) and s (second), is:
[Question ID = 761]
25)
26)
27)
28)
29)
1. m-
2
s
-1
[Option ID = 3042]
2. m
4
s
-1
[Option ID = 3041]
3. m
2
s
-1
[Option ID = 3043]
4. None of these [Option ID = 3044]
Correct Answer :-
None of these [Option ID = 3044]
Correct characteristics of the functional groups of adenine in DNA base pair are [Question ID = 706]
1. [Option ID = 2824]
2. [Option ID = 2821]
3. [Option ID = 2823]
4. [Option ID = 2822]
Correct Answer :-
[Option ID = 2822]
The carbon monoxide molecule has an internuclear distance of 1.13 Angstroms. What is the moment of Inertia of this molecule?
[Question ID = 740]
1. [Option ID = 2960]
2. [Option ID = 2957]
3. [Option ID = 2958]
4. [Option ID = 2959]
Correct Answer :-
[Option ID = 2957]
Which of the following represent/s non-linear optical technique? [Question ID = 744]
1. Second Harmonic generation [Option ID = 2974]
2. Two-photon photoluminescence [Option ID = 2975]
3. Four-wave mixing [Option ID = 2973]
4. All of these [Option ID = 2976]
Correct Answer :-
All of these [Option ID = 2976]
. Which of the following does not affect the intensity of spectral lines of a sample? [Question ID = 743]
1. Path length of a sample [Option ID = 2972]
2. Population of energy states [Option ID = 2970]
3. Heisenberg?s Uncertainty principle [Option ID = 2971]
4. Concentration of a sample [Option ID = 2969]
Correct Answer :-
Heisenberg?s Uncertainty principle [Option ID = 2971]
[Question ID = 716]
30)
31)
1. [Option ID = 2862]
2. [Option ID = 2863]
3. [Option ID = 2861]
4. [Option ID = 2864]
Correct Answer :-
[Option ID = 2864]
[Question ID = 712]
1. [Option ID = 2845]
2. [Option ID = 2846]
3. [Option ID = 2848]
4. [Option ID = 2847]
Correct Answer :-
[Option ID = 2847]
[Question ID = 686]
1. [Option ID = 2743]
2. [Option ID = 2742]
3. [Option ID = 2741]
FirstRanker.com - FirstRanker's Choice
1)
2)
3)
4)
5)
DU PhD in Chemistry
Topic:- DU_J18_PHD_CHEM
Which of the following statements about sulfur dioxide is true?
[Question ID = 677]
1. It forms a S-S dimer in condensed phase [Option ID = 2707]
2. Its anhydride of sulfuric acid [Option ID = 2706]
3. Its O-S-O angle is 180
0
[Option ID = 2708]
4. It is a product of the combustion of fossil fuels that contain sulfur [Option ID = 2705]
Correct Answer :-
It is a product of the combustion of fossil fuels that contain sulfur [Option ID = 2705]
Which of the following is a strong acid in pure liquid HF
[Question ID = 683]
1. H
2
O [Option ID = 2731]
2. NaF [Option ID = 2729]
3. CH
3
COOH [Option ID = 2730]
4. SbF
5
[Option ID = 2732]
Correct Answer :-
SbF
5
[Option ID = 2732]
Each of the following molecules can act as a chelating agent EXCEPT [Question ID = 679]
1. [Option ID = 2716]
2. [Option ID = 2714]
3. [Option ID = 2713]
4. [Option ID = 2715]
Correct Answer :-
[Option ID = 2714]
What is correct about h-index?
[Question ID = 758]
1. Alternative of impact factor [Option ID = 3031]
2. Based on most quoted papers [Option ID = 3030]
3. Quantify scientific productivity [Option ID = 3029]
4. All of these [Option ID = 3032]
Correct Answer :-
All of these [Option ID = 3032]
The hyperfine electron spin resonance (e.s.r.) spectrum of the benzene radical has how many lines? [Question ID = 748]
1. 12 [Option ID = 2992]
2. 7 [Option ID = 2990]
3. 1 [Option ID = 2991]
4. 6 [Option ID = 2989]
Correct Answer :-
7 [Option ID = 2990]
6)
7)
8)
9)
10)
11)
12)
The energy changes involving the core electrons of an atom or molecule are expressed in which region of the electromagnetic
spectrum? [Question ID = 742]
1. Ultraviolet and Visible region [Option ID = 2967]
2. X-ray region [Option ID = 2968]
3. Radiofrequency region [Option ID = 2966]
4. Infra-red region [Option ID = 2965]
Correct Answer :-
X-ray region [Option ID = 2968]
Find out the expected intensity ratio of M and M+1 signal for the Naphthalene molecular ion [Question ID = 726]
1. 99:1.1 [Option ID = 2903]
2. 1.1:99 [Option ID = 2904]
3. 9:01 [Option ID = 2901]
4. 1:9 [Option ID = 2902]
Correct Answer :-
9:01 [Option ID = 2901]
Cobalt-60 is used in radiation therapy of cancer and can be produced by the bombardment of Cobalt-59 with [Question ID = 692]
1. Alpha particles [Option ID = 2765]
2. Beta particles [Option ID = 2767]
3. Neutrons [Option ID = 2766]
4. Gamma rays [Option ID = 2768]
Correct Answer :-
Neutrons [Option ID = 2766]
The standard emf of galvanic cell involving 3 moles of electrons in its redox reaction is 0.59 V. The equilibrium constant for the
reaction of the cell is- [Question ID = 763]
1. [Option ID = 3051]
2. [Option ID = 3052]
3. [Option ID = 3049]
4. [Option ID = 3050]
Correct Answer :-
[Option ID = 3052]
A characteristic common to polymers that can be made to conduct electricity such as polyacetylene, polypyrrole is: [Question ID =
685]
1. Conjugation throughout the polymeric chain. [Option ID = 2740]
2. A high degree of cross linking [Option ID = 2738]
3. A very low glass transition temperature [Option ID = 2737]
4. Presence of stereogenic centers of the same configuration [Option ID = 2739]
Correct Answer :-
Conjugation throughout the polymeric chain. [Option ID = 2740]
Impact factor is [Question ID = 768]
1. Ratio between citations and recent citable items publish [Option ID = 3071]
2. All of these [Option ID = 3072]
3. Addition of citations and recent citable items publish [Option ID = 3069]
4. Ratio between recent citable items publish and citations [Option ID = 3070]
Correct Answer :-
Ratio between recent citable items publish and citations [Option ID = 3070]
On the basis of oxidation-reduction potential, which of the following is most likely to occur? [Question ID = 693]
13)
14)
15)
16)
17)
1. [Option ID = 2770]
2. [Option ID = 2772]
3. [Option ID = 2771]
4. [Option ID = 2769]
Correct Answer :-
[Option ID = 2769]
How many diastereoisomers are possible for the compound 2, 4 ?diphenylcyclobutane-1, 3 di carboxylic acids. [Question ID = 725]
1. 6 [Option ID = 2899]
2. 5 [Option ID = 2898]
3. 8 [Option ID = 2900]
4. 4 [Option ID = 2897]
Correct Answer :-
5 [Option ID = 2898]
An increase in equivalent conductance of a strong electrolyte with dilution is mainly due to- [Question ID = 764]
1. increase in ionic mobility of ions [Option ID = 3055]
2. increase in number of ions [Option ID = 3054]
3. 100% ionization of electrolyte at normal dilution [Option ID = 3056]
4. increase in both i.e. number of ions and ionic mobility of ions. [Option ID = 3053]
Correct Answer :-
increase in ionic mobility of ions [Option ID = 3055]
The solid state structures of the principal allotropes of elemental boron are made up of which of the following structural units
[Question ID = 699]
1. [Option ID = 2796]
2. [Option ID = 2795]
3. [Option ID = 2794]
4. [Option ID = 2793]
Correct Answer :-
[Option ID = 2793]
The molecular geometry of thionyl chloride is best described as [Question ID = 688]
1. T-shaped [Option ID = 2752]
2. Tetrahedral [Option ID = 2751]
3. Trigonal pyramidal [Option ID = 2749]
4. Trigonal planar [Option ID = 2750]
Correct Answer :-
Trigonal pyramidal [Option ID = 2749]
In a face-center cubic (FCC) type of crystal lattice, the number of atoms belonging exclusively to each unit cell within the lattice
is/are: [Question ID = 754]
1. 2 [Option ID = 3014]
2. 1 [Option ID = 3013]
3. 3 [Option ID = 3015]
4. 4 [Option ID = 3016]
Correct Answer :-
4 [Option ID = 3016]
18)
19)
20)
21)
22)
23)
24)
Among the following, the weakest oxidizing agent is [Question ID = 675]
1. Mg (s) [Option ID = 2698]
2. [Option ID = 2699]
3. [Option ID = 2700]
4. [Option ID = 2697]
Correct Answer :-
Mg (s) [Option ID = 2698]
For a polymer, which of the following statement/s is/are true? [Question ID = 759]
1. Weight average molecular weight is almost always higher than the number average molecular weight [Option ID = 3035]
2. Formation of a polypeptide from its monomers (amino acids) is an example of addition polymerization [Option ID = 3034]
3. All of these [Option ID = 3036]
4. Vinyl polymerization is an example of condensation polymerization. [Option ID = 3033]
Correct Answer :-
Weight average molecular weight is almost always higher than the number average molecular weight [Option ID = 3035]
Quantum dots are [Question ID = 762]
1. Three dimensional [Option ID = 3048]
2. One dimensional [Option ID = 3046]
3. Two dimensional [Option ID = 3047]
4. Zero dimensional [Option ID = 3045]
Correct Answer :-
Zero dimensional [Option ID = 3045]
The unit of rate constant for a third order reaction is: [Question ID = 749]
1. [Option ID = 2993]
2. [Option ID = 2995]
3. [Option ID = 2996]
4. [Option ID = 2994]
Correct Answer :-
[Option ID = 2996]
All the following elements have at least one isotope that is not radioactive EXCEPT [Question ID = 673]
1. Pb [Option ID = 2690]
2. O [Option ID = 2689]
3. Sn [Option ID = 2691]
4. No [Option ID = 2692]
Correct Answer :-
No [Option ID = 2692]
The conditions for a species to follow Bose-Einstein statistics are; [Question ID = 736]
1. Particles are indistinguishable, with no restriction on filling up of energy levels [Option ID = 2944]
2. Particles are indistinguishable, with a restriction on filling up of energy levels [Option ID = 2943]
3. Particles are distinguishable, with a restriction on filling up of energy levels [Option ID = 2941]
4. Particles are distinguishable, with no restriction on filling up of energy levels [Option ID = 2942]
Correct Answer :-
Particles are indistinguishable, with no restriction on filling up of energy levels [Option ID = 2944]
In the kinetic theory of collisions, the SI unit of collision number, in terms of m (meter) and s (second), is:
[Question ID = 761]
25)
26)
27)
28)
29)
1. m-
2
s
-1
[Option ID = 3042]
2. m
4
s
-1
[Option ID = 3041]
3. m
2
s
-1
[Option ID = 3043]
4. None of these [Option ID = 3044]
Correct Answer :-
None of these [Option ID = 3044]
Correct characteristics of the functional groups of adenine in DNA base pair are [Question ID = 706]
1. [Option ID = 2824]
2. [Option ID = 2821]
3. [Option ID = 2823]
4. [Option ID = 2822]
Correct Answer :-
[Option ID = 2822]
The carbon monoxide molecule has an internuclear distance of 1.13 Angstroms. What is the moment of Inertia of this molecule?
[Question ID = 740]
1. [Option ID = 2960]
2. [Option ID = 2957]
3. [Option ID = 2958]
4. [Option ID = 2959]
Correct Answer :-
[Option ID = 2957]
Which of the following represent/s non-linear optical technique? [Question ID = 744]
1. Second Harmonic generation [Option ID = 2974]
2. Two-photon photoluminescence [Option ID = 2975]
3. Four-wave mixing [Option ID = 2973]
4. All of these [Option ID = 2976]
Correct Answer :-
All of these [Option ID = 2976]
. Which of the following does not affect the intensity of spectral lines of a sample? [Question ID = 743]
1. Path length of a sample [Option ID = 2972]
2. Population of energy states [Option ID = 2970]
3. Heisenberg?s Uncertainty principle [Option ID = 2971]
4. Concentration of a sample [Option ID = 2969]
Correct Answer :-
Heisenberg?s Uncertainty principle [Option ID = 2971]
[Question ID = 716]
30)
31)
1. [Option ID = 2862]
2. [Option ID = 2863]
3. [Option ID = 2861]
4. [Option ID = 2864]
Correct Answer :-
[Option ID = 2864]
[Question ID = 712]
1. [Option ID = 2845]
2. [Option ID = 2846]
3. [Option ID = 2848]
4. [Option ID = 2847]
Correct Answer :-
[Option ID = 2847]
[Question ID = 686]
1. [Option ID = 2743]
2. [Option ID = 2742]
3. [Option ID = 2741]
32)
33)
4. [Option ID = 2744]
Correct Answer :-
[Option ID = 2743]
[Question ID = 5633]
1. [Option ID = 22523]
2. [Option ID = 22524]
3. [Option ID = 22526]
4. [Option ID = 22525]
Correct Answer :-
[Option ID = 22523]
[Question ID = 730]
1. [Option ID = 2918]
2. [Option ID = 2920]
3. [Option ID = 2917]
4. [Option ID = 2919]
Correct Answer :-
FirstRanker.com - FirstRanker's Choice
1)
2)
3)
4)
5)
DU PhD in Chemistry
Topic:- DU_J18_PHD_CHEM
Which of the following statements about sulfur dioxide is true?
[Question ID = 677]
1. It forms a S-S dimer in condensed phase [Option ID = 2707]
2. Its anhydride of sulfuric acid [Option ID = 2706]
3. Its O-S-O angle is 180
0
[Option ID = 2708]
4. It is a product of the combustion of fossil fuels that contain sulfur [Option ID = 2705]
Correct Answer :-
It is a product of the combustion of fossil fuels that contain sulfur [Option ID = 2705]
Which of the following is a strong acid in pure liquid HF
[Question ID = 683]
1. H
2
O [Option ID = 2731]
2. NaF [Option ID = 2729]
3. CH
3
COOH [Option ID = 2730]
4. SbF
5
[Option ID = 2732]
Correct Answer :-
SbF
5
[Option ID = 2732]
Each of the following molecules can act as a chelating agent EXCEPT [Question ID = 679]
1. [Option ID = 2716]
2. [Option ID = 2714]
3. [Option ID = 2713]
4. [Option ID = 2715]
Correct Answer :-
[Option ID = 2714]
What is correct about h-index?
[Question ID = 758]
1. Alternative of impact factor [Option ID = 3031]
2. Based on most quoted papers [Option ID = 3030]
3. Quantify scientific productivity [Option ID = 3029]
4. All of these [Option ID = 3032]
Correct Answer :-
All of these [Option ID = 3032]
The hyperfine electron spin resonance (e.s.r.) spectrum of the benzene radical has how many lines? [Question ID = 748]
1. 12 [Option ID = 2992]
2. 7 [Option ID = 2990]
3. 1 [Option ID = 2991]
4. 6 [Option ID = 2989]
Correct Answer :-
7 [Option ID = 2990]
6)
7)
8)
9)
10)
11)
12)
The energy changes involving the core electrons of an atom or molecule are expressed in which region of the electromagnetic
spectrum? [Question ID = 742]
1. Ultraviolet and Visible region [Option ID = 2967]
2. X-ray region [Option ID = 2968]
3. Radiofrequency region [Option ID = 2966]
4. Infra-red region [Option ID = 2965]
Correct Answer :-
X-ray region [Option ID = 2968]
Find out the expected intensity ratio of M and M+1 signal for the Naphthalene molecular ion [Question ID = 726]
1. 99:1.1 [Option ID = 2903]
2. 1.1:99 [Option ID = 2904]
3. 9:01 [Option ID = 2901]
4. 1:9 [Option ID = 2902]
Correct Answer :-
9:01 [Option ID = 2901]
Cobalt-60 is used in radiation therapy of cancer and can be produced by the bombardment of Cobalt-59 with [Question ID = 692]
1. Alpha particles [Option ID = 2765]
2. Beta particles [Option ID = 2767]
3. Neutrons [Option ID = 2766]
4. Gamma rays [Option ID = 2768]
Correct Answer :-
Neutrons [Option ID = 2766]
The standard emf of galvanic cell involving 3 moles of electrons in its redox reaction is 0.59 V. The equilibrium constant for the
reaction of the cell is- [Question ID = 763]
1. [Option ID = 3051]
2. [Option ID = 3052]
3. [Option ID = 3049]
4. [Option ID = 3050]
Correct Answer :-
[Option ID = 3052]
A characteristic common to polymers that can be made to conduct electricity such as polyacetylene, polypyrrole is: [Question ID =
685]
1. Conjugation throughout the polymeric chain. [Option ID = 2740]
2. A high degree of cross linking [Option ID = 2738]
3. A very low glass transition temperature [Option ID = 2737]
4. Presence of stereogenic centers of the same configuration [Option ID = 2739]
Correct Answer :-
Conjugation throughout the polymeric chain. [Option ID = 2740]
Impact factor is [Question ID = 768]
1. Ratio between citations and recent citable items publish [Option ID = 3071]
2. All of these [Option ID = 3072]
3. Addition of citations and recent citable items publish [Option ID = 3069]
4. Ratio between recent citable items publish and citations [Option ID = 3070]
Correct Answer :-
Ratio between recent citable items publish and citations [Option ID = 3070]
On the basis of oxidation-reduction potential, which of the following is most likely to occur? [Question ID = 693]
13)
14)
15)
16)
17)
1. [Option ID = 2770]
2. [Option ID = 2772]
3. [Option ID = 2771]
4. [Option ID = 2769]
Correct Answer :-
[Option ID = 2769]
How many diastereoisomers are possible for the compound 2, 4 ?diphenylcyclobutane-1, 3 di carboxylic acids. [Question ID = 725]
1. 6 [Option ID = 2899]
2. 5 [Option ID = 2898]
3. 8 [Option ID = 2900]
4. 4 [Option ID = 2897]
Correct Answer :-
5 [Option ID = 2898]
An increase in equivalent conductance of a strong electrolyte with dilution is mainly due to- [Question ID = 764]
1. increase in ionic mobility of ions [Option ID = 3055]
2. increase in number of ions [Option ID = 3054]
3. 100% ionization of electrolyte at normal dilution [Option ID = 3056]
4. increase in both i.e. number of ions and ionic mobility of ions. [Option ID = 3053]
Correct Answer :-
increase in ionic mobility of ions [Option ID = 3055]
The solid state structures of the principal allotropes of elemental boron are made up of which of the following structural units
[Question ID = 699]
1. [Option ID = 2796]
2. [Option ID = 2795]
3. [Option ID = 2794]
4. [Option ID = 2793]
Correct Answer :-
[Option ID = 2793]
The molecular geometry of thionyl chloride is best described as [Question ID = 688]
1. T-shaped [Option ID = 2752]
2. Tetrahedral [Option ID = 2751]
3. Trigonal pyramidal [Option ID = 2749]
4. Trigonal planar [Option ID = 2750]
Correct Answer :-
Trigonal pyramidal [Option ID = 2749]
In a face-center cubic (FCC) type of crystal lattice, the number of atoms belonging exclusively to each unit cell within the lattice
is/are: [Question ID = 754]
1. 2 [Option ID = 3014]
2. 1 [Option ID = 3013]
3. 3 [Option ID = 3015]
4. 4 [Option ID = 3016]
Correct Answer :-
4 [Option ID = 3016]
18)
19)
20)
21)
22)
23)
24)
Among the following, the weakest oxidizing agent is [Question ID = 675]
1. Mg (s) [Option ID = 2698]
2. [Option ID = 2699]
3. [Option ID = 2700]
4. [Option ID = 2697]
Correct Answer :-
Mg (s) [Option ID = 2698]
For a polymer, which of the following statement/s is/are true? [Question ID = 759]
1. Weight average molecular weight is almost always higher than the number average molecular weight [Option ID = 3035]
2. Formation of a polypeptide from its monomers (amino acids) is an example of addition polymerization [Option ID = 3034]
3. All of these [Option ID = 3036]
4. Vinyl polymerization is an example of condensation polymerization. [Option ID = 3033]
Correct Answer :-
Weight average molecular weight is almost always higher than the number average molecular weight [Option ID = 3035]
Quantum dots are [Question ID = 762]
1. Three dimensional [Option ID = 3048]
2. One dimensional [Option ID = 3046]
3. Two dimensional [Option ID = 3047]
4. Zero dimensional [Option ID = 3045]
Correct Answer :-
Zero dimensional [Option ID = 3045]
The unit of rate constant for a third order reaction is: [Question ID = 749]
1. [Option ID = 2993]
2. [Option ID = 2995]
3. [Option ID = 2996]
4. [Option ID = 2994]
Correct Answer :-
[Option ID = 2996]
All the following elements have at least one isotope that is not radioactive EXCEPT [Question ID = 673]
1. Pb [Option ID = 2690]
2. O [Option ID = 2689]
3. Sn [Option ID = 2691]
4. No [Option ID = 2692]
Correct Answer :-
No [Option ID = 2692]
The conditions for a species to follow Bose-Einstein statistics are; [Question ID = 736]
1. Particles are indistinguishable, with no restriction on filling up of energy levels [Option ID = 2944]
2. Particles are indistinguishable, with a restriction on filling up of energy levels [Option ID = 2943]
3. Particles are distinguishable, with a restriction on filling up of energy levels [Option ID = 2941]
4. Particles are distinguishable, with no restriction on filling up of energy levels [Option ID = 2942]
Correct Answer :-
Particles are indistinguishable, with no restriction on filling up of energy levels [Option ID = 2944]
In the kinetic theory of collisions, the SI unit of collision number, in terms of m (meter) and s (second), is:
[Question ID = 761]
25)
26)
27)
28)
29)
1. m-
2
s
-1
[Option ID = 3042]
2. m
4
s
-1
[Option ID = 3041]
3. m
2
s
-1
[Option ID = 3043]
4. None of these [Option ID = 3044]
Correct Answer :-
None of these [Option ID = 3044]
Correct characteristics of the functional groups of adenine in DNA base pair are [Question ID = 706]
1. [Option ID = 2824]
2. [Option ID = 2821]
3. [Option ID = 2823]
4. [Option ID = 2822]
Correct Answer :-
[Option ID = 2822]
The carbon monoxide molecule has an internuclear distance of 1.13 Angstroms. What is the moment of Inertia of this molecule?
[Question ID = 740]
1. [Option ID = 2960]
2. [Option ID = 2957]
3. [Option ID = 2958]
4. [Option ID = 2959]
Correct Answer :-
[Option ID = 2957]
Which of the following represent/s non-linear optical technique? [Question ID = 744]
1. Second Harmonic generation [Option ID = 2974]
2. Two-photon photoluminescence [Option ID = 2975]
3. Four-wave mixing [Option ID = 2973]
4. All of these [Option ID = 2976]
Correct Answer :-
All of these [Option ID = 2976]
. Which of the following does not affect the intensity of spectral lines of a sample? [Question ID = 743]
1. Path length of a sample [Option ID = 2972]
2. Population of energy states [Option ID = 2970]
3. Heisenberg?s Uncertainty principle [Option ID = 2971]
4. Concentration of a sample [Option ID = 2969]
Correct Answer :-
Heisenberg?s Uncertainty principle [Option ID = 2971]
[Question ID = 716]
30)
31)
1. [Option ID = 2862]
2. [Option ID = 2863]
3. [Option ID = 2861]
4. [Option ID = 2864]
Correct Answer :-
[Option ID = 2864]
[Question ID = 712]
1. [Option ID = 2845]
2. [Option ID = 2846]
3. [Option ID = 2848]
4. [Option ID = 2847]
Correct Answer :-
[Option ID = 2847]
[Question ID = 686]
1. [Option ID = 2743]
2. [Option ID = 2742]
3. [Option ID = 2741]
32)
33)
4. [Option ID = 2744]
Correct Answer :-
[Option ID = 2743]
[Question ID = 5633]
1. [Option ID = 22523]
2. [Option ID = 22524]
3. [Option ID = 22526]
4. [Option ID = 22525]
Correct Answer :-
[Option ID = 22523]
[Question ID = 730]
1. [Option ID = 2918]
2. [Option ID = 2920]
3. [Option ID = 2917]
4. [Option ID = 2919]
Correct Answer :-
34)
35)
36)
[Option ID = 2919]
[Question ID = 714]
1. [Option ID = 2856]
2. [Option ID = 2853]
3. [Option ID = 2855]
4. [Option ID = 2854]
Correct Answer :-
[Option ID = 2854]
[Question ID = 669]
1. 2 [Option ID = 2673]
2. 18 [Option ID = 2676]
3. 5 [Option ID = 2674]
4. 10 [Option ID = 2675]
Correct Answer :-
5 [Option ID = 2674]
[Question ID = 689]
1. 2 [Option ID = 2754]
2. 0 [Option ID = 2756]
3. 1 [Option ID = 2755]
FirstRanker.com - FirstRanker's Choice
1)
2)
3)
4)
5)
DU PhD in Chemistry
Topic:- DU_J18_PHD_CHEM
Which of the following statements about sulfur dioxide is true?
[Question ID = 677]
1. It forms a S-S dimer in condensed phase [Option ID = 2707]
2. Its anhydride of sulfuric acid [Option ID = 2706]
3. Its O-S-O angle is 180
0
[Option ID = 2708]
4. It is a product of the combustion of fossil fuels that contain sulfur [Option ID = 2705]
Correct Answer :-
It is a product of the combustion of fossil fuels that contain sulfur [Option ID = 2705]
Which of the following is a strong acid in pure liquid HF
[Question ID = 683]
1. H
2
O [Option ID = 2731]
2. NaF [Option ID = 2729]
3. CH
3
COOH [Option ID = 2730]
4. SbF
5
[Option ID = 2732]
Correct Answer :-
SbF
5
[Option ID = 2732]
Each of the following molecules can act as a chelating agent EXCEPT [Question ID = 679]
1. [Option ID = 2716]
2. [Option ID = 2714]
3. [Option ID = 2713]
4. [Option ID = 2715]
Correct Answer :-
[Option ID = 2714]
What is correct about h-index?
[Question ID = 758]
1. Alternative of impact factor [Option ID = 3031]
2. Based on most quoted papers [Option ID = 3030]
3. Quantify scientific productivity [Option ID = 3029]
4. All of these [Option ID = 3032]
Correct Answer :-
All of these [Option ID = 3032]
The hyperfine electron spin resonance (e.s.r.) spectrum of the benzene radical has how many lines? [Question ID = 748]
1. 12 [Option ID = 2992]
2. 7 [Option ID = 2990]
3. 1 [Option ID = 2991]
4. 6 [Option ID = 2989]
Correct Answer :-
7 [Option ID = 2990]
6)
7)
8)
9)
10)
11)
12)
The energy changes involving the core electrons of an atom or molecule are expressed in which region of the electromagnetic
spectrum? [Question ID = 742]
1. Ultraviolet and Visible region [Option ID = 2967]
2. X-ray region [Option ID = 2968]
3. Radiofrequency region [Option ID = 2966]
4. Infra-red region [Option ID = 2965]
Correct Answer :-
X-ray region [Option ID = 2968]
Find out the expected intensity ratio of M and M+1 signal for the Naphthalene molecular ion [Question ID = 726]
1. 99:1.1 [Option ID = 2903]
2. 1.1:99 [Option ID = 2904]
3. 9:01 [Option ID = 2901]
4. 1:9 [Option ID = 2902]
Correct Answer :-
9:01 [Option ID = 2901]
Cobalt-60 is used in radiation therapy of cancer and can be produced by the bombardment of Cobalt-59 with [Question ID = 692]
1. Alpha particles [Option ID = 2765]
2. Beta particles [Option ID = 2767]
3. Neutrons [Option ID = 2766]
4. Gamma rays [Option ID = 2768]
Correct Answer :-
Neutrons [Option ID = 2766]
The standard emf of galvanic cell involving 3 moles of electrons in its redox reaction is 0.59 V. The equilibrium constant for the
reaction of the cell is- [Question ID = 763]
1. [Option ID = 3051]
2. [Option ID = 3052]
3. [Option ID = 3049]
4. [Option ID = 3050]
Correct Answer :-
[Option ID = 3052]
A characteristic common to polymers that can be made to conduct electricity such as polyacetylene, polypyrrole is: [Question ID =
685]
1. Conjugation throughout the polymeric chain. [Option ID = 2740]
2. A high degree of cross linking [Option ID = 2738]
3. A very low glass transition temperature [Option ID = 2737]
4. Presence of stereogenic centers of the same configuration [Option ID = 2739]
Correct Answer :-
Conjugation throughout the polymeric chain. [Option ID = 2740]
Impact factor is [Question ID = 768]
1. Ratio between citations and recent citable items publish [Option ID = 3071]
2. All of these [Option ID = 3072]
3. Addition of citations and recent citable items publish [Option ID = 3069]
4. Ratio between recent citable items publish and citations [Option ID = 3070]
Correct Answer :-
Ratio between recent citable items publish and citations [Option ID = 3070]
On the basis of oxidation-reduction potential, which of the following is most likely to occur? [Question ID = 693]
13)
14)
15)
16)
17)
1. [Option ID = 2770]
2. [Option ID = 2772]
3. [Option ID = 2771]
4. [Option ID = 2769]
Correct Answer :-
[Option ID = 2769]
How many diastereoisomers are possible for the compound 2, 4 ?diphenylcyclobutane-1, 3 di carboxylic acids. [Question ID = 725]
1. 6 [Option ID = 2899]
2. 5 [Option ID = 2898]
3. 8 [Option ID = 2900]
4. 4 [Option ID = 2897]
Correct Answer :-
5 [Option ID = 2898]
An increase in equivalent conductance of a strong electrolyte with dilution is mainly due to- [Question ID = 764]
1. increase in ionic mobility of ions [Option ID = 3055]
2. increase in number of ions [Option ID = 3054]
3. 100% ionization of electrolyte at normal dilution [Option ID = 3056]
4. increase in both i.e. number of ions and ionic mobility of ions. [Option ID = 3053]
Correct Answer :-
increase in ionic mobility of ions [Option ID = 3055]
The solid state structures of the principal allotropes of elemental boron are made up of which of the following structural units
[Question ID = 699]
1. [Option ID = 2796]
2. [Option ID = 2795]
3. [Option ID = 2794]
4. [Option ID = 2793]
Correct Answer :-
[Option ID = 2793]
The molecular geometry of thionyl chloride is best described as [Question ID = 688]
1. T-shaped [Option ID = 2752]
2. Tetrahedral [Option ID = 2751]
3. Trigonal pyramidal [Option ID = 2749]
4. Trigonal planar [Option ID = 2750]
Correct Answer :-
Trigonal pyramidal [Option ID = 2749]
In a face-center cubic (FCC) type of crystal lattice, the number of atoms belonging exclusively to each unit cell within the lattice
is/are: [Question ID = 754]
1. 2 [Option ID = 3014]
2. 1 [Option ID = 3013]
3. 3 [Option ID = 3015]
4. 4 [Option ID = 3016]
Correct Answer :-
4 [Option ID = 3016]
18)
19)
20)
21)
22)
23)
24)
Among the following, the weakest oxidizing agent is [Question ID = 675]
1. Mg (s) [Option ID = 2698]
2. [Option ID = 2699]
3. [Option ID = 2700]
4. [Option ID = 2697]
Correct Answer :-
Mg (s) [Option ID = 2698]
For a polymer, which of the following statement/s is/are true? [Question ID = 759]
1. Weight average molecular weight is almost always higher than the number average molecular weight [Option ID = 3035]
2. Formation of a polypeptide from its monomers (amino acids) is an example of addition polymerization [Option ID = 3034]
3. All of these [Option ID = 3036]
4. Vinyl polymerization is an example of condensation polymerization. [Option ID = 3033]
Correct Answer :-
Weight average molecular weight is almost always higher than the number average molecular weight [Option ID = 3035]
Quantum dots are [Question ID = 762]
1. Three dimensional [Option ID = 3048]
2. One dimensional [Option ID = 3046]
3. Two dimensional [Option ID = 3047]
4. Zero dimensional [Option ID = 3045]
Correct Answer :-
Zero dimensional [Option ID = 3045]
The unit of rate constant for a third order reaction is: [Question ID = 749]
1. [Option ID = 2993]
2. [Option ID = 2995]
3. [Option ID = 2996]
4. [Option ID = 2994]
Correct Answer :-
[Option ID = 2996]
All the following elements have at least one isotope that is not radioactive EXCEPT [Question ID = 673]
1. Pb [Option ID = 2690]
2. O [Option ID = 2689]
3. Sn [Option ID = 2691]
4. No [Option ID = 2692]
Correct Answer :-
No [Option ID = 2692]
The conditions for a species to follow Bose-Einstein statistics are; [Question ID = 736]
1. Particles are indistinguishable, with no restriction on filling up of energy levels [Option ID = 2944]
2. Particles are indistinguishable, with a restriction on filling up of energy levels [Option ID = 2943]
3. Particles are distinguishable, with a restriction on filling up of energy levels [Option ID = 2941]
4. Particles are distinguishable, with no restriction on filling up of energy levels [Option ID = 2942]
Correct Answer :-
Particles are indistinguishable, with no restriction on filling up of energy levels [Option ID = 2944]
In the kinetic theory of collisions, the SI unit of collision number, in terms of m (meter) and s (second), is:
[Question ID = 761]
25)
26)
27)
28)
29)
1. m-
2
s
-1
[Option ID = 3042]
2. m
4
s
-1
[Option ID = 3041]
3. m
2
s
-1
[Option ID = 3043]
4. None of these [Option ID = 3044]
Correct Answer :-
None of these [Option ID = 3044]
Correct characteristics of the functional groups of adenine in DNA base pair are [Question ID = 706]
1. [Option ID = 2824]
2. [Option ID = 2821]
3. [Option ID = 2823]
4. [Option ID = 2822]
Correct Answer :-
[Option ID = 2822]
The carbon monoxide molecule has an internuclear distance of 1.13 Angstroms. What is the moment of Inertia of this molecule?
[Question ID = 740]
1. [Option ID = 2960]
2. [Option ID = 2957]
3. [Option ID = 2958]
4. [Option ID = 2959]
Correct Answer :-
[Option ID = 2957]
Which of the following represent/s non-linear optical technique? [Question ID = 744]
1. Second Harmonic generation [Option ID = 2974]
2. Two-photon photoluminescence [Option ID = 2975]
3. Four-wave mixing [Option ID = 2973]
4. All of these [Option ID = 2976]
Correct Answer :-
All of these [Option ID = 2976]
. Which of the following does not affect the intensity of spectral lines of a sample? [Question ID = 743]
1. Path length of a sample [Option ID = 2972]
2. Population of energy states [Option ID = 2970]
3. Heisenberg?s Uncertainty principle [Option ID = 2971]
4. Concentration of a sample [Option ID = 2969]
Correct Answer :-
Heisenberg?s Uncertainty principle [Option ID = 2971]
[Question ID = 716]
30)
31)
1. [Option ID = 2862]
2. [Option ID = 2863]
3. [Option ID = 2861]
4. [Option ID = 2864]
Correct Answer :-
[Option ID = 2864]
[Question ID = 712]
1. [Option ID = 2845]
2. [Option ID = 2846]
3. [Option ID = 2848]
4. [Option ID = 2847]
Correct Answer :-
[Option ID = 2847]
[Question ID = 686]
1. [Option ID = 2743]
2. [Option ID = 2742]
3. [Option ID = 2741]
32)
33)
4. [Option ID = 2744]
Correct Answer :-
[Option ID = 2743]
[Question ID = 5633]
1. [Option ID = 22523]
2. [Option ID = 22524]
3. [Option ID = 22526]
4. [Option ID = 22525]
Correct Answer :-
[Option ID = 22523]
[Question ID = 730]
1. [Option ID = 2918]
2. [Option ID = 2920]
3. [Option ID = 2917]
4. [Option ID = 2919]
Correct Answer :-
34)
35)
36)
[Option ID = 2919]
[Question ID = 714]
1. [Option ID = 2856]
2. [Option ID = 2853]
3. [Option ID = 2855]
4. [Option ID = 2854]
Correct Answer :-
[Option ID = 2854]
[Question ID = 669]
1. 2 [Option ID = 2673]
2. 18 [Option ID = 2676]
3. 5 [Option ID = 2674]
4. 10 [Option ID = 2675]
Correct Answer :-
5 [Option ID = 2674]
[Question ID = 689]
1. 2 [Option ID = 2754]
2. 0 [Option ID = 2756]
3. 1 [Option ID = 2755]
37)
38)
4. 3 [Option ID = 2753]
Correct Answer :-
1 [Option ID = 2755]
[Question ID = 718]
1. C) [Option ID = 2871]
2. B) [Option ID = 2870]
3. A) [Option ID = 2869]
4. D) both (B) and (C) [Option ID = 2872]
Correct Answer :-
A) [Option ID = 2869]
[Question ID = 709]
1. [Option ID = 2836]
2. [Option ID = 2833]
3. [Option ID = 2835]
4. [Option ID = 2834]
FirstRanker.com - FirstRanker's Choice
1)
2)
3)
4)
5)
DU PhD in Chemistry
Topic:- DU_J18_PHD_CHEM
Which of the following statements about sulfur dioxide is true?
[Question ID = 677]
1. It forms a S-S dimer in condensed phase [Option ID = 2707]
2. Its anhydride of sulfuric acid [Option ID = 2706]
3. Its O-S-O angle is 180
0
[Option ID = 2708]
4. It is a product of the combustion of fossil fuels that contain sulfur [Option ID = 2705]
Correct Answer :-
It is a product of the combustion of fossil fuels that contain sulfur [Option ID = 2705]
Which of the following is a strong acid in pure liquid HF
[Question ID = 683]
1. H
2
O [Option ID = 2731]
2. NaF [Option ID = 2729]
3. CH
3
COOH [Option ID = 2730]
4. SbF
5
[Option ID = 2732]
Correct Answer :-
SbF
5
[Option ID = 2732]
Each of the following molecules can act as a chelating agent EXCEPT [Question ID = 679]
1. [Option ID = 2716]
2. [Option ID = 2714]
3. [Option ID = 2713]
4. [Option ID = 2715]
Correct Answer :-
[Option ID = 2714]
What is correct about h-index?
[Question ID = 758]
1. Alternative of impact factor [Option ID = 3031]
2. Based on most quoted papers [Option ID = 3030]
3. Quantify scientific productivity [Option ID = 3029]
4. All of these [Option ID = 3032]
Correct Answer :-
All of these [Option ID = 3032]
The hyperfine electron spin resonance (e.s.r.) spectrum of the benzene radical has how many lines? [Question ID = 748]
1. 12 [Option ID = 2992]
2. 7 [Option ID = 2990]
3. 1 [Option ID = 2991]
4. 6 [Option ID = 2989]
Correct Answer :-
7 [Option ID = 2990]
6)
7)
8)
9)
10)
11)
12)
The energy changes involving the core electrons of an atom or molecule are expressed in which region of the electromagnetic
spectrum? [Question ID = 742]
1. Ultraviolet and Visible region [Option ID = 2967]
2. X-ray region [Option ID = 2968]
3. Radiofrequency region [Option ID = 2966]
4. Infra-red region [Option ID = 2965]
Correct Answer :-
X-ray region [Option ID = 2968]
Find out the expected intensity ratio of M and M+1 signal for the Naphthalene molecular ion [Question ID = 726]
1. 99:1.1 [Option ID = 2903]
2. 1.1:99 [Option ID = 2904]
3. 9:01 [Option ID = 2901]
4. 1:9 [Option ID = 2902]
Correct Answer :-
9:01 [Option ID = 2901]
Cobalt-60 is used in radiation therapy of cancer and can be produced by the bombardment of Cobalt-59 with [Question ID = 692]
1. Alpha particles [Option ID = 2765]
2. Beta particles [Option ID = 2767]
3. Neutrons [Option ID = 2766]
4. Gamma rays [Option ID = 2768]
Correct Answer :-
Neutrons [Option ID = 2766]
The standard emf of galvanic cell involving 3 moles of electrons in its redox reaction is 0.59 V. The equilibrium constant for the
reaction of the cell is- [Question ID = 763]
1. [Option ID = 3051]
2. [Option ID = 3052]
3. [Option ID = 3049]
4. [Option ID = 3050]
Correct Answer :-
[Option ID = 3052]
A characteristic common to polymers that can be made to conduct electricity such as polyacetylene, polypyrrole is: [Question ID =
685]
1. Conjugation throughout the polymeric chain. [Option ID = 2740]
2. A high degree of cross linking [Option ID = 2738]
3. A very low glass transition temperature [Option ID = 2737]
4. Presence of stereogenic centers of the same configuration [Option ID = 2739]
Correct Answer :-
Conjugation throughout the polymeric chain. [Option ID = 2740]
Impact factor is [Question ID = 768]
1. Ratio between citations and recent citable items publish [Option ID = 3071]
2. All of these [Option ID = 3072]
3. Addition of citations and recent citable items publish [Option ID = 3069]
4. Ratio between recent citable items publish and citations [Option ID = 3070]
Correct Answer :-
Ratio between recent citable items publish and citations [Option ID = 3070]
On the basis of oxidation-reduction potential, which of the following is most likely to occur? [Question ID = 693]
13)
14)
15)
16)
17)
1. [Option ID = 2770]
2. [Option ID = 2772]
3. [Option ID = 2771]
4. [Option ID = 2769]
Correct Answer :-
[Option ID = 2769]
How many diastereoisomers are possible for the compound 2, 4 ?diphenylcyclobutane-1, 3 di carboxylic acids. [Question ID = 725]
1. 6 [Option ID = 2899]
2. 5 [Option ID = 2898]
3. 8 [Option ID = 2900]
4. 4 [Option ID = 2897]
Correct Answer :-
5 [Option ID = 2898]
An increase in equivalent conductance of a strong electrolyte with dilution is mainly due to- [Question ID = 764]
1. increase in ionic mobility of ions [Option ID = 3055]
2. increase in number of ions [Option ID = 3054]
3. 100% ionization of electrolyte at normal dilution [Option ID = 3056]
4. increase in both i.e. number of ions and ionic mobility of ions. [Option ID = 3053]
Correct Answer :-
increase in ionic mobility of ions [Option ID = 3055]
The solid state structures of the principal allotropes of elemental boron are made up of which of the following structural units
[Question ID = 699]
1. [Option ID = 2796]
2. [Option ID = 2795]
3. [Option ID = 2794]
4. [Option ID = 2793]
Correct Answer :-
[Option ID = 2793]
The molecular geometry of thionyl chloride is best described as [Question ID = 688]
1. T-shaped [Option ID = 2752]
2. Tetrahedral [Option ID = 2751]
3. Trigonal pyramidal [Option ID = 2749]
4. Trigonal planar [Option ID = 2750]
Correct Answer :-
Trigonal pyramidal [Option ID = 2749]
In a face-center cubic (FCC) type of crystal lattice, the number of atoms belonging exclusively to each unit cell within the lattice
is/are: [Question ID = 754]
1. 2 [Option ID = 3014]
2. 1 [Option ID = 3013]
3. 3 [Option ID = 3015]
4. 4 [Option ID = 3016]
Correct Answer :-
4 [Option ID = 3016]
18)
19)
20)
21)
22)
23)
24)
Among the following, the weakest oxidizing agent is [Question ID = 675]
1. Mg (s) [Option ID = 2698]
2. [Option ID = 2699]
3. [Option ID = 2700]
4. [Option ID = 2697]
Correct Answer :-
Mg (s) [Option ID = 2698]
For a polymer, which of the following statement/s is/are true? [Question ID = 759]
1. Weight average molecular weight is almost always higher than the number average molecular weight [Option ID = 3035]
2. Formation of a polypeptide from its monomers (amino acids) is an example of addition polymerization [Option ID = 3034]
3. All of these [Option ID = 3036]
4. Vinyl polymerization is an example of condensation polymerization. [Option ID = 3033]
Correct Answer :-
Weight average molecular weight is almost always higher than the number average molecular weight [Option ID = 3035]
Quantum dots are [Question ID = 762]
1. Three dimensional [Option ID = 3048]
2. One dimensional [Option ID = 3046]
3. Two dimensional [Option ID = 3047]
4. Zero dimensional [Option ID = 3045]
Correct Answer :-
Zero dimensional [Option ID = 3045]
The unit of rate constant for a third order reaction is: [Question ID = 749]
1. [Option ID = 2993]
2. [Option ID = 2995]
3. [Option ID = 2996]
4. [Option ID = 2994]
Correct Answer :-
[Option ID = 2996]
All the following elements have at least one isotope that is not radioactive EXCEPT [Question ID = 673]
1. Pb [Option ID = 2690]
2. O [Option ID = 2689]
3. Sn [Option ID = 2691]
4. No [Option ID = 2692]
Correct Answer :-
No [Option ID = 2692]
The conditions for a species to follow Bose-Einstein statistics are; [Question ID = 736]
1. Particles are indistinguishable, with no restriction on filling up of energy levels [Option ID = 2944]
2. Particles are indistinguishable, with a restriction on filling up of energy levels [Option ID = 2943]
3. Particles are distinguishable, with a restriction on filling up of energy levels [Option ID = 2941]
4. Particles are distinguishable, with no restriction on filling up of energy levels [Option ID = 2942]
Correct Answer :-
Particles are indistinguishable, with no restriction on filling up of energy levels [Option ID = 2944]
In the kinetic theory of collisions, the SI unit of collision number, in terms of m (meter) and s (second), is:
[Question ID = 761]
25)
26)
27)
28)
29)
1. m-
2
s
-1
[Option ID = 3042]
2. m
4
s
-1
[Option ID = 3041]
3. m
2
s
-1
[Option ID = 3043]
4. None of these [Option ID = 3044]
Correct Answer :-
None of these [Option ID = 3044]
Correct characteristics of the functional groups of adenine in DNA base pair are [Question ID = 706]
1. [Option ID = 2824]
2. [Option ID = 2821]
3. [Option ID = 2823]
4. [Option ID = 2822]
Correct Answer :-
[Option ID = 2822]
The carbon monoxide molecule has an internuclear distance of 1.13 Angstroms. What is the moment of Inertia of this molecule?
[Question ID = 740]
1. [Option ID = 2960]
2. [Option ID = 2957]
3. [Option ID = 2958]
4. [Option ID = 2959]
Correct Answer :-
[Option ID = 2957]
Which of the following represent/s non-linear optical technique? [Question ID = 744]
1. Second Harmonic generation [Option ID = 2974]
2. Two-photon photoluminescence [Option ID = 2975]
3. Four-wave mixing [Option ID = 2973]
4. All of these [Option ID = 2976]
Correct Answer :-
All of these [Option ID = 2976]
. Which of the following does not affect the intensity of spectral lines of a sample? [Question ID = 743]
1. Path length of a sample [Option ID = 2972]
2. Population of energy states [Option ID = 2970]
3. Heisenberg?s Uncertainty principle [Option ID = 2971]
4. Concentration of a sample [Option ID = 2969]
Correct Answer :-
Heisenberg?s Uncertainty principle [Option ID = 2971]
[Question ID = 716]
30)
31)
1. [Option ID = 2862]
2. [Option ID = 2863]
3. [Option ID = 2861]
4. [Option ID = 2864]
Correct Answer :-
[Option ID = 2864]
[Question ID = 712]
1. [Option ID = 2845]
2. [Option ID = 2846]
3. [Option ID = 2848]
4. [Option ID = 2847]
Correct Answer :-
[Option ID = 2847]
[Question ID = 686]
1. [Option ID = 2743]
2. [Option ID = 2742]
3. [Option ID = 2741]
32)
33)
4. [Option ID = 2744]
Correct Answer :-
[Option ID = 2743]
[Question ID = 5633]
1. [Option ID = 22523]
2. [Option ID = 22524]
3. [Option ID = 22526]
4. [Option ID = 22525]
Correct Answer :-
[Option ID = 22523]
[Question ID = 730]
1. [Option ID = 2918]
2. [Option ID = 2920]
3. [Option ID = 2917]
4. [Option ID = 2919]
Correct Answer :-
34)
35)
36)
[Option ID = 2919]
[Question ID = 714]
1. [Option ID = 2856]
2. [Option ID = 2853]
3. [Option ID = 2855]
4. [Option ID = 2854]
Correct Answer :-
[Option ID = 2854]
[Question ID = 669]
1. 2 [Option ID = 2673]
2. 18 [Option ID = 2676]
3. 5 [Option ID = 2674]
4. 10 [Option ID = 2675]
Correct Answer :-
5 [Option ID = 2674]
[Question ID = 689]
1. 2 [Option ID = 2754]
2. 0 [Option ID = 2756]
3. 1 [Option ID = 2755]
37)
38)
4. 3 [Option ID = 2753]
Correct Answer :-
1 [Option ID = 2755]
[Question ID = 718]
1. C) [Option ID = 2871]
2. B) [Option ID = 2870]
3. A) [Option ID = 2869]
4. D) both (B) and (C) [Option ID = 2872]
Correct Answer :-
A) [Option ID = 2869]
[Question ID = 709]
1. [Option ID = 2836]
2. [Option ID = 2833]
3. [Option ID = 2835]
4. [Option ID = 2834]
39)
40)
41)
42)
Correct Answer :-
[Option ID = 2836]
[Question ID = 705]
1. (2Z, 4Z)-3-chlorohexa-2, 4-diene-1,6-diol. [Option ID = 2819]
2. (2E, 4E)-3-chlorohexa-2, 4-diene-1,6-diol. [Option ID = 2817]
3. (2Z, 4E)-3-chlorohexa-2, 4-diene-1,6-diol. [Option ID = 2818]
4. (2Z, 4E)-3-chlorohexa-2, 4-diene-1,6-diol. [Option ID = 2820]
Correct Answer :-
(2Z, 4E)-3-chlorohexa-2, 4-diene-1,6-diol. [Option ID = 2820]
[Question ID = 680]
1. The presence of other complexing ligands in solution affects the equilibrium concentration of metal-EDTA complexes [Option ID = 2719]
2. Metal-EDTA complexes have an equilibrium concentration independent of pH [Option ID = 2718]
3. Metal-EDTA complexes are often 2:1 in stoichiometry [Option ID = 2717]
4. Metal-EDTA complexes are less stable than the corresponding metal-ammine complexes [Option ID = 2720]
Correct Answer :-
The presence of other complexing ligands in solution affects the equilibrium concentration of metal-EDTA complexes [Option ID = 2719]
[Question ID = 671]
1. 150 g [Option ID = 2682]
2. 300 g [Option ID = 2684]
3. 120 g [Option ID = 2681]
4. 180 g [Option ID = 2683]
Correct Answer :-
300 g [Option ID = 2684]
[Question ID = 756]
1. [Option ID = 3021]
2. [Option ID = 3022]
3. [Option ID = 3024]
4. [Option ID = 3023]
Correct Answer :-
FirstRanker.com - FirstRanker's Choice
1)
2)
3)
4)
5)
DU PhD in Chemistry
Topic:- DU_J18_PHD_CHEM
Which of the following statements about sulfur dioxide is true?
[Question ID = 677]
1. It forms a S-S dimer in condensed phase [Option ID = 2707]
2. Its anhydride of sulfuric acid [Option ID = 2706]
3. Its O-S-O angle is 180
0
[Option ID = 2708]
4. It is a product of the combustion of fossil fuels that contain sulfur [Option ID = 2705]
Correct Answer :-
It is a product of the combustion of fossil fuels that contain sulfur [Option ID = 2705]
Which of the following is a strong acid in pure liquid HF
[Question ID = 683]
1. H
2
O [Option ID = 2731]
2. NaF [Option ID = 2729]
3. CH
3
COOH [Option ID = 2730]
4. SbF
5
[Option ID = 2732]
Correct Answer :-
SbF
5
[Option ID = 2732]
Each of the following molecules can act as a chelating agent EXCEPT [Question ID = 679]
1. [Option ID = 2716]
2. [Option ID = 2714]
3. [Option ID = 2713]
4. [Option ID = 2715]
Correct Answer :-
[Option ID = 2714]
What is correct about h-index?
[Question ID = 758]
1. Alternative of impact factor [Option ID = 3031]
2. Based on most quoted papers [Option ID = 3030]
3. Quantify scientific productivity [Option ID = 3029]
4. All of these [Option ID = 3032]
Correct Answer :-
All of these [Option ID = 3032]
The hyperfine electron spin resonance (e.s.r.) spectrum of the benzene radical has how many lines? [Question ID = 748]
1. 12 [Option ID = 2992]
2. 7 [Option ID = 2990]
3. 1 [Option ID = 2991]
4. 6 [Option ID = 2989]
Correct Answer :-
7 [Option ID = 2990]
6)
7)
8)
9)
10)
11)
12)
The energy changes involving the core electrons of an atom or molecule are expressed in which region of the electromagnetic
spectrum? [Question ID = 742]
1. Ultraviolet and Visible region [Option ID = 2967]
2. X-ray region [Option ID = 2968]
3. Radiofrequency region [Option ID = 2966]
4. Infra-red region [Option ID = 2965]
Correct Answer :-
X-ray region [Option ID = 2968]
Find out the expected intensity ratio of M and M+1 signal for the Naphthalene molecular ion [Question ID = 726]
1. 99:1.1 [Option ID = 2903]
2. 1.1:99 [Option ID = 2904]
3. 9:01 [Option ID = 2901]
4. 1:9 [Option ID = 2902]
Correct Answer :-
9:01 [Option ID = 2901]
Cobalt-60 is used in radiation therapy of cancer and can be produced by the bombardment of Cobalt-59 with [Question ID = 692]
1. Alpha particles [Option ID = 2765]
2. Beta particles [Option ID = 2767]
3. Neutrons [Option ID = 2766]
4. Gamma rays [Option ID = 2768]
Correct Answer :-
Neutrons [Option ID = 2766]
The standard emf of galvanic cell involving 3 moles of electrons in its redox reaction is 0.59 V. The equilibrium constant for the
reaction of the cell is- [Question ID = 763]
1. [Option ID = 3051]
2. [Option ID = 3052]
3. [Option ID = 3049]
4. [Option ID = 3050]
Correct Answer :-
[Option ID = 3052]
A characteristic common to polymers that can be made to conduct electricity such as polyacetylene, polypyrrole is: [Question ID =
685]
1. Conjugation throughout the polymeric chain. [Option ID = 2740]
2. A high degree of cross linking [Option ID = 2738]
3. A very low glass transition temperature [Option ID = 2737]
4. Presence of stereogenic centers of the same configuration [Option ID = 2739]
Correct Answer :-
Conjugation throughout the polymeric chain. [Option ID = 2740]
Impact factor is [Question ID = 768]
1. Ratio between citations and recent citable items publish [Option ID = 3071]
2. All of these [Option ID = 3072]
3. Addition of citations and recent citable items publish [Option ID = 3069]
4. Ratio between recent citable items publish and citations [Option ID = 3070]
Correct Answer :-
Ratio between recent citable items publish and citations [Option ID = 3070]
On the basis of oxidation-reduction potential, which of the following is most likely to occur? [Question ID = 693]
13)
14)
15)
16)
17)
1. [Option ID = 2770]
2. [Option ID = 2772]
3. [Option ID = 2771]
4. [Option ID = 2769]
Correct Answer :-
[Option ID = 2769]
How many diastereoisomers are possible for the compound 2, 4 ?diphenylcyclobutane-1, 3 di carboxylic acids. [Question ID = 725]
1. 6 [Option ID = 2899]
2. 5 [Option ID = 2898]
3. 8 [Option ID = 2900]
4. 4 [Option ID = 2897]
Correct Answer :-
5 [Option ID = 2898]
An increase in equivalent conductance of a strong electrolyte with dilution is mainly due to- [Question ID = 764]
1. increase in ionic mobility of ions [Option ID = 3055]
2. increase in number of ions [Option ID = 3054]
3. 100% ionization of electrolyte at normal dilution [Option ID = 3056]
4. increase in both i.e. number of ions and ionic mobility of ions. [Option ID = 3053]
Correct Answer :-
increase in ionic mobility of ions [Option ID = 3055]
The solid state structures of the principal allotropes of elemental boron are made up of which of the following structural units
[Question ID = 699]
1. [Option ID = 2796]
2. [Option ID = 2795]
3. [Option ID = 2794]
4. [Option ID = 2793]
Correct Answer :-
[Option ID = 2793]
The molecular geometry of thionyl chloride is best described as [Question ID = 688]
1. T-shaped [Option ID = 2752]
2. Tetrahedral [Option ID = 2751]
3. Trigonal pyramidal [Option ID = 2749]
4. Trigonal planar [Option ID = 2750]
Correct Answer :-
Trigonal pyramidal [Option ID = 2749]
In a face-center cubic (FCC) type of crystal lattice, the number of atoms belonging exclusively to each unit cell within the lattice
is/are: [Question ID = 754]
1. 2 [Option ID = 3014]
2. 1 [Option ID = 3013]
3. 3 [Option ID = 3015]
4. 4 [Option ID = 3016]
Correct Answer :-
4 [Option ID = 3016]
18)
19)
20)
21)
22)
23)
24)
Among the following, the weakest oxidizing agent is [Question ID = 675]
1. Mg (s) [Option ID = 2698]
2. [Option ID = 2699]
3. [Option ID = 2700]
4. [Option ID = 2697]
Correct Answer :-
Mg (s) [Option ID = 2698]
For a polymer, which of the following statement/s is/are true? [Question ID = 759]
1. Weight average molecular weight is almost always higher than the number average molecular weight [Option ID = 3035]
2. Formation of a polypeptide from its monomers (amino acids) is an example of addition polymerization [Option ID = 3034]
3. All of these [Option ID = 3036]
4. Vinyl polymerization is an example of condensation polymerization. [Option ID = 3033]
Correct Answer :-
Weight average molecular weight is almost always higher than the number average molecular weight [Option ID = 3035]
Quantum dots are [Question ID = 762]
1. Three dimensional [Option ID = 3048]
2. One dimensional [Option ID = 3046]
3. Two dimensional [Option ID = 3047]
4. Zero dimensional [Option ID = 3045]
Correct Answer :-
Zero dimensional [Option ID = 3045]
The unit of rate constant for a third order reaction is: [Question ID = 749]
1. [Option ID = 2993]
2. [Option ID = 2995]
3. [Option ID = 2996]
4. [Option ID = 2994]
Correct Answer :-
[Option ID = 2996]
All the following elements have at least one isotope that is not radioactive EXCEPT [Question ID = 673]
1. Pb [Option ID = 2690]
2. O [Option ID = 2689]
3. Sn [Option ID = 2691]
4. No [Option ID = 2692]
Correct Answer :-
No [Option ID = 2692]
The conditions for a species to follow Bose-Einstein statistics are; [Question ID = 736]
1. Particles are indistinguishable, with no restriction on filling up of energy levels [Option ID = 2944]
2. Particles are indistinguishable, with a restriction on filling up of energy levels [Option ID = 2943]
3. Particles are distinguishable, with a restriction on filling up of energy levels [Option ID = 2941]
4. Particles are distinguishable, with no restriction on filling up of energy levels [Option ID = 2942]
Correct Answer :-
Particles are indistinguishable, with no restriction on filling up of energy levels [Option ID = 2944]
In the kinetic theory of collisions, the SI unit of collision number, in terms of m (meter) and s (second), is:
[Question ID = 761]
25)
26)
27)
28)
29)
1. m-
2
s
-1
[Option ID = 3042]
2. m
4
s
-1
[Option ID = 3041]
3. m
2
s
-1
[Option ID = 3043]
4. None of these [Option ID = 3044]
Correct Answer :-
None of these [Option ID = 3044]
Correct characteristics of the functional groups of adenine in DNA base pair are [Question ID = 706]
1. [Option ID = 2824]
2. [Option ID = 2821]
3. [Option ID = 2823]
4. [Option ID = 2822]
Correct Answer :-
[Option ID = 2822]
The carbon monoxide molecule has an internuclear distance of 1.13 Angstroms. What is the moment of Inertia of this molecule?
[Question ID = 740]
1. [Option ID = 2960]
2. [Option ID = 2957]
3. [Option ID = 2958]
4. [Option ID = 2959]
Correct Answer :-
[Option ID = 2957]
Which of the following represent/s non-linear optical technique? [Question ID = 744]
1. Second Harmonic generation [Option ID = 2974]
2. Two-photon photoluminescence [Option ID = 2975]
3. Four-wave mixing [Option ID = 2973]
4. All of these [Option ID = 2976]
Correct Answer :-
All of these [Option ID = 2976]
. Which of the following does not affect the intensity of spectral lines of a sample? [Question ID = 743]
1. Path length of a sample [Option ID = 2972]
2. Population of energy states [Option ID = 2970]
3. Heisenberg?s Uncertainty principle [Option ID = 2971]
4. Concentration of a sample [Option ID = 2969]
Correct Answer :-
Heisenberg?s Uncertainty principle [Option ID = 2971]
[Question ID = 716]
30)
31)
1. [Option ID = 2862]
2. [Option ID = 2863]
3. [Option ID = 2861]
4. [Option ID = 2864]
Correct Answer :-
[Option ID = 2864]
[Question ID = 712]
1. [Option ID = 2845]
2. [Option ID = 2846]
3. [Option ID = 2848]
4. [Option ID = 2847]
Correct Answer :-
[Option ID = 2847]
[Question ID = 686]
1. [Option ID = 2743]
2. [Option ID = 2742]
3. [Option ID = 2741]
32)
33)
4. [Option ID = 2744]
Correct Answer :-
[Option ID = 2743]
[Question ID = 5633]
1. [Option ID = 22523]
2. [Option ID = 22524]
3. [Option ID = 22526]
4. [Option ID = 22525]
Correct Answer :-
[Option ID = 22523]
[Question ID = 730]
1. [Option ID = 2918]
2. [Option ID = 2920]
3. [Option ID = 2917]
4. [Option ID = 2919]
Correct Answer :-
34)
35)
36)
[Option ID = 2919]
[Question ID = 714]
1. [Option ID = 2856]
2. [Option ID = 2853]
3. [Option ID = 2855]
4. [Option ID = 2854]
Correct Answer :-
[Option ID = 2854]
[Question ID = 669]
1. 2 [Option ID = 2673]
2. 18 [Option ID = 2676]
3. 5 [Option ID = 2674]
4. 10 [Option ID = 2675]
Correct Answer :-
5 [Option ID = 2674]
[Question ID = 689]
1. 2 [Option ID = 2754]
2. 0 [Option ID = 2756]
3. 1 [Option ID = 2755]
37)
38)
4. 3 [Option ID = 2753]
Correct Answer :-
1 [Option ID = 2755]
[Question ID = 718]
1. C) [Option ID = 2871]
2. B) [Option ID = 2870]
3. A) [Option ID = 2869]
4. D) both (B) and (C) [Option ID = 2872]
Correct Answer :-
A) [Option ID = 2869]
[Question ID = 709]
1. [Option ID = 2836]
2. [Option ID = 2833]
3. [Option ID = 2835]
4. [Option ID = 2834]
39)
40)
41)
42)
Correct Answer :-
[Option ID = 2836]
[Question ID = 705]
1. (2Z, 4Z)-3-chlorohexa-2, 4-diene-1,6-diol. [Option ID = 2819]
2. (2E, 4E)-3-chlorohexa-2, 4-diene-1,6-diol. [Option ID = 2817]
3. (2Z, 4E)-3-chlorohexa-2, 4-diene-1,6-diol. [Option ID = 2818]
4. (2Z, 4E)-3-chlorohexa-2, 4-diene-1,6-diol. [Option ID = 2820]
Correct Answer :-
(2Z, 4E)-3-chlorohexa-2, 4-diene-1,6-diol. [Option ID = 2820]
[Question ID = 680]
1. The presence of other complexing ligands in solution affects the equilibrium concentration of metal-EDTA complexes [Option ID = 2719]
2. Metal-EDTA complexes have an equilibrium concentration independent of pH [Option ID = 2718]
3. Metal-EDTA complexes are often 2:1 in stoichiometry [Option ID = 2717]
4. Metal-EDTA complexes are less stable than the corresponding metal-ammine complexes [Option ID = 2720]
Correct Answer :-
The presence of other complexing ligands in solution affects the equilibrium concentration of metal-EDTA complexes [Option ID = 2719]
[Question ID = 671]
1. 150 g [Option ID = 2682]
2. 300 g [Option ID = 2684]
3. 120 g [Option ID = 2681]
4. 180 g [Option ID = 2683]
Correct Answer :-
300 g [Option ID = 2684]
[Question ID = 756]
1. [Option ID = 3021]
2. [Option ID = 3022]
3. [Option ID = 3024]
4. [Option ID = 3023]
Correct Answer :-
43)
44)
45)
[Option ID = 3024]
[Question ID = 750]
1. [Option ID = 2999]
2. [Option ID = 3000]
3. [Option ID = 2997]
4. [Option ID = 2998]
Correct Answer :-
[Option ID = 2997]
[Question ID = 715]
1. [Option ID = 2857]
2. [Option ID = 2860]
3. [Option ID = 2858]
4. [Option ID = 2859]
Correct Answer :-
[Option ID = 2860]
FirstRanker.com - FirstRanker's Choice
1)
2)
3)
4)
5)
DU PhD in Chemistry
Topic:- DU_J18_PHD_CHEM
Which of the following statements about sulfur dioxide is true?
[Question ID = 677]
1. It forms a S-S dimer in condensed phase [Option ID = 2707]
2. Its anhydride of sulfuric acid [Option ID = 2706]
3. Its O-S-O angle is 180
0
[Option ID = 2708]
4. It is a product of the combustion of fossil fuels that contain sulfur [Option ID = 2705]
Correct Answer :-
It is a product of the combustion of fossil fuels that contain sulfur [Option ID = 2705]
Which of the following is a strong acid in pure liquid HF
[Question ID = 683]
1. H
2
O [Option ID = 2731]
2. NaF [Option ID = 2729]
3. CH
3
COOH [Option ID = 2730]
4. SbF
5
[Option ID = 2732]
Correct Answer :-
SbF
5
[Option ID = 2732]
Each of the following molecules can act as a chelating agent EXCEPT [Question ID = 679]
1. [Option ID = 2716]
2. [Option ID = 2714]
3. [Option ID = 2713]
4. [Option ID = 2715]
Correct Answer :-
[Option ID = 2714]
What is correct about h-index?
[Question ID = 758]
1. Alternative of impact factor [Option ID = 3031]
2. Based on most quoted papers [Option ID = 3030]
3. Quantify scientific productivity [Option ID = 3029]
4. All of these [Option ID = 3032]
Correct Answer :-
All of these [Option ID = 3032]
The hyperfine electron spin resonance (e.s.r.) spectrum of the benzene radical has how many lines? [Question ID = 748]
1. 12 [Option ID = 2992]
2. 7 [Option ID = 2990]
3. 1 [Option ID = 2991]
4. 6 [Option ID = 2989]
Correct Answer :-
7 [Option ID = 2990]
6)
7)
8)
9)
10)
11)
12)
The energy changes involving the core electrons of an atom or molecule are expressed in which region of the electromagnetic
spectrum? [Question ID = 742]
1. Ultraviolet and Visible region [Option ID = 2967]
2. X-ray region [Option ID = 2968]
3. Radiofrequency region [Option ID = 2966]
4. Infra-red region [Option ID = 2965]
Correct Answer :-
X-ray region [Option ID = 2968]
Find out the expected intensity ratio of M and M+1 signal for the Naphthalene molecular ion [Question ID = 726]
1. 99:1.1 [Option ID = 2903]
2. 1.1:99 [Option ID = 2904]
3. 9:01 [Option ID = 2901]
4. 1:9 [Option ID = 2902]
Correct Answer :-
9:01 [Option ID = 2901]
Cobalt-60 is used in radiation therapy of cancer and can be produced by the bombardment of Cobalt-59 with [Question ID = 692]
1. Alpha particles [Option ID = 2765]
2. Beta particles [Option ID = 2767]
3. Neutrons [Option ID = 2766]
4. Gamma rays [Option ID = 2768]
Correct Answer :-
Neutrons [Option ID = 2766]
The standard emf of galvanic cell involving 3 moles of electrons in its redox reaction is 0.59 V. The equilibrium constant for the
reaction of the cell is- [Question ID = 763]
1. [Option ID = 3051]
2. [Option ID = 3052]
3. [Option ID = 3049]
4. [Option ID = 3050]
Correct Answer :-
[Option ID = 3052]
A characteristic common to polymers that can be made to conduct electricity such as polyacetylene, polypyrrole is: [Question ID =
685]
1. Conjugation throughout the polymeric chain. [Option ID = 2740]
2. A high degree of cross linking [Option ID = 2738]
3. A very low glass transition temperature [Option ID = 2737]
4. Presence of stereogenic centers of the same configuration [Option ID = 2739]
Correct Answer :-
Conjugation throughout the polymeric chain. [Option ID = 2740]
Impact factor is [Question ID = 768]
1. Ratio between citations and recent citable items publish [Option ID = 3071]
2. All of these [Option ID = 3072]
3. Addition of citations and recent citable items publish [Option ID = 3069]
4. Ratio between recent citable items publish and citations [Option ID = 3070]
Correct Answer :-
Ratio between recent citable items publish and citations [Option ID = 3070]
On the basis of oxidation-reduction potential, which of the following is most likely to occur? [Question ID = 693]
13)
14)
15)
16)
17)
1. [Option ID = 2770]
2. [Option ID = 2772]
3. [Option ID = 2771]
4. [Option ID = 2769]
Correct Answer :-
[Option ID = 2769]
How many diastereoisomers are possible for the compound 2, 4 ?diphenylcyclobutane-1, 3 di carboxylic acids. [Question ID = 725]
1. 6 [Option ID = 2899]
2. 5 [Option ID = 2898]
3. 8 [Option ID = 2900]
4. 4 [Option ID = 2897]
Correct Answer :-
5 [Option ID = 2898]
An increase in equivalent conductance of a strong electrolyte with dilution is mainly due to- [Question ID = 764]
1. increase in ionic mobility of ions [Option ID = 3055]
2. increase in number of ions [Option ID = 3054]
3. 100% ionization of electrolyte at normal dilution [Option ID = 3056]
4. increase in both i.e. number of ions and ionic mobility of ions. [Option ID = 3053]
Correct Answer :-
increase in ionic mobility of ions [Option ID = 3055]
The solid state structures of the principal allotropes of elemental boron are made up of which of the following structural units
[Question ID = 699]
1. [Option ID = 2796]
2. [Option ID = 2795]
3. [Option ID = 2794]
4. [Option ID = 2793]
Correct Answer :-
[Option ID = 2793]
The molecular geometry of thionyl chloride is best described as [Question ID = 688]
1. T-shaped [Option ID = 2752]
2. Tetrahedral [Option ID = 2751]
3. Trigonal pyramidal [Option ID = 2749]
4. Trigonal planar [Option ID = 2750]
Correct Answer :-
Trigonal pyramidal [Option ID = 2749]
In a face-center cubic (FCC) type of crystal lattice, the number of atoms belonging exclusively to each unit cell within the lattice
is/are: [Question ID = 754]
1. 2 [Option ID = 3014]
2. 1 [Option ID = 3013]
3. 3 [Option ID = 3015]
4. 4 [Option ID = 3016]
Correct Answer :-
4 [Option ID = 3016]
18)
19)
20)
21)
22)
23)
24)
Among the following, the weakest oxidizing agent is [Question ID = 675]
1. Mg (s) [Option ID = 2698]
2. [Option ID = 2699]
3. [Option ID = 2700]
4. [Option ID = 2697]
Correct Answer :-
Mg (s) [Option ID = 2698]
For a polymer, which of the following statement/s is/are true? [Question ID = 759]
1. Weight average molecular weight is almost always higher than the number average molecular weight [Option ID = 3035]
2. Formation of a polypeptide from its monomers (amino acids) is an example of addition polymerization [Option ID = 3034]
3. All of these [Option ID = 3036]
4. Vinyl polymerization is an example of condensation polymerization. [Option ID = 3033]
Correct Answer :-
Weight average molecular weight is almost always higher than the number average molecular weight [Option ID = 3035]
Quantum dots are [Question ID = 762]
1. Three dimensional [Option ID = 3048]
2. One dimensional [Option ID = 3046]
3. Two dimensional [Option ID = 3047]
4. Zero dimensional [Option ID = 3045]
Correct Answer :-
Zero dimensional [Option ID = 3045]
The unit of rate constant for a third order reaction is: [Question ID = 749]
1. [Option ID = 2993]
2. [Option ID = 2995]
3. [Option ID = 2996]
4. [Option ID = 2994]
Correct Answer :-
[Option ID = 2996]
All the following elements have at least one isotope that is not radioactive EXCEPT [Question ID = 673]
1. Pb [Option ID = 2690]
2. O [Option ID = 2689]
3. Sn [Option ID = 2691]
4. No [Option ID = 2692]
Correct Answer :-
No [Option ID = 2692]
The conditions for a species to follow Bose-Einstein statistics are; [Question ID = 736]
1. Particles are indistinguishable, with no restriction on filling up of energy levels [Option ID = 2944]
2. Particles are indistinguishable, with a restriction on filling up of energy levels [Option ID = 2943]
3. Particles are distinguishable, with a restriction on filling up of energy levels [Option ID = 2941]
4. Particles are distinguishable, with no restriction on filling up of energy levels [Option ID = 2942]
Correct Answer :-
Particles are indistinguishable, with no restriction on filling up of energy levels [Option ID = 2944]
In the kinetic theory of collisions, the SI unit of collision number, in terms of m (meter) and s (second), is:
[Question ID = 761]
25)
26)
27)
28)
29)
1. m-
2
s
-1
[Option ID = 3042]
2. m
4
s
-1
[Option ID = 3041]
3. m
2
s
-1
[Option ID = 3043]
4. None of these [Option ID = 3044]
Correct Answer :-
None of these [Option ID = 3044]
Correct characteristics of the functional groups of adenine in DNA base pair are [Question ID = 706]
1. [Option ID = 2824]
2. [Option ID = 2821]
3. [Option ID = 2823]
4. [Option ID = 2822]
Correct Answer :-
[Option ID = 2822]
The carbon monoxide molecule has an internuclear distance of 1.13 Angstroms. What is the moment of Inertia of this molecule?
[Question ID = 740]
1. [Option ID = 2960]
2. [Option ID = 2957]
3. [Option ID = 2958]
4. [Option ID = 2959]
Correct Answer :-
[Option ID = 2957]
Which of the following represent/s non-linear optical technique? [Question ID = 744]
1. Second Harmonic generation [Option ID = 2974]
2. Two-photon photoluminescence [Option ID = 2975]
3. Four-wave mixing [Option ID = 2973]
4. All of these [Option ID = 2976]
Correct Answer :-
All of these [Option ID = 2976]
. Which of the following does not affect the intensity of spectral lines of a sample? [Question ID = 743]
1. Path length of a sample [Option ID = 2972]
2. Population of energy states [Option ID = 2970]
3. Heisenberg?s Uncertainty principle [Option ID = 2971]
4. Concentration of a sample [Option ID = 2969]
Correct Answer :-
Heisenberg?s Uncertainty principle [Option ID = 2971]
[Question ID = 716]
30)
31)
1. [Option ID = 2862]
2. [Option ID = 2863]
3. [Option ID = 2861]
4. [Option ID = 2864]
Correct Answer :-
[Option ID = 2864]
[Question ID = 712]
1. [Option ID = 2845]
2. [Option ID = 2846]
3. [Option ID = 2848]
4. [Option ID = 2847]
Correct Answer :-
[Option ID = 2847]
[Question ID = 686]
1. [Option ID = 2743]
2. [Option ID = 2742]
3. [Option ID = 2741]
32)
33)
4. [Option ID = 2744]
Correct Answer :-
[Option ID = 2743]
[Question ID = 5633]
1. [Option ID = 22523]
2. [Option ID = 22524]
3. [Option ID = 22526]
4. [Option ID = 22525]
Correct Answer :-
[Option ID = 22523]
[Question ID = 730]
1. [Option ID = 2918]
2. [Option ID = 2920]
3. [Option ID = 2917]
4. [Option ID = 2919]
Correct Answer :-
34)
35)
36)
[Option ID = 2919]
[Question ID = 714]
1. [Option ID = 2856]
2. [Option ID = 2853]
3. [Option ID = 2855]
4. [Option ID = 2854]
Correct Answer :-
[Option ID = 2854]
[Question ID = 669]
1. 2 [Option ID = 2673]
2. 18 [Option ID = 2676]
3. 5 [Option ID = 2674]
4. 10 [Option ID = 2675]
Correct Answer :-
5 [Option ID = 2674]
[Question ID = 689]
1. 2 [Option ID = 2754]
2. 0 [Option ID = 2756]
3. 1 [Option ID = 2755]
37)
38)
4. 3 [Option ID = 2753]
Correct Answer :-
1 [Option ID = 2755]
[Question ID = 718]
1. C) [Option ID = 2871]
2. B) [Option ID = 2870]
3. A) [Option ID = 2869]
4. D) both (B) and (C) [Option ID = 2872]
Correct Answer :-
A) [Option ID = 2869]
[Question ID = 709]
1. [Option ID = 2836]
2. [Option ID = 2833]
3. [Option ID = 2835]
4. [Option ID = 2834]
39)
40)
41)
42)
Correct Answer :-
[Option ID = 2836]
[Question ID = 705]
1. (2Z, 4Z)-3-chlorohexa-2, 4-diene-1,6-diol. [Option ID = 2819]
2. (2E, 4E)-3-chlorohexa-2, 4-diene-1,6-diol. [Option ID = 2817]
3. (2Z, 4E)-3-chlorohexa-2, 4-diene-1,6-diol. [Option ID = 2818]
4. (2Z, 4E)-3-chlorohexa-2, 4-diene-1,6-diol. [Option ID = 2820]
Correct Answer :-
(2Z, 4E)-3-chlorohexa-2, 4-diene-1,6-diol. [Option ID = 2820]
[Question ID = 680]
1. The presence of other complexing ligands in solution affects the equilibrium concentration of metal-EDTA complexes [Option ID = 2719]
2. Metal-EDTA complexes have an equilibrium concentration independent of pH [Option ID = 2718]
3. Metal-EDTA complexes are often 2:1 in stoichiometry [Option ID = 2717]
4. Metal-EDTA complexes are less stable than the corresponding metal-ammine complexes [Option ID = 2720]
Correct Answer :-
The presence of other complexing ligands in solution affects the equilibrium concentration of metal-EDTA complexes [Option ID = 2719]
[Question ID = 671]
1. 150 g [Option ID = 2682]
2. 300 g [Option ID = 2684]
3. 120 g [Option ID = 2681]
4. 180 g [Option ID = 2683]
Correct Answer :-
300 g [Option ID = 2684]
[Question ID = 756]
1. [Option ID = 3021]
2. [Option ID = 3022]
3. [Option ID = 3024]
4. [Option ID = 3023]
Correct Answer :-
43)
44)
45)
[Option ID = 3024]
[Question ID = 750]
1. [Option ID = 2999]
2. [Option ID = 3000]
3. [Option ID = 2997]
4. [Option ID = 2998]
Correct Answer :-
[Option ID = 2997]
[Question ID = 715]
1. [Option ID = 2857]
2. [Option ID = 2860]
3. [Option ID = 2858]
4. [Option ID = 2859]
Correct Answer :-
[Option ID = 2860]
46)
47)
[Question ID = 719]
1. [Option ID = 2873]
2. [Option ID = 2876]
3. [Option ID = 2874]
4. [Option ID = 2875]
Correct Answer :-
[Question ID = 674]
1. Evaporating some water to decrease the volume of the solution. [Option ID = 2696]
2. [Option ID = 2695]
3. [Option ID = 2694]
4. [Option ID = 2693]
Correct Answer :-
[Option ID = 2693]
[Question ID = 728]
1. [Option ID = 2910]
2. [Option ID = 2911]
3. [Option ID = 2909]
4. [Option ID = 2912]
FirstRanker.com - FirstRanker's Choice
1)
2)
3)
4)
5)
DU PhD in Chemistry
Topic:- DU_J18_PHD_CHEM
Which of the following statements about sulfur dioxide is true?
[Question ID = 677]
1. It forms a S-S dimer in condensed phase [Option ID = 2707]
2. Its anhydride of sulfuric acid [Option ID = 2706]
3. Its O-S-O angle is 180
0
[Option ID = 2708]
4. It is a product of the combustion of fossil fuels that contain sulfur [Option ID = 2705]
Correct Answer :-
It is a product of the combustion of fossil fuels that contain sulfur [Option ID = 2705]
Which of the following is a strong acid in pure liquid HF
[Question ID = 683]
1. H
2
O [Option ID = 2731]
2. NaF [Option ID = 2729]
3. CH
3
COOH [Option ID = 2730]
4. SbF
5
[Option ID = 2732]
Correct Answer :-
SbF
5
[Option ID = 2732]
Each of the following molecules can act as a chelating agent EXCEPT [Question ID = 679]
1. [Option ID = 2716]
2. [Option ID = 2714]
3. [Option ID = 2713]
4. [Option ID = 2715]
Correct Answer :-
[Option ID = 2714]
What is correct about h-index?
[Question ID = 758]
1. Alternative of impact factor [Option ID = 3031]
2. Based on most quoted papers [Option ID = 3030]
3. Quantify scientific productivity [Option ID = 3029]
4. All of these [Option ID = 3032]
Correct Answer :-
All of these [Option ID = 3032]
The hyperfine electron spin resonance (e.s.r.) spectrum of the benzene radical has how many lines? [Question ID = 748]
1. 12 [Option ID = 2992]
2. 7 [Option ID = 2990]
3. 1 [Option ID = 2991]
4. 6 [Option ID = 2989]
Correct Answer :-
7 [Option ID = 2990]
6)
7)
8)
9)
10)
11)
12)
The energy changes involving the core electrons of an atom or molecule are expressed in which region of the electromagnetic
spectrum? [Question ID = 742]
1. Ultraviolet and Visible region [Option ID = 2967]
2. X-ray region [Option ID = 2968]
3. Radiofrequency region [Option ID = 2966]
4. Infra-red region [Option ID = 2965]
Correct Answer :-
X-ray region [Option ID = 2968]
Find out the expected intensity ratio of M and M+1 signal for the Naphthalene molecular ion [Question ID = 726]
1. 99:1.1 [Option ID = 2903]
2. 1.1:99 [Option ID = 2904]
3. 9:01 [Option ID = 2901]
4. 1:9 [Option ID = 2902]
Correct Answer :-
9:01 [Option ID = 2901]
Cobalt-60 is used in radiation therapy of cancer and can be produced by the bombardment of Cobalt-59 with [Question ID = 692]
1. Alpha particles [Option ID = 2765]
2. Beta particles [Option ID = 2767]
3. Neutrons [Option ID = 2766]
4. Gamma rays [Option ID = 2768]
Correct Answer :-
Neutrons [Option ID = 2766]
The standard emf of galvanic cell involving 3 moles of electrons in its redox reaction is 0.59 V. The equilibrium constant for the
reaction of the cell is- [Question ID = 763]
1. [Option ID = 3051]
2. [Option ID = 3052]
3. [Option ID = 3049]
4. [Option ID = 3050]
Correct Answer :-
[Option ID = 3052]
A characteristic common to polymers that can be made to conduct electricity such as polyacetylene, polypyrrole is: [Question ID =
685]
1. Conjugation throughout the polymeric chain. [Option ID = 2740]
2. A high degree of cross linking [Option ID = 2738]
3. A very low glass transition temperature [Option ID = 2737]
4. Presence of stereogenic centers of the same configuration [Option ID = 2739]
Correct Answer :-
Conjugation throughout the polymeric chain. [Option ID = 2740]
Impact factor is [Question ID = 768]
1. Ratio between citations and recent citable items publish [Option ID = 3071]
2. All of these [Option ID = 3072]
3. Addition of citations and recent citable items publish [Option ID = 3069]
4. Ratio between recent citable items publish and citations [Option ID = 3070]
Correct Answer :-
Ratio between recent citable items publish and citations [Option ID = 3070]
On the basis of oxidation-reduction potential, which of the following is most likely to occur? [Question ID = 693]
13)
14)
15)
16)
17)
1. [Option ID = 2770]
2. [Option ID = 2772]
3. [Option ID = 2771]
4. [Option ID = 2769]
Correct Answer :-
[Option ID = 2769]
How many diastereoisomers are possible for the compound 2, 4 ?diphenylcyclobutane-1, 3 di carboxylic acids. [Question ID = 725]
1. 6 [Option ID = 2899]
2. 5 [Option ID = 2898]
3. 8 [Option ID = 2900]
4. 4 [Option ID = 2897]
Correct Answer :-
5 [Option ID = 2898]
An increase in equivalent conductance of a strong electrolyte with dilution is mainly due to- [Question ID = 764]
1. increase in ionic mobility of ions [Option ID = 3055]
2. increase in number of ions [Option ID = 3054]
3. 100% ionization of electrolyte at normal dilution [Option ID = 3056]
4. increase in both i.e. number of ions and ionic mobility of ions. [Option ID = 3053]
Correct Answer :-
increase in ionic mobility of ions [Option ID = 3055]
The solid state structures of the principal allotropes of elemental boron are made up of which of the following structural units
[Question ID = 699]
1. [Option ID = 2796]
2. [Option ID = 2795]
3. [Option ID = 2794]
4. [Option ID = 2793]
Correct Answer :-
[Option ID = 2793]
The molecular geometry of thionyl chloride is best described as [Question ID = 688]
1. T-shaped [Option ID = 2752]
2. Tetrahedral [Option ID = 2751]
3. Trigonal pyramidal [Option ID = 2749]
4. Trigonal planar [Option ID = 2750]
Correct Answer :-
Trigonal pyramidal [Option ID = 2749]
In a face-center cubic (FCC) type of crystal lattice, the number of atoms belonging exclusively to each unit cell within the lattice
is/are: [Question ID = 754]
1. 2 [Option ID = 3014]
2. 1 [Option ID = 3013]
3. 3 [Option ID = 3015]
4. 4 [Option ID = 3016]
Correct Answer :-
4 [Option ID = 3016]
18)
19)
20)
21)
22)
23)
24)
Among the following, the weakest oxidizing agent is [Question ID = 675]
1. Mg (s) [Option ID = 2698]
2. [Option ID = 2699]
3. [Option ID = 2700]
4. [Option ID = 2697]
Correct Answer :-
Mg (s) [Option ID = 2698]
For a polymer, which of the following statement/s is/are true? [Question ID = 759]
1. Weight average molecular weight is almost always higher than the number average molecular weight [Option ID = 3035]
2. Formation of a polypeptide from its monomers (amino acids) is an example of addition polymerization [Option ID = 3034]
3. All of these [Option ID = 3036]
4. Vinyl polymerization is an example of condensation polymerization. [Option ID = 3033]
Correct Answer :-
Weight average molecular weight is almost always higher than the number average molecular weight [Option ID = 3035]
Quantum dots are [Question ID = 762]
1. Three dimensional [Option ID = 3048]
2. One dimensional [Option ID = 3046]
3. Two dimensional [Option ID = 3047]
4. Zero dimensional [Option ID = 3045]
Correct Answer :-
Zero dimensional [Option ID = 3045]
The unit of rate constant for a third order reaction is: [Question ID = 749]
1. [Option ID = 2993]
2. [Option ID = 2995]
3. [Option ID = 2996]
4. [Option ID = 2994]
Correct Answer :-
[Option ID = 2996]
All the following elements have at least one isotope that is not radioactive EXCEPT [Question ID = 673]
1. Pb [Option ID = 2690]
2. O [Option ID = 2689]
3. Sn [Option ID = 2691]
4. No [Option ID = 2692]
Correct Answer :-
No [Option ID = 2692]
The conditions for a species to follow Bose-Einstein statistics are; [Question ID = 736]
1. Particles are indistinguishable, with no restriction on filling up of energy levels [Option ID = 2944]
2. Particles are indistinguishable, with a restriction on filling up of energy levels [Option ID = 2943]
3. Particles are distinguishable, with a restriction on filling up of energy levels [Option ID = 2941]
4. Particles are distinguishable, with no restriction on filling up of energy levels [Option ID = 2942]
Correct Answer :-
Particles are indistinguishable, with no restriction on filling up of energy levels [Option ID = 2944]
In the kinetic theory of collisions, the SI unit of collision number, in terms of m (meter) and s (second), is:
[Question ID = 761]
25)
26)
27)
28)
29)
1. m-
2
s
-1
[Option ID = 3042]
2. m
4
s
-1
[Option ID = 3041]
3. m
2
s
-1
[Option ID = 3043]
4. None of these [Option ID = 3044]
Correct Answer :-
None of these [Option ID = 3044]
Correct characteristics of the functional groups of adenine in DNA base pair are [Question ID = 706]
1. [Option ID = 2824]
2. [Option ID = 2821]
3. [Option ID = 2823]
4. [Option ID = 2822]
Correct Answer :-
[Option ID = 2822]
The carbon monoxide molecule has an internuclear distance of 1.13 Angstroms. What is the moment of Inertia of this molecule?
[Question ID = 740]
1. [Option ID = 2960]
2. [Option ID = 2957]
3. [Option ID = 2958]
4. [Option ID = 2959]
Correct Answer :-
[Option ID = 2957]
Which of the following represent/s non-linear optical technique? [Question ID = 744]
1. Second Harmonic generation [Option ID = 2974]
2. Two-photon photoluminescence [Option ID = 2975]
3. Four-wave mixing [Option ID = 2973]
4. All of these [Option ID = 2976]
Correct Answer :-
All of these [Option ID = 2976]
. Which of the following does not affect the intensity of spectral lines of a sample? [Question ID = 743]
1. Path length of a sample [Option ID = 2972]
2. Population of energy states [Option ID = 2970]
3. Heisenberg?s Uncertainty principle [Option ID = 2971]
4. Concentration of a sample [Option ID = 2969]
Correct Answer :-
Heisenberg?s Uncertainty principle [Option ID = 2971]
[Question ID = 716]
30)
31)
1. [Option ID = 2862]
2. [Option ID = 2863]
3. [Option ID = 2861]
4. [Option ID = 2864]
Correct Answer :-
[Option ID = 2864]
[Question ID = 712]
1. [Option ID = 2845]
2. [Option ID = 2846]
3. [Option ID = 2848]
4. [Option ID = 2847]
Correct Answer :-
[Option ID = 2847]
[Question ID = 686]
1. [Option ID = 2743]
2. [Option ID = 2742]
3. [Option ID = 2741]
32)
33)
4. [Option ID = 2744]
Correct Answer :-
[Option ID = 2743]
[Question ID = 5633]
1. [Option ID = 22523]
2. [Option ID = 22524]
3. [Option ID = 22526]
4. [Option ID = 22525]
Correct Answer :-
[Option ID = 22523]
[Question ID = 730]
1. [Option ID = 2918]
2. [Option ID = 2920]
3. [Option ID = 2917]
4. [Option ID = 2919]
Correct Answer :-
34)
35)
36)
[Option ID = 2919]
[Question ID = 714]
1. [Option ID = 2856]
2. [Option ID = 2853]
3. [Option ID = 2855]
4. [Option ID = 2854]
Correct Answer :-
[Option ID = 2854]
[Question ID = 669]
1. 2 [Option ID = 2673]
2. 18 [Option ID = 2676]
3. 5 [Option ID = 2674]
4. 10 [Option ID = 2675]
Correct Answer :-
5 [Option ID = 2674]
[Question ID = 689]
1. 2 [Option ID = 2754]
2. 0 [Option ID = 2756]
3. 1 [Option ID = 2755]
37)
38)
4. 3 [Option ID = 2753]
Correct Answer :-
1 [Option ID = 2755]
[Question ID = 718]
1. C) [Option ID = 2871]
2. B) [Option ID = 2870]
3. A) [Option ID = 2869]
4. D) both (B) and (C) [Option ID = 2872]
Correct Answer :-
A) [Option ID = 2869]
[Question ID = 709]
1. [Option ID = 2836]
2. [Option ID = 2833]
3. [Option ID = 2835]
4. [Option ID = 2834]
39)
40)
41)
42)
Correct Answer :-
[Option ID = 2836]
[Question ID = 705]
1. (2Z, 4Z)-3-chlorohexa-2, 4-diene-1,6-diol. [Option ID = 2819]
2. (2E, 4E)-3-chlorohexa-2, 4-diene-1,6-diol. [Option ID = 2817]
3. (2Z, 4E)-3-chlorohexa-2, 4-diene-1,6-diol. [Option ID = 2818]
4. (2Z, 4E)-3-chlorohexa-2, 4-diene-1,6-diol. [Option ID = 2820]
Correct Answer :-
(2Z, 4E)-3-chlorohexa-2, 4-diene-1,6-diol. [Option ID = 2820]
[Question ID = 680]
1. The presence of other complexing ligands in solution affects the equilibrium concentration of metal-EDTA complexes [Option ID = 2719]
2. Metal-EDTA complexes have an equilibrium concentration independent of pH [Option ID = 2718]
3. Metal-EDTA complexes are often 2:1 in stoichiometry [Option ID = 2717]
4. Metal-EDTA complexes are less stable than the corresponding metal-ammine complexes [Option ID = 2720]
Correct Answer :-
The presence of other complexing ligands in solution affects the equilibrium concentration of metal-EDTA complexes [Option ID = 2719]
[Question ID = 671]
1. 150 g [Option ID = 2682]
2. 300 g [Option ID = 2684]
3. 120 g [Option ID = 2681]
4. 180 g [Option ID = 2683]
Correct Answer :-
300 g [Option ID = 2684]
[Question ID = 756]
1. [Option ID = 3021]
2. [Option ID = 3022]
3. [Option ID = 3024]
4. [Option ID = 3023]
Correct Answer :-
43)
44)
45)
[Option ID = 3024]
[Question ID = 750]
1. [Option ID = 2999]
2. [Option ID = 3000]
3. [Option ID = 2997]
4. [Option ID = 2998]
Correct Answer :-
[Option ID = 2997]
[Question ID = 715]
1. [Option ID = 2857]
2. [Option ID = 2860]
3. [Option ID = 2858]
4. [Option ID = 2859]
Correct Answer :-
[Option ID = 2860]
46)
47)
[Question ID = 719]
1. [Option ID = 2873]
2. [Option ID = 2876]
3. [Option ID = 2874]
4. [Option ID = 2875]
Correct Answer :-
[Question ID = 674]
1. Evaporating some water to decrease the volume of the solution. [Option ID = 2696]
2. [Option ID = 2695]
3. [Option ID = 2694]
4. [Option ID = 2693]
Correct Answer :-
[Option ID = 2693]
[Question ID = 728]
1. [Option ID = 2910]
2. [Option ID = 2911]
3. [Option ID = 2909]
4. [Option ID = 2912]
48)
49)
50)
51)
Correct Answer :-
[Option ID = 2912]
[Question ID = 697]
1. III only [Option ID = 2788]
2. II only [Option ID = 2787]
3. I only [Option ID = 2785]
4. I, II and III [Option ID = 2786]
Correct Answer :-
I, II and III [Option ID = 2786]
[Question ID = 700]
1. [Option ID = 2798]
2. [Option ID = 2797]
3. [Option ID = 2800]
4. [Option ID = 2799]
Correct Answer :-
[Option ID = 2799]
[Question ID = 678]
1. Cr
2+
has a low cationic charge [Option ID = 2711]
2. F has -1 anionic charge and highly electronegative [Option ID = 2709]
3. Spin-orbit coupling in Cr
2+
[Option ID = 2712]
4. The Jahn-Teller effect [Option ID = 2710]
Correct Answer :-
The Jahn-Teller effect [Option ID = 2710]
[Question ID = 723]
1. [Option ID = 2889]
FirstRanker.com - FirstRanker's Choice
1)
2)
3)
4)
5)
DU PhD in Chemistry
Topic:- DU_J18_PHD_CHEM
Which of the following statements about sulfur dioxide is true?
[Question ID = 677]
1. It forms a S-S dimer in condensed phase [Option ID = 2707]
2. Its anhydride of sulfuric acid [Option ID = 2706]
3. Its O-S-O angle is 180
0
[Option ID = 2708]
4. It is a product of the combustion of fossil fuels that contain sulfur [Option ID = 2705]
Correct Answer :-
It is a product of the combustion of fossil fuels that contain sulfur [Option ID = 2705]
Which of the following is a strong acid in pure liquid HF
[Question ID = 683]
1. H
2
O [Option ID = 2731]
2. NaF [Option ID = 2729]
3. CH
3
COOH [Option ID = 2730]
4. SbF
5
[Option ID = 2732]
Correct Answer :-
SbF
5
[Option ID = 2732]
Each of the following molecules can act as a chelating agent EXCEPT [Question ID = 679]
1. [Option ID = 2716]
2. [Option ID = 2714]
3. [Option ID = 2713]
4. [Option ID = 2715]
Correct Answer :-
[Option ID = 2714]
What is correct about h-index?
[Question ID = 758]
1. Alternative of impact factor [Option ID = 3031]
2. Based on most quoted papers [Option ID = 3030]
3. Quantify scientific productivity [Option ID = 3029]
4. All of these [Option ID = 3032]
Correct Answer :-
All of these [Option ID = 3032]
The hyperfine electron spin resonance (e.s.r.) spectrum of the benzene radical has how many lines? [Question ID = 748]
1. 12 [Option ID = 2992]
2. 7 [Option ID = 2990]
3. 1 [Option ID = 2991]
4. 6 [Option ID = 2989]
Correct Answer :-
7 [Option ID = 2990]
6)
7)
8)
9)
10)
11)
12)
The energy changes involving the core electrons of an atom or molecule are expressed in which region of the electromagnetic
spectrum? [Question ID = 742]
1. Ultraviolet and Visible region [Option ID = 2967]
2. X-ray region [Option ID = 2968]
3. Radiofrequency region [Option ID = 2966]
4. Infra-red region [Option ID = 2965]
Correct Answer :-
X-ray region [Option ID = 2968]
Find out the expected intensity ratio of M and M+1 signal for the Naphthalene molecular ion [Question ID = 726]
1. 99:1.1 [Option ID = 2903]
2. 1.1:99 [Option ID = 2904]
3. 9:01 [Option ID = 2901]
4. 1:9 [Option ID = 2902]
Correct Answer :-
9:01 [Option ID = 2901]
Cobalt-60 is used in radiation therapy of cancer and can be produced by the bombardment of Cobalt-59 with [Question ID = 692]
1. Alpha particles [Option ID = 2765]
2. Beta particles [Option ID = 2767]
3. Neutrons [Option ID = 2766]
4. Gamma rays [Option ID = 2768]
Correct Answer :-
Neutrons [Option ID = 2766]
The standard emf of galvanic cell involving 3 moles of electrons in its redox reaction is 0.59 V. The equilibrium constant for the
reaction of the cell is- [Question ID = 763]
1. [Option ID = 3051]
2. [Option ID = 3052]
3. [Option ID = 3049]
4. [Option ID = 3050]
Correct Answer :-
[Option ID = 3052]
A characteristic common to polymers that can be made to conduct electricity such as polyacetylene, polypyrrole is: [Question ID =
685]
1. Conjugation throughout the polymeric chain. [Option ID = 2740]
2. A high degree of cross linking [Option ID = 2738]
3. A very low glass transition temperature [Option ID = 2737]
4. Presence of stereogenic centers of the same configuration [Option ID = 2739]
Correct Answer :-
Conjugation throughout the polymeric chain. [Option ID = 2740]
Impact factor is [Question ID = 768]
1. Ratio between citations and recent citable items publish [Option ID = 3071]
2. All of these [Option ID = 3072]
3. Addition of citations and recent citable items publish [Option ID = 3069]
4. Ratio between recent citable items publish and citations [Option ID = 3070]
Correct Answer :-
Ratio between recent citable items publish and citations [Option ID = 3070]
On the basis of oxidation-reduction potential, which of the following is most likely to occur? [Question ID = 693]
13)
14)
15)
16)
17)
1. [Option ID = 2770]
2. [Option ID = 2772]
3. [Option ID = 2771]
4. [Option ID = 2769]
Correct Answer :-
[Option ID = 2769]
How many diastereoisomers are possible for the compound 2, 4 ?diphenylcyclobutane-1, 3 di carboxylic acids. [Question ID = 725]
1. 6 [Option ID = 2899]
2. 5 [Option ID = 2898]
3. 8 [Option ID = 2900]
4. 4 [Option ID = 2897]
Correct Answer :-
5 [Option ID = 2898]
An increase in equivalent conductance of a strong electrolyte with dilution is mainly due to- [Question ID = 764]
1. increase in ionic mobility of ions [Option ID = 3055]
2. increase in number of ions [Option ID = 3054]
3. 100% ionization of electrolyte at normal dilution [Option ID = 3056]
4. increase in both i.e. number of ions and ionic mobility of ions. [Option ID = 3053]
Correct Answer :-
increase in ionic mobility of ions [Option ID = 3055]
The solid state structures of the principal allotropes of elemental boron are made up of which of the following structural units
[Question ID = 699]
1. [Option ID = 2796]
2. [Option ID = 2795]
3. [Option ID = 2794]
4. [Option ID = 2793]
Correct Answer :-
[Option ID = 2793]
The molecular geometry of thionyl chloride is best described as [Question ID = 688]
1. T-shaped [Option ID = 2752]
2. Tetrahedral [Option ID = 2751]
3. Trigonal pyramidal [Option ID = 2749]
4. Trigonal planar [Option ID = 2750]
Correct Answer :-
Trigonal pyramidal [Option ID = 2749]
In a face-center cubic (FCC) type of crystal lattice, the number of atoms belonging exclusively to each unit cell within the lattice
is/are: [Question ID = 754]
1. 2 [Option ID = 3014]
2. 1 [Option ID = 3013]
3. 3 [Option ID = 3015]
4. 4 [Option ID = 3016]
Correct Answer :-
4 [Option ID = 3016]
18)
19)
20)
21)
22)
23)
24)
Among the following, the weakest oxidizing agent is [Question ID = 675]
1. Mg (s) [Option ID = 2698]
2. [Option ID = 2699]
3. [Option ID = 2700]
4. [Option ID = 2697]
Correct Answer :-
Mg (s) [Option ID = 2698]
For a polymer, which of the following statement/s is/are true? [Question ID = 759]
1. Weight average molecular weight is almost always higher than the number average molecular weight [Option ID = 3035]
2. Formation of a polypeptide from its monomers (amino acids) is an example of addition polymerization [Option ID = 3034]
3. All of these [Option ID = 3036]
4. Vinyl polymerization is an example of condensation polymerization. [Option ID = 3033]
Correct Answer :-
Weight average molecular weight is almost always higher than the number average molecular weight [Option ID = 3035]
Quantum dots are [Question ID = 762]
1. Three dimensional [Option ID = 3048]
2. One dimensional [Option ID = 3046]
3. Two dimensional [Option ID = 3047]
4. Zero dimensional [Option ID = 3045]
Correct Answer :-
Zero dimensional [Option ID = 3045]
The unit of rate constant for a third order reaction is: [Question ID = 749]
1. [Option ID = 2993]
2. [Option ID = 2995]
3. [Option ID = 2996]
4. [Option ID = 2994]
Correct Answer :-
[Option ID = 2996]
All the following elements have at least one isotope that is not radioactive EXCEPT [Question ID = 673]
1. Pb [Option ID = 2690]
2. O [Option ID = 2689]
3. Sn [Option ID = 2691]
4. No [Option ID = 2692]
Correct Answer :-
No [Option ID = 2692]
The conditions for a species to follow Bose-Einstein statistics are; [Question ID = 736]
1. Particles are indistinguishable, with no restriction on filling up of energy levels [Option ID = 2944]
2. Particles are indistinguishable, with a restriction on filling up of energy levels [Option ID = 2943]
3. Particles are distinguishable, with a restriction on filling up of energy levels [Option ID = 2941]
4. Particles are distinguishable, with no restriction on filling up of energy levels [Option ID = 2942]
Correct Answer :-
Particles are indistinguishable, with no restriction on filling up of energy levels [Option ID = 2944]
In the kinetic theory of collisions, the SI unit of collision number, in terms of m (meter) and s (second), is:
[Question ID = 761]
25)
26)
27)
28)
29)
1. m-
2
s
-1
[Option ID = 3042]
2. m
4
s
-1
[Option ID = 3041]
3. m
2
s
-1
[Option ID = 3043]
4. None of these [Option ID = 3044]
Correct Answer :-
None of these [Option ID = 3044]
Correct characteristics of the functional groups of adenine in DNA base pair are [Question ID = 706]
1. [Option ID = 2824]
2. [Option ID = 2821]
3. [Option ID = 2823]
4. [Option ID = 2822]
Correct Answer :-
[Option ID = 2822]
The carbon monoxide molecule has an internuclear distance of 1.13 Angstroms. What is the moment of Inertia of this molecule?
[Question ID = 740]
1. [Option ID = 2960]
2. [Option ID = 2957]
3. [Option ID = 2958]
4. [Option ID = 2959]
Correct Answer :-
[Option ID = 2957]
Which of the following represent/s non-linear optical technique? [Question ID = 744]
1. Second Harmonic generation [Option ID = 2974]
2. Two-photon photoluminescence [Option ID = 2975]
3. Four-wave mixing [Option ID = 2973]
4. All of these [Option ID = 2976]
Correct Answer :-
All of these [Option ID = 2976]
. Which of the following does not affect the intensity of spectral lines of a sample? [Question ID = 743]
1. Path length of a sample [Option ID = 2972]
2. Population of energy states [Option ID = 2970]
3. Heisenberg?s Uncertainty principle [Option ID = 2971]
4. Concentration of a sample [Option ID = 2969]
Correct Answer :-
Heisenberg?s Uncertainty principle [Option ID = 2971]
[Question ID = 716]
30)
31)
1. [Option ID = 2862]
2. [Option ID = 2863]
3. [Option ID = 2861]
4. [Option ID = 2864]
Correct Answer :-
[Option ID = 2864]
[Question ID = 712]
1. [Option ID = 2845]
2. [Option ID = 2846]
3. [Option ID = 2848]
4. [Option ID = 2847]
Correct Answer :-
[Option ID = 2847]
[Question ID = 686]
1. [Option ID = 2743]
2. [Option ID = 2742]
3. [Option ID = 2741]
32)
33)
4. [Option ID = 2744]
Correct Answer :-
[Option ID = 2743]
[Question ID = 5633]
1. [Option ID = 22523]
2. [Option ID = 22524]
3. [Option ID = 22526]
4. [Option ID = 22525]
Correct Answer :-
[Option ID = 22523]
[Question ID = 730]
1. [Option ID = 2918]
2. [Option ID = 2920]
3. [Option ID = 2917]
4. [Option ID = 2919]
Correct Answer :-
34)
35)
36)
[Option ID = 2919]
[Question ID = 714]
1. [Option ID = 2856]
2. [Option ID = 2853]
3. [Option ID = 2855]
4. [Option ID = 2854]
Correct Answer :-
[Option ID = 2854]
[Question ID = 669]
1. 2 [Option ID = 2673]
2. 18 [Option ID = 2676]
3. 5 [Option ID = 2674]
4. 10 [Option ID = 2675]
Correct Answer :-
5 [Option ID = 2674]
[Question ID = 689]
1. 2 [Option ID = 2754]
2. 0 [Option ID = 2756]
3. 1 [Option ID = 2755]
37)
38)
4. 3 [Option ID = 2753]
Correct Answer :-
1 [Option ID = 2755]
[Question ID = 718]
1. C) [Option ID = 2871]
2. B) [Option ID = 2870]
3. A) [Option ID = 2869]
4. D) both (B) and (C) [Option ID = 2872]
Correct Answer :-
A) [Option ID = 2869]
[Question ID = 709]
1. [Option ID = 2836]
2. [Option ID = 2833]
3. [Option ID = 2835]
4. [Option ID = 2834]
39)
40)
41)
42)
Correct Answer :-
[Option ID = 2836]
[Question ID = 705]
1. (2Z, 4Z)-3-chlorohexa-2, 4-diene-1,6-diol. [Option ID = 2819]
2. (2E, 4E)-3-chlorohexa-2, 4-diene-1,6-diol. [Option ID = 2817]
3. (2Z, 4E)-3-chlorohexa-2, 4-diene-1,6-diol. [Option ID = 2818]
4. (2Z, 4E)-3-chlorohexa-2, 4-diene-1,6-diol. [Option ID = 2820]
Correct Answer :-
(2Z, 4E)-3-chlorohexa-2, 4-diene-1,6-diol. [Option ID = 2820]
[Question ID = 680]
1. The presence of other complexing ligands in solution affects the equilibrium concentration of metal-EDTA complexes [Option ID = 2719]
2. Metal-EDTA complexes have an equilibrium concentration independent of pH [Option ID = 2718]
3. Metal-EDTA complexes are often 2:1 in stoichiometry [Option ID = 2717]
4. Metal-EDTA complexes are less stable than the corresponding metal-ammine complexes [Option ID = 2720]
Correct Answer :-
The presence of other complexing ligands in solution affects the equilibrium concentration of metal-EDTA complexes [Option ID = 2719]
[Question ID = 671]
1. 150 g [Option ID = 2682]
2. 300 g [Option ID = 2684]
3. 120 g [Option ID = 2681]
4. 180 g [Option ID = 2683]
Correct Answer :-
300 g [Option ID = 2684]
[Question ID = 756]
1. [Option ID = 3021]
2. [Option ID = 3022]
3. [Option ID = 3024]
4. [Option ID = 3023]
Correct Answer :-
43)
44)
45)
[Option ID = 3024]
[Question ID = 750]
1. [Option ID = 2999]
2. [Option ID = 3000]
3. [Option ID = 2997]
4. [Option ID = 2998]
Correct Answer :-
[Option ID = 2997]
[Question ID = 715]
1. [Option ID = 2857]
2. [Option ID = 2860]
3. [Option ID = 2858]
4. [Option ID = 2859]
Correct Answer :-
[Option ID = 2860]
46)
47)
[Question ID = 719]
1. [Option ID = 2873]
2. [Option ID = 2876]
3. [Option ID = 2874]
4. [Option ID = 2875]
Correct Answer :-
[Question ID = 674]
1. Evaporating some water to decrease the volume of the solution. [Option ID = 2696]
2. [Option ID = 2695]
3. [Option ID = 2694]
4. [Option ID = 2693]
Correct Answer :-
[Option ID = 2693]
[Question ID = 728]
1. [Option ID = 2910]
2. [Option ID = 2911]
3. [Option ID = 2909]
4. [Option ID = 2912]
48)
49)
50)
51)
Correct Answer :-
[Option ID = 2912]
[Question ID = 697]
1. III only [Option ID = 2788]
2. II only [Option ID = 2787]
3. I only [Option ID = 2785]
4. I, II and III [Option ID = 2786]
Correct Answer :-
I, II and III [Option ID = 2786]
[Question ID = 700]
1. [Option ID = 2798]
2. [Option ID = 2797]
3. [Option ID = 2800]
4. [Option ID = 2799]
Correct Answer :-
[Option ID = 2799]
[Question ID = 678]
1. Cr
2+
has a low cationic charge [Option ID = 2711]
2. F has -1 anionic charge and highly electronegative [Option ID = 2709]
3. Spin-orbit coupling in Cr
2+
[Option ID = 2712]
4. The Jahn-Teller effect [Option ID = 2710]
Correct Answer :-
The Jahn-Teller effect [Option ID = 2710]
[Question ID = 723]
1. [Option ID = 2889]
52)
53)
2. [Option ID = 2890]
3. [Option ID = 2891]
4. [Option ID = 2892]
Correct Answer :-
[Option ID = 2889]
[Question ID = 727]
1. [Option ID = 2907]
2. [Option ID = 2906]
3. [Option ID = 2905]
4. [Option ID = 2908]
Correct Answer :-
[Option ID = 2905]
[Question ID = 707]
FirstRanker.com - FirstRanker's Choice
1)
2)
3)
4)
5)
DU PhD in Chemistry
Topic:- DU_J18_PHD_CHEM
Which of the following statements about sulfur dioxide is true?
[Question ID = 677]
1. It forms a S-S dimer in condensed phase [Option ID = 2707]
2. Its anhydride of sulfuric acid [Option ID = 2706]
3. Its O-S-O angle is 180
0
[Option ID = 2708]
4. It is a product of the combustion of fossil fuels that contain sulfur [Option ID = 2705]
Correct Answer :-
It is a product of the combustion of fossil fuels that contain sulfur [Option ID = 2705]
Which of the following is a strong acid in pure liquid HF
[Question ID = 683]
1. H
2
O [Option ID = 2731]
2. NaF [Option ID = 2729]
3. CH
3
COOH [Option ID = 2730]
4. SbF
5
[Option ID = 2732]
Correct Answer :-
SbF
5
[Option ID = 2732]
Each of the following molecules can act as a chelating agent EXCEPT [Question ID = 679]
1. [Option ID = 2716]
2. [Option ID = 2714]
3. [Option ID = 2713]
4. [Option ID = 2715]
Correct Answer :-
[Option ID = 2714]
What is correct about h-index?
[Question ID = 758]
1. Alternative of impact factor [Option ID = 3031]
2. Based on most quoted papers [Option ID = 3030]
3. Quantify scientific productivity [Option ID = 3029]
4. All of these [Option ID = 3032]
Correct Answer :-
All of these [Option ID = 3032]
The hyperfine electron spin resonance (e.s.r.) spectrum of the benzene radical has how many lines? [Question ID = 748]
1. 12 [Option ID = 2992]
2. 7 [Option ID = 2990]
3. 1 [Option ID = 2991]
4. 6 [Option ID = 2989]
Correct Answer :-
7 [Option ID = 2990]
6)
7)
8)
9)
10)
11)
12)
The energy changes involving the core electrons of an atom or molecule are expressed in which region of the electromagnetic
spectrum? [Question ID = 742]
1. Ultraviolet and Visible region [Option ID = 2967]
2. X-ray region [Option ID = 2968]
3. Radiofrequency region [Option ID = 2966]
4. Infra-red region [Option ID = 2965]
Correct Answer :-
X-ray region [Option ID = 2968]
Find out the expected intensity ratio of M and M+1 signal for the Naphthalene molecular ion [Question ID = 726]
1. 99:1.1 [Option ID = 2903]
2. 1.1:99 [Option ID = 2904]
3. 9:01 [Option ID = 2901]
4. 1:9 [Option ID = 2902]
Correct Answer :-
9:01 [Option ID = 2901]
Cobalt-60 is used in radiation therapy of cancer and can be produced by the bombardment of Cobalt-59 with [Question ID = 692]
1. Alpha particles [Option ID = 2765]
2. Beta particles [Option ID = 2767]
3. Neutrons [Option ID = 2766]
4. Gamma rays [Option ID = 2768]
Correct Answer :-
Neutrons [Option ID = 2766]
The standard emf of galvanic cell involving 3 moles of electrons in its redox reaction is 0.59 V. The equilibrium constant for the
reaction of the cell is- [Question ID = 763]
1. [Option ID = 3051]
2. [Option ID = 3052]
3. [Option ID = 3049]
4. [Option ID = 3050]
Correct Answer :-
[Option ID = 3052]
A characteristic common to polymers that can be made to conduct electricity such as polyacetylene, polypyrrole is: [Question ID =
685]
1. Conjugation throughout the polymeric chain. [Option ID = 2740]
2. A high degree of cross linking [Option ID = 2738]
3. A very low glass transition temperature [Option ID = 2737]
4. Presence of stereogenic centers of the same configuration [Option ID = 2739]
Correct Answer :-
Conjugation throughout the polymeric chain. [Option ID = 2740]
Impact factor is [Question ID = 768]
1. Ratio between citations and recent citable items publish [Option ID = 3071]
2. All of these [Option ID = 3072]
3. Addition of citations and recent citable items publish [Option ID = 3069]
4. Ratio between recent citable items publish and citations [Option ID = 3070]
Correct Answer :-
Ratio between recent citable items publish and citations [Option ID = 3070]
On the basis of oxidation-reduction potential, which of the following is most likely to occur? [Question ID = 693]
13)
14)
15)
16)
17)
1. [Option ID = 2770]
2. [Option ID = 2772]
3. [Option ID = 2771]
4. [Option ID = 2769]
Correct Answer :-
[Option ID = 2769]
How many diastereoisomers are possible for the compound 2, 4 ?diphenylcyclobutane-1, 3 di carboxylic acids. [Question ID = 725]
1. 6 [Option ID = 2899]
2. 5 [Option ID = 2898]
3. 8 [Option ID = 2900]
4. 4 [Option ID = 2897]
Correct Answer :-
5 [Option ID = 2898]
An increase in equivalent conductance of a strong electrolyte with dilution is mainly due to- [Question ID = 764]
1. increase in ionic mobility of ions [Option ID = 3055]
2. increase in number of ions [Option ID = 3054]
3. 100% ionization of electrolyte at normal dilution [Option ID = 3056]
4. increase in both i.e. number of ions and ionic mobility of ions. [Option ID = 3053]
Correct Answer :-
increase in ionic mobility of ions [Option ID = 3055]
The solid state structures of the principal allotropes of elemental boron are made up of which of the following structural units
[Question ID = 699]
1. [Option ID = 2796]
2. [Option ID = 2795]
3. [Option ID = 2794]
4. [Option ID = 2793]
Correct Answer :-
[Option ID = 2793]
The molecular geometry of thionyl chloride is best described as [Question ID = 688]
1. T-shaped [Option ID = 2752]
2. Tetrahedral [Option ID = 2751]
3. Trigonal pyramidal [Option ID = 2749]
4. Trigonal planar [Option ID = 2750]
Correct Answer :-
Trigonal pyramidal [Option ID = 2749]
In a face-center cubic (FCC) type of crystal lattice, the number of atoms belonging exclusively to each unit cell within the lattice
is/are: [Question ID = 754]
1. 2 [Option ID = 3014]
2. 1 [Option ID = 3013]
3. 3 [Option ID = 3015]
4. 4 [Option ID = 3016]
Correct Answer :-
4 [Option ID = 3016]
18)
19)
20)
21)
22)
23)
24)
Among the following, the weakest oxidizing agent is [Question ID = 675]
1. Mg (s) [Option ID = 2698]
2. [Option ID = 2699]
3. [Option ID = 2700]
4. [Option ID = 2697]
Correct Answer :-
Mg (s) [Option ID = 2698]
For a polymer, which of the following statement/s is/are true? [Question ID = 759]
1. Weight average molecular weight is almost always higher than the number average molecular weight [Option ID = 3035]
2. Formation of a polypeptide from its monomers (amino acids) is an example of addition polymerization [Option ID = 3034]
3. All of these [Option ID = 3036]
4. Vinyl polymerization is an example of condensation polymerization. [Option ID = 3033]
Correct Answer :-
Weight average molecular weight is almost always higher than the number average molecular weight [Option ID = 3035]
Quantum dots are [Question ID = 762]
1. Three dimensional [Option ID = 3048]
2. One dimensional [Option ID = 3046]
3. Two dimensional [Option ID = 3047]
4. Zero dimensional [Option ID = 3045]
Correct Answer :-
Zero dimensional [Option ID = 3045]
The unit of rate constant for a third order reaction is: [Question ID = 749]
1. [Option ID = 2993]
2. [Option ID = 2995]
3. [Option ID = 2996]
4. [Option ID = 2994]
Correct Answer :-
[Option ID = 2996]
All the following elements have at least one isotope that is not radioactive EXCEPT [Question ID = 673]
1. Pb [Option ID = 2690]
2. O [Option ID = 2689]
3. Sn [Option ID = 2691]
4. No [Option ID = 2692]
Correct Answer :-
No [Option ID = 2692]
The conditions for a species to follow Bose-Einstein statistics are; [Question ID = 736]
1. Particles are indistinguishable, with no restriction on filling up of energy levels [Option ID = 2944]
2. Particles are indistinguishable, with a restriction on filling up of energy levels [Option ID = 2943]
3. Particles are distinguishable, with a restriction on filling up of energy levels [Option ID = 2941]
4. Particles are distinguishable, with no restriction on filling up of energy levels [Option ID = 2942]
Correct Answer :-
Particles are indistinguishable, with no restriction on filling up of energy levels [Option ID = 2944]
In the kinetic theory of collisions, the SI unit of collision number, in terms of m (meter) and s (second), is:
[Question ID = 761]
25)
26)
27)
28)
29)
1. m-
2
s
-1
[Option ID = 3042]
2. m
4
s
-1
[Option ID = 3041]
3. m
2
s
-1
[Option ID = 3043]
4. None of these [Option ID = 3044]
Correct Answer :-
None of these [Option ID = 3044]
Correct characteristics of the functional groups of adenine in DNA base pair are [Question ID = 706]
1. [Option ID = 2824]
2. [Option ID = 2821]
3. [Option ID = 2823]
4. [Option ID = 2822]
Correct Answer :-
[Option ID = 2822]
The carbon monoxide molecule has an internuclear distance of 1.13 Angstroms. What is the moment of Inertia of this molecule?
[Question ID = 740]
1. [Option ID = 2960]
2. [Option ID = 2957]
3. [Option ID = 2958]
4. [Option ID = 2959]
Correct Answer :-
[Option ID = 2957]
Which of the following represent/s non-linear optical technique? [Question ID = 744]
1. Second Harmonic generation [Option ID = 2974]
2. Two-photon photoluminescence [Option ID = 2975]
3. Four-wave mixing [Option ID = 2973]
4. All of these [Option ID = 2976]
Correct Answer :-
All of these [Option ID = 2976]
. Which of the following does not affect the intensity of spectral lines of a sample? [Question ID = 743]
1. Path length of a sample [Option ID = 2972]
2. Population of energy states [Option ID = 2970]
3. Heisenberg?s Uncertainty principle [Option ID = 2971]
4. Concentration of a sample [Option ID = 2969]
Correct Answer :-
Heisenberg?s Uncertainty principle [Option ID = 2971]
[Question ID = 716]
30)
31)
1. [Option ID = 2862]
2. [Option ID = 2863]
3. [Option ID = 2861]
4. [Option ID = 2864]
Correct Answer :-
[Option ID = 2864]
[Question ID = 712]
1. [Option ID = 2845]
2. [Option ID = 2846]
3. [Option ID = 2848]
4. [Option ID = 2847]
Correct Answer :-
[Option ID = 2847]
[Question ID = 686]
1. [Option ID = 2743]
2. [Option ID = 2742]
3. [Option ID = 2741]
32)
33)
4. [Option ID = 2744]
Correct Answer :-
[Option ID = 2743]
[Question ID = 5633]
1. [Option ID = 22523]
2. [Option ID = 22524]
3. [Option ID = 22526]
4. [Option ID = 22525]
Correct Answer :-
[Option ID = 22523]
[Question ID = 730]
1. [Option ID = 2918]
2. [Option ID = 2920]
3. [Option ID = 2917]
4. [Option ID = 2919]
Correct Answer :-
34)
35)
36)
[Option ID = 2919]
[Question ID = 714]
1. [Option ID = 2856]
2. [Option ID = 2853]
3. [Option ID = 2855]
4. [Option ID = 2854]
Correct Answer :-
[Option ID = 2854]
[Question ID = 669]
1. 2 [Option ID = 2673]
2. 18 [Option ID = 2676]
3. 5 [Option ID = 2674]
4. 10 [Option ID = 2675]
Correct Answer :-
5 [Option ID = 2674]
[Question ID = 689]
1. 2 [Option ID = 2754]
2. 0 [Option ID = 2756]
3. 1 [Option ID = 2755]
37)
38)
4. 3 [Option ID = 2753]
Correct Answer :-
1 [Option ID = 2755]
[Question ID = 718]
1. C) [Option ID = 2871]
2. B) [Option ID = 2870]
3. A) [Option ID = 2869]
4. D) both (B) and (C) [Option ID = 2872]
Correct Answer :-
A) [Option ID = 2869]
[Question ID = 709]
1. [Option ID = 2836]
2. [Option ID = 2833]
3. [Option ID = 2835]
4. [Option ID = 2834]
39)
40)
41)
42)
Correct Answer :-
[Option ID = 2836]
[Question ID = 705]
1. (2Z, 4Z)-3-chlorohexa-2, 4-diene-1,6-diol. [Option ID = 2819]
2. (2E, 4E)-3-chlorohexa-2, 4-diene-1,6-diol. [Option ID = 2817]
3. (2Z, 4E)-3-chlorohexa-2, 4-diene-1,6-diol. [Option ID = 2818]
4. (2Z, 4E)-3-chlorohexa-2, 4-diene-1,6-diol. [Option ID = 2820]
Correct Answer :-
(2Z, 4E)-3-chlorohexa-2, 4-diene-1,6-diol. [Option ID = 2820]
[Question ID = 680]
1. The presence of other complexing ligands in solution affects the equilibrium concentration of metal-EDTA complexes [Option ID = 2719]
2. Metal-EDTA complexes have an equilibrium concentration independent of pH [Option ID = 2718]
3. Metal-EDTA complexes are often 2:1 in stoichiometry [Option ID = 2717]
4. Metal-EDTA complexes are less stable than the corresponding metal-ammine complexes [Option ID = 2720]
Correct Answer :-
The presence of other complexing ligands in solution affects the equilibrium concentration of metal-EDTA complexes [Option ID = 2719]
[Question ID = 671]
1. 150 g [Option ID = 2682]
2. 300 g [Option ID = 2684]
3. 120 g [Option ID = 2681]
4. 180 g [Option ID = 2683]
Correct Answer :-
300 g [Option ID = 2684]
[Question ID = 756]
1. [Option ID = 3021]
2. [Option ID = 3022]
3. [Option ID = 3024]
4. [Option ID = 3023]
Correct Answer :-
43)
44)
45)
[Option ID = 3024]
[Question ID = 750]
1. [Option ID = 2999]
2. [Option ID = 3000]
3. [Option ID = 2997]
4. [Option ID = 2998]
Correct Answer :-
[Option ID = 2997]
[Question ID = 715]
1. [Option ID = 2857]
2. [Option ID = 2860]
3. [Option ID = 2858]
4. [Option ID = 2859]
Correct Answer :-
[Option ID = 2860]
46)
47)
[Question ID = 719]
1. [Option ID = 2873]
2. [Option ID = 2876]
3. [Option ID = 2874]
4. [Option ID = 2875]
Correct Answer :-
[Question ID = 674]
1. Evaporating some water to decrease the volume of the solution. [Option ID = 2696]
2. [Option ID = 2695]
3. [Option ID = 2694]
4. [Option ID = 2693]
Correct Answer :-
[Option ID = 2693]
[Question ID = 728]
1. [Option ID = 2910]
2. [Option ID = 2911]
3. [Option ID = 2909]
4. [Option ID = 2912]
48)
49)
50)
51)
Correct Answer :-
[Option ID = 2912]
[Question ID = 697]
1. III only [Option ID = 2788]
2. II only [Option ID = 2787]
3. I only [Option ID = 2785]
4. I, II and III [Option ID = 2786]
Correct Answer :-
I, II and III [Option ID = 2786]
[Question ID = 700]
1. [Option ID = 2798]
2. [Option ID = 2797]
3. [Option ID = 2800]
4. [Option ID = 2799]
Correct Answer :-
[Option ID = 2799]
[Question ID = 678]
1. Cr
2+
has a low cationic charge [Option ID = 2711]
2. F has -1 anionic charge and highly electronegative [Option ID = 2709]
3. Spin-orbit coupling in Cr
2+
[Option ID = 2712]
4. The Jahn-Teller effect [Option ID = 2710]
Correct Answer :-
The Jahn-Teller effect [Option ID = 2710]
[Question ID = 723]
1. [Option ID = 2889]
52)
53)
2. [Option ID = 2890]
3. [Option ID = 2891]
4. [Option ID = 2892]
Correct Answer :-
[Option ID = 2889]
[Question ID = 727]
1. [Option ID = 2907]
2. [Option ID = 2906]
3. [Option ID = 2905]
4. [Option ID = 2908]
Correct Answer :-
[Option ID = 2905]
[Question ID = 707]
54)
55)
56)
1. N9 [Option ID = 2828]
2. N7 [Option ID = 2827]
3. N1 [Option ID = 2825]
4. N3 [Option ID = 2826]
Correct Answer :-
N7 [Option ID = 2827]
[Question ID = 672]
1. Bicapped prism [Option ID = 2688]
2. Square pyramidal [Option ID = 2686]
3. Trigonal planar [Option ID = 2685]
4. Bent [Option ID = 2687]
Correct Answer :-
Square pyramidal [Option ID = 2686]
[Question ID = 704]
1. [Option ID = 2816]
2. [Option ID = 2814]
3. [Option ID = 2813]
4. [Option ID = 2815]
Correct Answer :-
[Option ID = 2813]
FirstRanker.com - FirstRanker's Choice
1)
2)
3)
4)
5)
DU PhD in Chemistry
Topic:- DU_J18_PHD_CHEM
Which of the following statements about sulfur dioxide is true?
[Question ID = 677]
1. It forms a S-S dimer in condensed phase [Option ID = 2707]
2. Its anhydride of sulfuric acid [Option ID = 2706]
3. Its O-S-O angle is 180
0
[Option ID = 2708]
4. It is a product of the combustion of fossil fuels that contain sulfur [Option ID = 2705]
Correct Answer :-
It is a product of the combustion of fossil fuels that contain sulfur [Option ID = 2705]
Which of the following is a strong acid in pure liquid HF
[Question ID = 683]
1. H
2
O [Option ID = 2731]
2. NaF [Option ID = 2729]
3. CH
3
COOH [Option ID = 2730]
4. SbF
5
[Option ID = 2732]
Correct Answer :-
SbF
5
[Option ID = 2732]
Each of the following molecules can act as a chelating agent EXCEPT [Question ID = 679]
1. [Option ID = 2716]
2. [Option ID = 2714]
3. [Option ID = 2713]
4. [Option ID = 2715]
Correct Answer :-
[Option ID = 2714]
What is correct about h-index?
[Question ID = 758]
1. Alternative of impact factor [Option ID = 3031]
2. Based on most quoted papers [Option ID = 3030]
3. Quantify scientific productivity [Option ID = 3029]
4. All of these [Option ID = 3032]
Correct Answer :-
All of these [Option ID = 3032]
The hyperfine electron spin resonance (e.s.r.) spectrum of the benzene radical has how many lines? [Question ID = 748]
1. 12 [Option ID = 2992]
2. 7 [Option ID = 2990]
3. 1 [Option ID = 2991]
4. 6 [Option ID = 2989]
Correct Answer :-
7 [Option ID = 2990]
6)
7)
8)
9)
10)
11)
12)
The energy changes involving the core electrons of an atom or molecule are expressed in which region of the electromagnetic
spectrum? [Question ID = 742]
1. Ultraviolet and Visible region [Option ID = 2967]
2. X-ray region [Option ID = 2968]
3. Radiofrequency region [Option ID = 2966]
4. Infra-red region [Option ID = 2965]
Correct Answer :-
X-ray region [Option ID = 2968]
Find out the expected intensity ratio of M and M+1 signal for the Naphthalene molecular ion [Question ID = 726]
1. 99:1.1 [Option ID = 2903]
2. 1.1:99 [Option ID = 2904]
3. 9:01 [Option ID = 2901]
4. 1:9 [Option ID = 2902]
Correct Answer :-
9:01 [Option ID = 2901]
Cobalt-60 is used in radiation therapy of cancer and can be produced by the bombardment of Cobalt-59 with [Question ID = 692]
1. Alpha particles [Option ID = 2765]
2. Beta particles [Option ID = 2767]
3. Neutrons [Option ID = 2766]
4. Gamma rays [Option ID = 2768]
Correct Answer :-
Neutrons [Option ID = 2766]
The standard emf of galvanic cell involving 3 moles of electrons in its redox reaction is 0.59 V. The equilibrium constant for the
reaction of the cell is- [Question ID = 763]
1. [Option ID = 3051]
2. [Option ID = 3052]
3. [Option ID = 3049]
4. [Option ID = 3050]
Correct Answer :-
[Option ID = 3052]
A characteristic common to polymers that can be made to conduct electricity such as polyacetylene, polypyrrole is: [Question ID =
685]
1. Conjugation throughout the polymeric chain. [Option ID = 2740]
2. A high degree of cross linking [Option ID = 2738]
3. A very low glass transition temperature [Option ID = 2737]
4. Presence of stereogenic centers of the same configuration [Option ID = 2739]
Correct Answer :-
Conjugation throughout the polymeric chain. [Option ID = 2740]
Impact factor is [Question ID = 768]
1. Ratio between citations and recent citable items publish [Option ID = 3071]
2. All of these [Option ID = 3072]
3. Addition of citations and recent citable items publish [Option ID = 3069]
4. Ratio between recent citable items publish and citations [Option ID = 3070]
Correct Answer :-
Ratio between recent citable items publish and citations [Option ID = 3070]
On the basis of oxidation-reduction potential, which of the following is most likely to occur? [Question ID = 693]
13)
14)
15)
16)
17)
1. [Option ID = 2770]
2. [Option ID = 2772]
3. [Option ID = 2771]
4. [Option ID = 2769]
Correct Answer :-
[Option ID = 2769]
How many diastereoisomers are possible for the compound 2, 4 ?diphenylcyclobutane-1, 3 di carboxylic acids. [Question ID = 725]
1. 6 [Option ID = 2899]
2. 5 [Option ID = 2898]
3. 8 [Option ID = 2900]
4. 4 [Option ID = 2897]
Correct Answer :-
5 [Option ID = 2898]
An increase in equivalent conductance of a strong electrolyte with dilution is mainly due to- [Question ID = 764]
1. increase in ionic mobility of ions [Option ID = 3055]
2. increase in number of ions [Option ID = 3054]
3. 100% ionization of electrolyte at normal dilution [Option ID = 3056]
4. increase in both i.e. number of ions and ionic mobility of ions. [Option ID = 3053]
Correct Answer :-
increase in ionic mobility of ions [Option ID = 3055]
The solid state structures of the principal allotropes of elemental boron are made up of which of the following structural units
[Question ID = 699]
1. [Option ID = 2796]
2. [Option ID = 2795]
3. [Option ID = 2794]
4. [Option ID = 2793]
Correct Answer :-
[Option ID = 2793]
The molecular geometry of thionyl chloride is best described as [Question ID = 688]
1. T-shaped [Option ID = 2752]
2. Tetrahedral [Option ID = 2751]
3. Trigonal pyramidal [Option ID = 2749]
4. Trigonal planar [Option ID = 2750]
Correct Answer :-
Trigonal pyramidal [Option ID = 2749]
In a face-center cubic (FCC) type of crystal lattice, the number of atoms belonging exclusively to each unit cell within the lattice
is/are: [Question ID = 754]
1. 2 [Option ID = 3014]
2. 1 [Option ID = 3013]
3. 3 [Option ID = 3015]
4. 4 [Option ID = 3016]
Correct Answer :-
4 [Option ID = 3016]
18)
19)
20)
21)
22)
23)
24)
Among the following, the weakest oxidizing agent is [Question ID = 675]
1. Mg (s) [Option ID = 2698]
2. [Option ID = 2699]
3. [Option ID = 2700]
4. [Option ID = 2697]
Correct Answer :-
Mg (s) [Option ID = 2698]
For a polymer, which of the following statement/s is/are true? [Question ID = 759]
1. Weight average molecular weight is almost always higher than the number average molecular weight [Option ID = 3035]
2. Formation of a polypeptide from its monomers (amino acids) is an example of addition polymerization [Option ID = 3034]
3. All of these [Option ID = 3036]
4. Vinyl polymerization is an example of condensation polymerization. [Option ID = 3033]
Correct Answer :-
Weight average molecular weight is almost always higher than the number average molecular weight [Option ID = 3035]
Quantum dots are [Question ID = 762]
1. Three dimensional [Option ID = 3048]
2. One dimensional [Option ID = 3046]
3. Two dimensional [Option ID = 3047]
4. Zero dimensional [Option ID = 3045]
Correct Answer :-
Zero dimensional [Option ID = 3045]
The unit of rate constant for a third order reaction is: [Question ID = 749]
1. [Option ID = 2993]
2. [Option ID = 2995]
3. [Option ID = 2996]
4. [Option ID = 2994]
Correct Answer :-
[Option ID = 2996]
All the following elements have at least one isotope that is not radioactive EXCEPT [Question ID = 673]
1. Pb [Option ID = 2690]
2. O [Option ID = 2689]
3. Sn [Option ID = 2691]
4. No [Option ID = 2692]
Correct Answer :-
No [Option ID = 2692]
The conditions for a species to follow Bose-Einstein statistics are; [Question ID = 736]
1. Particles are indistinguishable, with no restriction on filling up of energy levels [Option ID = 2944]
2. Particles are indistinguishable, with a restriction on filling up of energy levels [Option ID = 2943]
3. Particles are distinguishable, with a restriction on filling up of energy levels [Option ID = 2941]
4. Particles are distinguishable, with no restriction on filling up of energy levels [Option ID = 2942]
Correct Answer :-
Particles are indistinguishable, with no restriction on filling up of energy levels [Option ID = 2944]
In the kinetic theory of collisions, the SI unit of collision number, in terms of m (meter) and s (second), is:
[Question ID = 761]
25)
26)
27)
28)
29)
1. m-
2
s
-1
[Option ID = 3042]
2. m
4
s
-1
[Option ID = 3041]
3. m
2
s
-1
[Option ID = 3043]
4. None of these [Option ID = 3044]
Correct Answer :-
None of these [Option ID = 3044]
Correct characteristics of the functional groups of adenine in DNA base pair are [Question ID = 706]
1. [Option ID = 2824]
2. [Option ID = 2821]
3. [Option ID = 2823]
4. [Option ID = 2822]
Correct Answer :-
[Option ID = 2822]
The carbon monoxide molecule has an internuclear distance of 1.13 Angstroms. What is the moment of Inertia of this molecule?
[Question ID = 740]
1. [Option ID = 2960]
2. [Option ID = 2957]
3. [Option ID = 2958]
4. [Option ID = 2959]
Correct Answer :-
[Option ID = 2957]
Which of the following represent/s non-linear optical technique? [Question ID = 744]
1. Second Harmonic generation [Option ID = 2974]
2. Two-photon photoluminescence [Option ID = 2975]
3. Four-wave mixing [Option ID = 2973]
4. All of these [Option ID = 2976]
Correct Answer :-
All of these [Option ID = 2976]
. Which of the following does not affect the intensity of spectral lines of a sample? [Question ID = 743]
1. Path length of a sample [Option ID = 2972]
2. Population of energy states [Option ID = 2970]
3. Heisenberg?s Uncertainty principle [Option ID = 2971]
4. Concentration of a sample [Option ID = 2969]
Correct Answer :-
Heisenberg?s Uncertainty principle [Option ID = 2971]
[Question ID = 716]
30)
31)
1. [Option ID = 2862]
2. [Option ID = 2863]
3. [Option ID = 2861]
4. [Option ID = 2864]
Correct Answer :-
[Option ID = 2864]
[Question ID = 712]
1. [Option ID = 2845]
2. [Option ID = 2846]
3. [Option ID = 2848]
4. [Option ID = 2847]
Correct Answer :-
[Option ID = 2847]
[Question ID = 686]
1. [Option ID = 2743]
2. [Option ID = 2742]
3. [Option ID = 2741]
32)
33)
4. [Option ID = 2744]
Correct Answer :-
[Option ID = 2743]
[Question ID = 5633]
1. [Option ID = 22523]
2. [Option ID = 22524]
3. [Option ID = 22526]
4. [Option ID = 22525]
Correct Answer :-
[Option ID = 22523]
[Question ID = 730]
1. [Option ID = 2918]
2. [Option ID = 2920]
3. [Option ID = 2917]
4. [Option ID = 2919]
Correct Answer :-
34)
35)
36)
[Option ID = 2919]
[Question ID = 714]
1. [Option ID = 2856]
2. [Option ID = 2853]
3. [Option ID = 2855]
4. [Option ID = 2854]
Correct Answer :-
[Option ID = 2854]
[Question ID = 669]
1. 2 [Option ID = 2673]
2. 18 [Option ID = 2676]
3. 5 [Option ID = 2674]
4. 10 [Option ID = 2675]
Correct Answer :-
5 [Option ID = 2674]
[Question ID = 689]
1. 2 [Option ID = 2754]
2. 0 [Option ID = 2756]
3. 1 [Option ID = 2755]
37)
38)
4. 3 [Option ID = 2753]
Correct Answer :-
1 [Option ID = 2755]
[Question ID = 718]
1. C) [Option ID = 2871]
2. B) [Option ID = 2870]
3. A) [Option ID = 2869]
4. D) both (B) and (C) [Option ID = 2872]
Correct Answer :-
A) [Option ID = 2869]
[Question ID = 709]
1. [Option ID = 2836]
2. [Option ID = 2833]
3. [Option ID = 2835]
4. [Option ID = 2834]
39)
40)
41)
42)
Correct Answer :-
[Option ID = 2836]
[Question ID = 705]
1. (2Z, 4Z)-3-chlorohexa-2, 4-diene-1,6-diol. [Option ID = 2819]
2. (2E, 4E)-3-chlorohexa-2, 4-diene-1,6-diol. [Option ID = 2817]
3. (2Z, 4E)-3-chlorohexa-2, 4-diene-1,6-diol. [Option ID = 2818]
4. (2Z, 4E)-3-chlorohexa-2, 4-diene-1,6-diol. [Option ID = 2820]
Correct Answer :-
(2Z, 4E)-3-chlorohexa-2, 4-diene-1,6-diol. [Option ID = 2820]
[Question ID = 680]
1. The presence of other complexing ligands in solution affects the equilibrium concentration of metal-EDTA complexes [Option ID = 2719]
2. Metal-EDTA complexes have an equilibrium concentration independent of pH [Option ID = 2718]
3. Metal-EDTA complexes are often 2:1 in stoichiometry [Option ID = 2717]
4. Metal-EDTA complexes are less stable than the corresponding metal-ammine complexes [Option ID = 2720]
Correct Answer :-
The presence of other complexing ligands in solution affects the equilibrium concentration of metal-EDTA complexes [Option ID = 2719]
[Question ID = 671]
1. 150 g [Option ID = 2682]
2. 300 g [Option ID = 2684]
3. 120 g [Option ID = 2681]
4. 180 g [Option ID = 2683]
Correct Answer :-
300 g [Option ID = 2684]
[Question ID = 756]
1. [Option ID = 3021]
2. [Option ID = 3022]
3. [Option ID = 3024]
4. [Option ID = 3023]
Correct Answer :-
43)
44)
45)
[Option ID = 3024]
[Question ID = 750]
1. [Option ID = 2999]
2. [Option ID = 3000]
3. [Option ID = 2997]
4. [Option ID = 2998]
Correct Answer :-
[Option ID = 2997]
[Question ID = 715]
1. [Option ID = 2857]
2. [Option ID = 2860]
3. [Option ID = 2858]
4. [Option ID = 2859]
Correct Answer :-
[Option ID = 2860]
46)
47)
[Question ID = 719]
1. [Option ID = 2873]
2. [Option ID = 2876]
3. [Option ID = 2874]
4. [Option ID = 2875]
Correct Answer :-
[Question ID = 674]
1. Evaporating some water to decrease the volume of the solution. [Option ID = 2696]
2. [Option ID = 2695]
3. [Option ID = 2694]
4. [Option ID = 2693]
Correct Answer :-
[Option ID = 2693]
[Question ID = 728]
1. [Option ID = 2910]
2. [Option ID = 2911]
3. [Option ID = 2909]
4. [Option ID = 2912]
48)
49)
50)
51)
Correct Answer :-
[Option ID = 2912]
[Question ID = 697]
1. III only [Option ID = 2788]
2. II only [Option ID = 2787]
3. I only [Option ID = 2785]
4. I, II and III [Option ID = 2786]
Correct Answer :-
I, II and III [Option ID = 2786]
[Question ID = 700]
1. [Option ID = 2798]
2. [Option ID = 2797]
3. [Option ID = 2800]
4. [Option ID = 2799]
Correct Answer :-
[Option ID = 2799]
[Question ID = 678]
1. Cr
2+
has a low cationic charge [Option ID = 2711]
2. F has -1 anionic charge and highly electronegative [Option ID = 2709]
3. Spin-orbit coupling in Cr
2+
[Option ID = 2712]
4. The Jahn-Teller effect [Option ID = 2710]
Correct Answer :-
The Jahn-Teller effect [Option ID = 2710]
[Question ID = 723]
1. [Option ID = 2889]
52)
53)
2. [Option ID = 2890]
3. [Option ID = 2891]
4. [Option ID = 2892]
Correct Answer :-
[Option ID = 2889]
[Question ID = 727]
1. [Option ID = 2907]
2. [Option ID = 2906]
3. [Option ID = 2905]
4. [Option ID = 2908]
Correct Answer :-
[Option ID = 2905]
[Question ID = 707]
54)
55)
56)
1. N9 [Option ID = 2828]
2. N7 [Option ID = 2827]
3. N1 [Option ID = 2825]
4. N3 [Option ID = 2826]
Correct Answer :-
N7 [Option ID = 2827]
[Question ID = 672]
1. Bicapped prism [Option ID = 2688]
2. Square pyramidal [Option ID = 2686]
3. Trigonal planar [Option ID = 2685]
4. Bent [Option ID = 2687]
Correct Answer :-
Square pyramidal [Option ID = 2686]
[Question ID = 704]
1. [Option ID = 2816]
2. [Option ID = 2814]
3. [Option ID = 2813]
4. [Option ID = 2815]
Correct Answer :-
[Option ID = 2813]
57)
58)
[Question ID = 708]
1. [Option ID = 2829]
2. [Option ID = 2832]
3. [Option ID = 2831]
4. [Option ID = 2830]
Correct Answer :-
[Option ID = 2831]
[Question ID = 717]
1. [Option ID = 2866]
2. [Option ID = 2865]
3. [Option ID = 2867]
4. [Option ID = 2868]
Correct Answer :-
[Option ID = 2866]
FirstRanker.com - FirstRanker's Choice
1)
2)
3)
4)
5)
DU PhD in Chemistry
Topic:- DU_J18_PHD_CHEM
Which of the following statements about sulfur dioxide is true?
[Question ID = 677]
1. It forms a S-S dimer in condensed phase [Option ID = 2707]
2. Its anhydride of sulfuric acid [Option ID = 2706]
3. Its O-S-O angle is 180
0
[Option ID = 2708]
4. It is a product of the combustion of fossil fuels that contain sulfur [Option ID = 2705]
Correct Answer :-
It is a product of the combustion of fossil fuels that contain sulfur [Option ID = 2705]
Which of the following is a strong acid in pure liquid HF
[Question ID = 683]
1. H
2
O [Option ID = 2731]
2. NaF [Option ID = 2729]
3. CH
3
COOH [Option ID = 2730]
4. SbF
5
[Option ID = 2732]
Correct Answer :-
SbF
5
[Option ID = 2732]
Each of the following molecules can act as a chelating agent EXCEPT [Question ID = 679]
1. [Option ID = 2716]
2. [Option ID = 2714]
3. [Option ID = 2713]
4. [Option ID = 2715]
Correct Answer :-
[Option ID = 2714]
What is correct about h-index?
[Question ID = 758]
1. Alternative of impact factor [Option ID = 3031]
2. Based on most quoted papers [Option ID = 3030]
3. Quantify scientific productivity [Option ID = 3029]
4. All of these [Option ID = 3032]
Correct Answer :-
All of these [Option ID = 3032]
The hyperfine electron spin resonance (e.s.r.) spectrum of the benzene radical has how many lines? [Question ID = 748]
1. 12 [Option ID = 2992]
2. 7 [Option ID = 2990]
3. 1 [Option ID = 2991]
4. 6 [Option ID = 2989]
Correct Answer :-
7 [Option ID = 2990]
6)
7)
8)
9)
10)
11)
12)
The energy changes involving the core electrons of an atom or molecule are expressed in which region of the electromagnetic
spectrum? [Question ID = 742]
1. Ultraviolet and Visible region [Option ID = 2967]
2. X-ray region [Option ID = 2968]
3. Radiofrequency region [Option ID = 2966]
4. Infra-red region [Option ID = 2965]
Correct Answer :-
X-ray region [Option ID = 2968]
Find out the expected intensity ratio of M and M+1 signal for the Naphthalene molecular ion [Question ID = 726]
1. 99:1.1 [Option ID = 2903]
2. 1.1:99 [Option ID = 2904]
3. 9:01 [Option ID = 2901]
4. 1:9 [Option ID = 2902]
Correct Answer :-
9:01 [Option ID = 2901]
Cobalt-60 is used in radiation therapy of cancer and can be produced by the bombardment of Cobalt-59 with [Question ID = 692]
1. Alpha particles [Option ID = 2765]
2. Beta particles [Option ID = 2767]
3. Neutrons [Option ID = 2766]
4. Gamma rays [Option ID = 2768]
Correct Answer :-
Neutrons [Option ID = 2766]
The standard emf of galvanic cell involving 3 moles of electrons in its redox reaction is 0.59 V. The equilibrium constant for the
reaction of the cell is- [Question ID = 763]
1. [Option ID = 3051]
2. [Option ID = 3052]
3. [Option ID = 3049]
4. [Option ID = 3050]
Correct Answer :-
[Option ID = 3052]
A characteristic common to polymers that can be made to conduct electricity such as polyacetylene, polypyrrole is: [Question ID =
685]
1. Conjugation throughout the polymeric chain. [Option ID = 2740]
2. A high degree of cross linking [Option ID = 2738]
3. A very low glass transition temperature [Option ID = 2737]
4. Presence of stereogenic centers of the same configuration [Option ID = 2739]
Correct Answer :-
Conjugation throughout the polymeric chain. [Option ID = 2740]
Impact factor is [Question ID = 768]
1. Ratio between citations and recent citable items publish [Option ID = 3071]
2. All of these [Option ID = 3072]
3. Addition of citations and recent citable items publish [Option ID = 3069]
4. Ratio between recent citable items publish and citations [Option ID = 3070]
Correct Answer :-
Ratio between recent citable items publish and citations [Option ID = 3070]
On the basis of oxidation-reduction potential, which of the following is most likely to occur? [Question ID = 693]
13)
14)
15)
16)
17)
1. [Option ID = 2770]
2. [Option ID = 2772]
3. [Option ID = 2771]
4. [Option ID = 2769]
Correct Answer :-
[Option ID = 2769]
How many diastereoisomers are possible for the compound 2, 4 ?diphenylcyclobutane-1, 3 di carboxylic acids. [Question ID = 725]
1. 6 [Option ID = 2899]
2. 5 [Option ID = 2898]
3. 8 [Option ID = 2900]
4. 4 [Option ID = 2897]
Correct Answer :-
5 [Option ID = 2898]
An increase in equivalent conductance of a strong electrolyte with dilution is mainly due to- [Question ID = 764]
1. increase in ionic mobility of ions [Option ID = 3055]
2. increase in number of ions [Option ID = 3054]
3. 100% ionization of electrolyte at normal dilution [Option ID = 3056]
4. increase in both i.e. number of ions and ionic mobility of ions. [Option ID = 3053]
Correct Answer :-
increase in ionic mobility of ions [Option ID = 3055]
The solid state structures of the principal allotropes of elemental boron are made up of which of the following structural units
[Question ID = 699]
1. [Option ID = 2796]
2. [Option ID = 2795]
3. [Option ID = 2794]
4. [Option ID = 2793]
Correct Answer :-
[Option ID = 2793]
The molecular geometry of thionyl chloride is best described as [Question ID = 688]
1. T-shaped [Option ID = 2752]
2. Tetrahedral [Option ID = 2751]
3. Trigonal pyramidal [Option ID = 2749]
4. Trigonal planar [Option ID = 2750]
Correct Answer :-
Trigonal pyramidal [Option ID = 2749]
In a face-center cubic (FCC) type of crystal lattice, the number of atoms belonging exclusively to each unit cell within the lattice
is/are: [Question ID = 754]
1. 2 [Option ID = 3014]
2. 1 [Option ID = 3013]
3. 3 [Option ID = 3015]
4. 4 [Option ID = 3016]
Correct Answer :-
4 [Option ID = 3016]
18)
19)
20)
21)
22)
23)
24)
Among the following, the weakest oxidizing agent is [Question ID = 675]
1. Mg (s) [Option ID = 2698]
2. [Option ID = 2699]
3. [Option ID = 2700]
4. [Option ID = 2697]
Correct Answer :-
Mg (s) [Option ID = 2698]
For a polymer, which of the following statement/s is/are true? [Question ID = 759]
1. Weight average molecular weight is almost always higher than the number average molecular weight [Option ID = 3035]
2. Formation of a polypeptide from its monomers (amino acids) is an example of addition polymerization [Option ID = 3034]
3. All of these [Option ID = 3036]
4. Vinyl polymerization is an example of condensation polymerization. [Option ID = 3033]
Correct Answer :-
Weight average molecular weight is almost always higher than the number average molecular weight [Option ID = 3035]
Quantum dots are [Question ID = 762]
1. Three dimensional [Option ID = 3048]
2. One dimensional [Option ID = 3046]
3. Two dimensional [Option ID = 3047]
4. Zero dimensional [Option ID = 3045]
Correct Answer :-
Zero dimensional [Option ID = 3045]
The unit of rate constant for a third order reaction is: [Question ID = 749]
1. [Option ID = 2993]
2. [Option ID = 2995]
3. [Option ID = 2996]
4. [Option ID = 2994]
Correct Answer :-
[Option ID = 2996]
All the following elements have at least one isotope that is not radioactive EXCEPT [Question ID = 673]
1. Pb [Option ID = 2690]
2. O [Option ID = 2689]
3. Sn [Option ID = 2691]
4. No [Option ID = 2692]
Correct Answer :-
No [Option ID = 2692]
The conditions for a species to follow Bose-Einstein statistics are; [Question ID = 736]
1. Particles are indistinguishable, with no restriction on filling up of energy levels [Option ID = 2944]
2. Particles are indistinguishable, with a restriction on filling up of energy levels [Option ID = 2943]
3. Particles are distinguishable, with a restriction on filling up of energy levels [Option ID = 2941]
4. Particles are distinguishable, with no restriction on filling up of energy levels [Option ID = 2942]
Correct Answer :-
Particles are indistinguishable, with no restriction on filling up of energy levels [Option ID = 2944]
In the kinetic theory of collisions, the SI unit of collision number, in terms of m (meter) and s (second), is:
[Question ID = 761]
25)
26)
27)
28)
29)
1. m-
2
s
-1
[Option ID = 3042]
2. m
4
s
-1
[Option ID = 3041]
3. m
2
s
-1
[Option ID = 3043]
4. None of these [Option ID = 3044]
Correct Answer :-
None of these [Option ID = 3044]
Correct characteristics of the functional groups of adenine in DNA base pair are [Question ID = 706]
1. [Option ID = 2824]
2. [Option ID = 2821]
3. [Option ID = 2823]
4. [Option ID = 2822]
Correct Answer :-
[Option ID = 2822]
The carbon monoxide molecule has an internuclear distance of 1.13 Angstroms. What is the moment of Inertia of this molecule?
[Question ID = 740]
1. [Option ID = 2960]
2. [Option ID = 2957]
3. [Option ID = 2958]
4. [Option ID = 2959]
Correct Answer :-
[Option ID = 2957]
Which of the following represent/s non-linear optical technique? [Question ID = 744]
1. Second Harmonic generation [Option ID = 2974]
2. Two-photon photoluminescence [Option ID = 2975]
3. Four-wave mixing [Option ID = 2973]
4. All of these [Option ID = 2976]
Correct Answer :-
All of these [Option ID = 2976]
. Which of the following does not affect the intensity of spectral lines of a sample? [Question ID = 743]
1. Path length of a sample [Option ID = 2972]
2. Population of energy states [Option ID = 2970]
3. Heisenberg?s Uncertainty principle [Option ID = 2971]
4. Concentration of a sample [Option ID = 2969]
Correct Answer :-
Heisenberg?s Uncertainty principle [Option ID = 2971]
[Question ID = 716]
30)
31)
1. [Option ID = 2862]
2. [Option ID = 2863]
3. [Option ID = 2861]
4. [Option ID = 2864]
Correct Answer :-
[Option ID = 2864]
[Question ID = 712]
1. [Option ID = 2845]
2. [Option ID = 2846]
3. [Option ID = 2848]
4. [Option ID = 2847]
Correct Answer :-
[Option ID = 2847]
[Question ID = 686]
1. [Option ID = 2743]
2. [Option ID = 2742]
3. [Option ID = 2741]
32)
33)
4. [Option ID = 2744]
Correct Answer :-
[Option ID = 2743]
[Question ID = 5633]
1. [Option ID = 22523]
2. [Option ID = 22524]
3. [Option ID = 22526]
4. [Option ID = 22525]
Correct Answer :-
[Option ID = 22523]
[Question ID = 730]
1. [Option ID = 2918]
2. [Option ID = 2920]
3. [Option ID = 2917]
4. [Option ID = 2919]
Correct Answer :-
34)
35)
36)
[Option ID = 2919]
[Question ID = 714]
1. [Option ID = 2856]
2. [Option ID = 2853]
3. [Option ID = 2855]
4. [Option ID = 2854]
Correct Answer :-
[Option ID = 2854]
[Question ID = 669]
1. 2 [Option ID = 2673]
2. 18 [Option ID = 2676]
3. 5 [Option ID = 2674]
4. 10 [Option ID = 2675]
Correct Answer :-
5 [Option ID = 2674]
[Question ID = 689]
1. 2 [Option ID = 2754]
2. 0 [Option ID = 2756]
3. 1 [Option ID = 2755]
37)
38)
4. 3 [Option ID = 2753]
Correct Answer :-
1 [Option ID = 2755]
[Question ID = 718]
1. C) [Option ID = 2871]
2. B) [Option ID = 2870]
3. A) [Option ID = 2869]
4. D) both (B) and (C) [Option ID = 2872]
Correct Answer :-
A) [Option ID = 2869]
[Question ID = 709]
1. [Option ID = 2836]
2. [Option ID = 2833]
3. [Option ID = 2835]
4. [Option ID = 2834]
39)
40)
41)
42)
Correct Answer :-
[Option ID = 2836]
[Question ID = 705]
1. (2Z, 4Z)-3-chlorohexa-2, 4-diene-1,6-diol. [Option ID = 2819]
2. (2E, 4E)-3-chlorohexa-2, 4-diene-1,6-diol. [Option ID = 2817]
3. (2Z, 4E)-3-chlorohexa-2, 4-diene-1,6-diol. [Option ID = 2818]
4. (2Z, 4E)-3-chlorohexa-2, 4-diene-1,6-diol. [Option ID = 2820]
Correct Answer :-
(2Z, 4E)-3-chlorohexa-2, 4-diene-1,6-diol. [Option ID = 2820]
[Question ID = 680]
1. The presence of other complexing ligands in solution affects the equilibrium concentration of metal-EDTA complexes [Option ID = 2719]
2. Metal-EDTA complexes have an equilibrium concentration independent of pH [Option ID = 2718]
3. Metal-EDTA complexes are often 2:1 in stoichiometry [Option ID = 2717]
4. Metal-EDTA complexes are less stable than the corresponding metal-ammine complexes [Option ID = 2720]
Correct Answer :-
The presence of other complexing ligands in solution affects the equilibrium concentration of metal-EDTA complexes [Option ID = 2719]
[Question ID = 671]
1. 150 g [Option ID = 2682]
2. 300 g [Option ID = 2684]
3. 120 g [Option ID = 2681]
4. 180 g [Option ID = 2683]
Correct Answer :-
300 g [Option ID = 2684]
[Question ID = 756]
1. [Option ID = 3021]
2. [Option ID = 3022]
3. [Option ID = 3024]
4. [Option ID = 3023]
Correct Answer :-
43)
44)
45)
[Option ID = 3024]
[Question ID = 750]
1. [Option ID = 2999]
2. [Option ID = 3000]
3. [Option ID = 2997]
4. [Option ID = 2998]
Correct Answer :-
[Option ID = 2997]
[Question ID = 715]
1. [Option ID = 2857]
2. [Option ID = 2860]
3. [Option ID = 2858]
4. [Option ID = 2859]
Correct Answer :-
[Option ID = 2860]
46)
47)
[Question ID = 719]
1. [Option ID = 2873]
2. [Option ID = 2876]
3. [Option ID = 2874]
4. [Option ID = 2875]
Correct Answer :-
[Question ID = 674]
1. Evaporating some water to decrease the volume of the solution. [Option ID = 2696]
2. [Option ID = 2695]
3. [Option ID = 2694]
4. [Option ID = 2693]
Correct Answer :-
[Option ID = 2693]
[Question ID = 728]
1. [Option ID = 2910]
2. [Option ID = 2911]
3. [Option ID = 2909]
4. [Option ID = 2912]
48)
49)
50)
51)
Correct Answer :-
[Option ID = 2912]
[Question ID = 697]
1. III only [Option ID = 2788]
2. II only [Option ID = 2787]
3. I only [Option ID = 2785]
4. I, II and III [Option ID = 2786]
Correct Answer :-
I, II and III [Option ID = 2786]
[Question ID = 700]
1. [Option ID = 2798]
2. [Option ID = 2797]
3. [Option ID = 2800]
4. [Option ID = 2799]
Correct Answer :-
[Option ID = 2799]
[Question ID = 678]
1. Cr
2+
has a low cationic charge [Option ID = 2711]
2. F has -1 anionic charge and highly electronegative [Option ID = 2709]
3. Spin-orbit coupling in Cr
2+
[Option ID = 2712]
4. The Jahn-Teller effect [Option ID = 2710]
Correct Answer :-
The Jahn-Teller effect [Option ID = 2710]
[Question ID = 723]
1. [Option ID = 2889]
52)
53)
2. [Option ID = 2890]
3. [Option ID = 2891]
4. [Option ID = 2892]
Correct Answer :-
[Option ID = 2889]
[Question ID = 727]
1. [Option ID = 2907]
2. [Option ID = 2906]
3. [Option ID = 2905]
4. [Option ID = 2908]
Correct Answer :-
[Option ID = 2905]
[Question ID = 707]
54)
55)
56)
1. N9 [Option ID = 2828]
2. N7 [Option ID = 2827]
3. N1 [Option ID = 2825]
4. N3 [Option ID = 2826]
Correct Answer :-
N7 [Option ID = 2827]
[Question ID = 672]
1. Bicapped prism [Option ID = 2688]
2. Square pyramidal [Option ID = 2686]
3. Trigonal planar [Option ID = 2685]
4. Bent [Option ID = 2687]
Correct Answer :-
Square pyramidal [Option ID = 2686]
[Question ID = 704]
1. [Option ID = 2816]
2. [Option ID = 2814]
3. [Option ID = 2813]
4. [Option ID = 2815]
Correct Answer :-
[Option ID = 2813]
57)
58)
[Question ID = 708]
1. [Option ID = 2829]
2. [Option ID = 2832]
3. [Option ID = 2831]
4. [Option ID = 2830]
Correct Answer :-
[Option ID = 2831]
[Question ID = 717]
1. [Option ID = 2866]
2. [Option ID = 2865]
3. [Option ID = 2867]
4. [Option ID = 2868]
Correct Answer :-
[Option ID = 2866]
59)
[Question ID = 711]
1. [Option ID = 2843]
2. [Option ID = 2842]
3. [Option ID = 2844]
4. [Option ID = 2841]
Correct Answer :-
[Option ID = 2842]
[Question ID = 710]
1. [Option ID = 2838]
2. [Option ID = 2839]
3. [Option ID = 2837]
4. [Option ID = 2840]
Correct Answer :-
FirstRanker.com - FirstRanker's Choice
1)
2)
3)
4)
5)
DU PhD in Chemistry
Topic:- DU_J18_PHD_CHEM
Which of the following statements about sulfur dioxide is true?
[Question ID = 677]
1. It forms a S-S dimer in condensed phase [Option ID = 2707]
2. Its anhydride of sulfuric acid [Option ID = 2706]
3. Its O-S-O angle is 180
0
[Option ID = 2708]
4. It is a product of the combustion of fossil fuels that contain sulfur [Option ID = 2705]
Correct Answer :-
It is a product of the combustion of fossil fuels that contain sulfur [Option ID = 2705]
Which of the following is a strong acid in pure liquid HF
[Question ID = 683]
1. H
2
O [Option ID = 2731]
2. NaF [Option ID = 2729]
3. CH
3
COOH [Option ID = 2730]
4. SbF
5
[Option ID = 2732]
Correct Answer :-
SbF
5
[Option ID = 2732]
Each of the following molecules can act as a chelating agent EXCEPT [Question ID = 679]
1. [Option ID = 2716]
2. [Option ID = 2714]
3. [Option ID = 2713]
4. [Option ID = 2715]
Correct Answer :-
[Option ID = 2714]
What is correct about h-index?
[Question ID = 758]
1. Alternative of impact factor [Option ID = 3031]
2. Based on most quoted papers [Option ID = 3030]
3. Quantify scientific productivity [Option ID = 3029]
4. All of these [Option ID = 3032]
Correct Answer :-
All of these [Option ID = 3032]
The hyperfine electron spin resonance (e.s.r.) spectrum of the benzene radical has how many lines? [Question ID = 748]
1. 12 [Option ID = 2992]
2. 7 [Option ID = 2990]
3. 1 [Option ID = 2991]
4. 6 [Option ID = 2989]
Correct Answer :-
7 [Option ID = 2990]
6)
7)
8)
9)
10)
11)
12)
The energy changes involving the core electrons of an atom or molecule are expressed in which region of the electromagnetic
spectrum? [Question ID = 742]
1. Ultraviolet and Visible region [Option ID = 2967]
2. X-ray region [Option ID = 2968]
3. Radiofrequency region [Option ID = 2966]
4. Infra-red region [Option ID = 2965]
Correct Answer :-
X-ray region [Option ID = 2968]
Find out the expected intensity ratio of M and M+1 signal for the Naphthalene molecular ion [Question ID = 726]
1. 99:1.1 [Option ID = 2903]
2. 1.1:99 [Option ID = 2904]
3. 9:01 [Option ID = 2901]
4. 1:9 [Option ID = 2902]
Correct Answer :-
9:01 [Option ID = 2901]
Cobalt-60 is used in radiation therapy of cancer and can be produced by the bombardment of Cobalt-59 with [Question ID = 692]
1. Alpha particles [Option ID = 2765]
2. Beta particles [Option ID = 2767]
3. Neutrons [Option ID = 2766]
4. Gamma rays [Option ID = 2768]
Correct Answer :-
Neutrons [Option ID = 2766]
The standard emf of galvanic cell involving 3 moles of electrons in its redox reaction is 0.59 V. The equilibrium constant for the
reaction of the cell is- [Question ID = 763]
1. [Option ID = 3051]
2. [Option ID = 3052]
3. [Option ID = 3049]
4. [Option ID = 3050]
Correct Answer :-
[Option ID = 3052]
A characteristic common to polymers that can be made to conduct electricity such as polyacetylene, polypyrrole is: [Question ID =
685]
1. Conjugation throughout the polymeric chain. [Option ID = 2740]
2. A high degree of cross linking [Option ID = 2738]
3. A very low glass transition temperature [Option ID = 2737]
4. Presence of stereogenic centers of the same configuration [Option ID = 2739]
Correct Answer :-
Conjugation throughout the polymeric chain. [Option ID = 2740]
Impact factor is [Question ID = 768]
1. Ratio between citations and recent citable items publish [Option ID = 3071]
2. All of these [Option ID = 3072]
3. Addition of citations and recent citable items publish [Option ID = 3069]
4. Ratio between recent citable items publish and citations [Option ID = 3070]
Correct Answer :-
Ratio between recent citable items publish and citations [Option ID = 3070]
On the basis of oxidation-reduction potential, which of the following is most likely to occur? [Question ID = 693]
13)
14)
15)
16)
17)
1. [Option ID = 2770]
2. [Option ID = 2772]
3. [Option ID = 2771]
4. [Option ID = 2769]
Correct Answer :-
[Option ID = 2769]
How many diastereoisomers are possible for the compound 2, 4 ?diphenylcyclobutane-1, 3 di carboxylic acids. [Question ID = 725]
1. 6 [Option ID = 2899]
2. 5 [Option ID = 2898]
3. 8 [Option ID = 2900]
4. 4 [Option ID = 2897]
Correct Answer :-
5 [Option ID = 2898]
An increase in equivalent conductance of a strong electrolyte with dilution is mainly due to- [Question ID = 764]
1. increase in ionic mobility of ions [Option ID = 3055]
2. increase in number of ions [Option ID = 3054]
3. 100% ionization of electrolyte at normal dilution [Option ID = 3056]
4. increase in both i.e. number of ions and ionic mobility of ions. [Option ID = 3053]
Correct Answer :-
increase in ionic mobility of ions [Option ID = 3055]
The solid state structures of the principal allotropes of elemental boron are made up of which of the following structural units
[Question ID = 699]
1. [Option ID = 2796]
2. [Option ID = 2795]
3. [Option ID = 2794]
4. [Option ID = 2793]
Correct Answer :-
[Option ID = 2793]
The molecular geometry of thionyl chloride is best described as [Question ID = 688]
1. T-shaped [Option ID = 2752]
2. Tetrahedral [Option ID = 2751]
3. Trigonal pyramidal [Option ID = 2749]
4. Trigonal planar [Option ID = 2750]
Correct Answer :-
Trigonal pyramidal [Option ID = 2749]
In a face-center cubic (FCC) type of crystal lattice, the number of atoms belonging exclusively to each unit cell within the lattice
is/are: [Question ID = 754]
1. 2 [Option ID = 3014]
2. 1 [Option ID = 3013]
3. 3 [Option ID = 3015]
4. 4 [Option ID = 3016]
Correct Answer :-
4 [Option ID = 3016]
18)
19)
20)
21)
22)
23)
24)
Among the following, the weakest oxidizing agent is [Question ID = 675]
1. Mg (s) [Option ID = 2698]
2. [Option ID = 2699]
3. [Option ID = 2700]
4. [Option ID = 2697]
Correct Answer :-
Mg (s) [Option ID = 2698]
For a polymer, which of the following statement/s is/are true? [Question ID = 759]
1. Weight average molecular weight is almost always higher than the number average molecular weight [Option ID = 3035]
2. Formation of a polypeptide from its monomers (amino acids) is an example of addition polymerization [Option ID = 3034]
3. All of these [Option ID = 3036]
4. Vinyl polymerization is an example of condensation polymerization. [Option ID = 3033]
Correct Answer :-
Weight average molecular weight is almost always higher than the number average molecular weight [Option ID = 3035]
Quantum dots are [Question ID = 762]
1. Three dimensional [Option ID = 3048]
2. One dimensional [Option ID = 3046]
3. Two dimensional [Option ID = 3047]
4. Zero dimensional [Option ID = 3045]
Correct Answer :-
Zero dimensional [Option ID = 3045]
The unit of rate constant for a third order reaction is: [Question ID = 749]
1. [Option ID = 2993]
2. [Option ID = 2995]
3. [Option ID = 2996]
4. [Option ID = 2994]
Correct Answer :-
[Option ID = 2996]
All the following elements have at least one isotope that is not radioactive EXCEPT [Question ID = 673]
1. Pb [Option ID = 2690]
2. O [Option ID = 2689]
3. Sn [Option ID = 2691]
4. No [Option ID = 2692]
Correct Answer :-
No [Option ID = 2692]
The conditions for a species to follow Bose-Einstein statistics are; [Question ID = 736]
1. Particles are indistinguishable, with no restriction on filling up of energy levels [Option ID = 2944]
2. Particles are indistinguishable, with a restriction on filling up of energy levels [Option ID = 2943]
3. Particles are distinguishable, with a restriction on filling up of energy levels [Option ID = 2941]
4. Particles are distinguishable, with no restriction on filling up of energy levels [Option ID = 2942]
Correct Answer :-
Particles are indistinguishable, with no restriction on filling up of energy levels [Option ID = 2944]
In the kinetic theory of collisions, the SI unit of collision number, in terms of m (meter) and s (second), is:
[Question ID = 761]
25)
26)
27)
28)
29)
1. m-
2
s
-1
[Option ID = 3042]
2. m
4
s
-1
[Option ID = 3041]
3. m
2
s
-1
[Option ID = 3043]
4. None of these [Option ID = 3044]
Correct Answer :-
None of these [Option ID = 3044]
Correct characteristics of the functional groups of adenine in DNA base pair are [Question ID = 706]
1. [Option ID = 2824]
2. [Option ID = 2821]
3. [Option ID = 2823]
4. [Option ID = 2822]
Correct Answer :-
[Option ID = 2822]
The carbon monoxide molecule has an internuclear distance of 1.13 Angstroms. What is the moment of Inertia of this molecule?
[Question ID = 740]
1. [Option ID = 2960]
2. [Option ID = 2957]
3. [Option ID = 2958]
4. [Option ID = 2959]
Correct Answer :-
[Option ID = 2957]
Which of the following represent/s non-linear optical technique? [Question ID = 744]
1. Second Harmonic generation [Option ID = 2974]
2. Two-photon photoluminescence [Option ID = 2975]
3. Four-wave mixing [Option ID = 2973]
4. All of these [Option ID = 2976]
Correct Answer :-
All of these [Option ID = 2976]
. Which of the following does not affect the intensity of spectral lines of a sample? [Question ID = 743]
1. Path length of a sample [Option ID = 2972]
2. Population of energy states [Option ID = 2970]
3. Heisenberg?s Uncertainty principle [Option ID = 2971]
4. Concentration of a sample [Option ID = 2969]
Correct Answer :-
Heisenberg?s Uncertainty principle [Option ID = 2971]
[Question ID = 716]
30)
31)
1. [Option ID = 2862]
2. [Option ID = 2863]
3. [Option ID = 2861]
4. [Option ID = 2864]
Correct Answer :-
[Option ID = 2864]
[Question ID = 712]
1. [Option ID = 2845]
2. [Option ID = 2846]
3. [Option ID = 2848]
4. [Option ID = 2847]
Correct Answer :-
[Option ID = 2847]
[Question ID = 686]
1. [Option ID = 2743]
2. [Option ID = 2742]
3. [Option ID = 2741]
32)
33)
4. [Option ID = 2744]
Correct Answer :-
[Option ID = 2743]
[Question ID = 5633]
1. [Option ID = 22523]
2. [Option ID = 22524]
3. [Option ID = 22526]
4. [Option ID = 22525]
Correct Answer :-
[Option ID = 22523]
[Question ID = 730]
1. [Option ID = 2918]
2. [Option ID = 2920]
3. [Option ID = 2917]
4. [Option ID = 2919]
Correct Answer :-
34)
35)
36)
[Option ID = 2919]
[Question ID = 714]
1. [Option ID = 2856]
2. [Option ID = 2853]
3. [Option ID = 2855]
4. [Option ID = 2854]
Correct Answer :-
[Option ID = 2854]
[Question ID = 669]
1. 2 [Option ID = 2673]
2. 18 [Option ID = 2676]
3. 5 [Option ID = 2674]
4. 10 [Option ID = 2675]
Correct Answer :-
5 [Option ID = 2674]
[Question ID = 689]
1. 2 [Option ID = 2754]
2. 0 [Option ID = 2756]
3. 1 [Option ID = 2755]
37)
38)
4. 3 [Option ID = 2753]
Correct Answer :-
1 [Option ID = 2755]
[Question ID = 718]
1. C) [Option ID = 2871]
2. B) [Option ID = 2870]
3. A) [Option ID = 2869]
4. D) both (B) and (C) [Option ID = 2872]
Correct Answer :-
A) [Option ID = 2869]
[Question ID = 709]
1. [Option ID = 2836]
2. [Option ID = 2833]
3. [Option ID = 2835]
4. [Option ID = 2834]
39)
40)
41)
42)
Correct Answer :-
[Option ID = 2836]
[Question ID = 705]
1. (2Z, 4Z)-3-chlorohexa-2, 4-diene-1,6-diol. [Option ID = 2819]
2. (2E, 4E)-3-chlorohexa-2, 4-diene-1,6-diol. [Option ID = 2817]
3. (2Z, 4E)-3-chlorohexa-2, 4-diene-1,6-diol. [Option ID = 2818]
4. (2Z, 4E)-3-chlorohexa-2, 4-diene-1,6-diol. [Option ID = 2820]
Correct Answer :-
(2Z, 4E)-3-chlorohexa-2, 4-diene-1,6-diol. [Option ID = 2820]
[Question ID = 680]
1. The presence of other complexing ligands in solution affects the equilibrium concentration of metal-EDTA complexes [Option ID = 2719]
2. Metal-EDTA complexes have an equilibrium concentration independent of pH [Option ID = 2718]
3. Metal-EDTA complexes are often 2:1 in stoichiometry [Option ID = 2717]
4. Metal-EDTA complexes are less stable than the corresponding metal-ammine complexes [Option ID = 2720]
Correct Answer :-
The presence of other complexing ligands in solution affects the equilibrium concentration of metal-EDTA complexes [Option ID = 2719]
[Question ID = 671]
1. 150 g [Option ID = 2682]
2. 300 g [Option ID = 2684]
3. 120 g [Option ID = 2681]
4. 180 g [Option ID = 2683]
Correct Answer :-
300 g [Option ID = 2684]
[Question ID = 756]
1. [Option ID = 3021]
2. [Option ID = 3022]
3. [Option ID = 3024]
4. [Option ID = 3023]
Correct Answer :-
43)
44)
45)
[Option ID = 3024]
[Question ID = 750]
1. [Option ID = 2999]
2. [Option ID = 3000]
3. [Option ID = 2997]
4. [Option ID = 2998]
Correct Answer :-
[Option ID = 2997]
[Question ID = 715]
1. [Option ID = 2857]
2. [Option ID = 2860]
3. [Option ID = 2858]
4. [Option ID = 2859]
Correct Answer :-
[Option ID = 2860]
46)
47)
[Question ID = 719]
1. [Option ID = 2873]
2. [Option ID = 2876]
3. [Option ID = 2874]
4. [Option ID = 2875]
Correct Answer :-
[Question ID = 674]
1. Evaporating some water to decrease the volume of the solution. [Option ID = 2696]
2. [Option ID = 2695]
3. [Option ID = 2694]
4. [Option ID = 2693]
Correct Answer :-
[Option ID = 2693]
[Question ID = 728]
1. [Option ID = 2910]
2. [Option ID = 2911]
3. [Option ID = 2909]
4. [Option ID = 2912]
48)
49)
50)
51)
Correct Answer :-
[Option ID = 2912]
[Question ID = 697]
1. III only [Option ID = 2788]
2. II only [Option ID = 2787]
3. I only [Option ID = 2785]
4. I, II and III [Option ID = 2786]
Correct Answer :-
I, II and III [Option ID = 2786]
[Question ID = 700]
1. [Option ID = 2798]
2. [Option ID = 2797]
3. [Option ID = 2800]
4. [Option ID = 2799]
Correct Answer :-
[Option ID = 2799]
[Question ID = 678]
1. Cr
2+
has a low cationic charge [Option ID = 2711]
2. F has -1 anionic charge and highly electronegative [Option ID = 2709]
3. Spin-orbit coupling in Cr
2+
[Option ID = 2712]
4. The Jahn-Teller effect [Option ID = 2710]
Correct Answer :-
The Jahn-Teller effect [Option ID = 2710]
[Question ID = 723]
1. [Option ID = 2889]
52)
53)
2. [Option ID = 2890]
3. [Option ID = 2891]
4. [Option ID = 2892]
Correct Answer :-
[Option ID = 2889]
[Question ID = 727]
1. [Option ID = 2907]
2. [Option ID = 2906]
3. [Option ID = 2905]
4. [Option ID = 2908]
Correct Answer :-
[Option ID = 2905]
[Question ID = 707]
54)
55)
56)
1. N9 [Option ID = 2828]
2. N7 [Option ID = 2827]
3. N1 [Option ID = 2825]
4. N3 [Option ID = 2826]
Correct Answer :-
N7 [Option ID = 2827]
[Question ID = 672]
1. Bicapped prism [Option ID = 2688]
2. Square pyramidal [Option ID = 2686]
3. Trigonal planar [Option ID = 2685]
4. Bent [Option ID = 2687]
Correct Answer :-
Square pyramidal [Option ID = 2686]
[Question ID = 704]
1. [Option ID = 2816]
2. [Option ID = 2814]
3. [Option ID = 2813]
4. [Option ID = 2815]
Correct Answer :-
[Option ID = 2813]
57)
58)
[Question ID = 708]
1. [Option ID = 2829]
2. [Option ID = 2832]
3. [Option ID = 2831]
4. [Option ID = 2830]
Correct Answer :-
[Option ID = 2831]
[Question ID = 717]
1. [Option ID = 2866]
2. [Option ID = 2865]
3. [Option ID = 2867]
4. [Option ID = 2868]
Correct Answer :-
[Option ID = 2866]
59)
[Question ID = 711]
1. [Option ID = 2843]
2. [Option ID = 2842]
3. [Option ID = 2844]
4. [Option ID = 2841]
Correct Answer :-
[Option ID = 2842]
[Question ID = 710]
1. [Option ID = 2838]
2. [Option ID = 2839]
3. [Option ID = 2837]
4. [Option ID = 2840]
Correct Answer :-
60)
61)
62)
[Option ID = 2838]
[Question ID = 682]
1. 0.30 M [Option ID = 2728]
2. 0.25 M [Option ID = 2727]
3. 0.60 M [Option ID = 2725]
4. 0.10 M [Option ID = 2726]
Correct Answer :-
0.30 M [Option ID = 2728]
[Question ID = 772]
1. [Option ID = 3087]
2. [Option ID = 3088]
3. [Option ID = 3086]
4. [Option ID = 3085]
Correct Answer :-
[Option ID = 3086]
[Question ID = 733]
1. [Option ID = 2932]
FirstRanker.com - FirstRanker's Choice
1)
2)
3)
4)
5)
DU PhD in Chemistry
Topic:- DU_J18_PHD_CHEM
Which of the following statements about sulfur dioxide is true?
[Question ID = 677]
1. It forms a S-S dimer in condensed phase [Option ID = 2707]
2. Its anhydride of sulfuric acid [Option ID = 2706]
3. Its O-S-O angle is 180
0
[Option ID = 2708]
4. It is a product of the combustion of fossil fuels that contain sulfur [Option ID = 2705]
Correct Answer :-
It is a product of the combustion of fossil fuels that contain sulfur [Option ID = 2705]
Which of the following is a strong acid in pure liquid HF
[Question ID = 683]
1. H
2
O [Option ID = 2731]
2. NaF [Option ID = 2729]
3. CH
3
COOH [Option ID = 2730]
4. SbF
5
[Option ID = 2732]
Correct Answer :-
SbF
5
[Option ID = 2732]
Each of the following molecules can act as a chelating agent EXCEPT [Question ID = 679]
1. [Option ID = 2716]
2. [Option ID = 2714]
3. [Option ID = 2713]
4. [Option ID = 2715]
Correct Answer :-
[Option ID = 2714]
What is correct about h-index?
[Question ID = 758]
1. Alternative of impact factor [Option ID = 3031]
2. Based on most quoted papers [Option ID = 3030]
3. Quantify scientific productivity [Option ID = 3029]
4. All of these [Option ID = 3032]
Correct Answer :-
All of these [Option ID = 3032]
The hyperfine electron spin resonance (e.s.r.) spectrum of the benzene radical has how many lines? [Question ID = 748]
1. 12 [Option ID = 2992]
2. 7 [Option ID = 2990]
3. 1 [Option ID = 2991]
4. 6 [Option ID = 2989]
Correct Answer :-
7 [Option ID = 2990]
6)
7)
8)
9)
10)
11)
12)
The energy changes involving the core electrons of an atom or molecule are expressed in which region of the electromagnetic
spectrum? [Question ID = 742]
1. Ultraviolet and Visible region [Option ID = 2967]
2. X-ray region [Option ID = 2968]
3. Radiofrequency region [Option ID = 2966]
4. Infra-red region [Option ID = 2965]
Correct Answer :-
X-ray region [Option ID = 2968]
Find out the expected intensity ratio of M and M+1 signal for the Naphthalene molecular ion [Question ID = 726]
1. 99:1.1 [Option ID = 2903]
2. 1.1:99 [Option ID = 2904]
3. 9:01 [Option ID = 2901]
4. 1:9 [Option ID = 2902]
Correct Answer :-
9:01 [Option ID = 2901]
Cobalt-60 is used in radiation therapy of cancer and can be produced by the bombardment of Cobalt-59 with [Question ID = 692]
1. Alpha particles [Option ID = 2765]
2. Beta particles [Option ID = 2767]
3. Neutrons [Option ID = 2766]
4. Gamma rays [Option ID = 2768]
Correct Answer :-
Neutrons [Option ID = 2766]
The standard emf of galvanic cell involving 3 moles of electrons in its redox reaction is 0.59 V. The equilibrium constant for the
reaction of the cell is- [Question ID = 763]
1. [Option ID = 3051]
2. [Option ID = 3052]
3. [Option ID = 3049]
4. [Option ID = 3050]
Correct Answer :-
[Option ID = 3052]
A characteristic common to polymers that can be made to conduct electricity such as polyacetylene, polypyrrole is: [Question ID =
685]
1. Conjugation throughout the polymeric chain. [Option ID = 2740]
2. A high degree of cross linking [Option ID = 2738]
3. A very low glass transition temperature [Option ID = 2737]
4. Presence of stereogenic centers of the same configuration [Option ID = 2739]
Correct Answer :-
Conjugation throughout the polymeric chain. [Option ID = 2740]
Impact factor is [Question ID = 768]
1. Ratio between citations and recent citable items publish [Option ID = 3071]
2. All of these [Option ID = 3072]
3. Addition of citations and recent citable items publish [Option ID = 3069]
4. Ratio between recent citable items publish and citations [Option ID = 3070]
Correct Answer :-
Ratio between recent citable items publish and citations [Option ID = 3070]
On the basis of oxidation-reduction potential, which of the following is most likely to occur? [Question ID = 693]
13)
14)
15)
16)
17)
1. [Option ID = 2770]
2. [Option ID = 2772]
3. [Option ID = 2771]
4. [Option ID = 2769]
Correct Answer :-
[Option ID = 2769]
How many diastereoisomers are possible for the compound 2, 4 ?diphenylcyclobutane-1, 3 di carboxylic acids. [Question ID = 725]
1. 6 [Option ID = 2899]
2. 5 [Option ID = 2898]
3. 8 [Option ID = 2900]
4. 4 [Option ID = 2897]
Correct Answer :-
5 [Option ID = 2898]
An increase in equivalent conductance of a strong electrolyte with dilution is mainly due to- [Question ID = 764]
1. increase in ionic mobility of ions [Option ID = 3055]
2. increase in number of ions [Option ID = 3054]
3. 100% ionization of electrolyte at normal dilution [Option ID = 3056]
4. increase in both i.e. number of ions and ionic mobility of ions. [Option ID = 3053]
Correct Answer :-
increase in ionic mobility of ions [Option ID = 3055]
The solid state structures of the principal allotropes of elemental boron are made up of which of the following structural units
[Question ID = 699]
1. [Option ID = 2796]
2. [Option ID = 2795]
3. [Option ID = 2794]
4. [Option ID = 2793]
Correct Answer :-
[Option ID = 2793]
The molecular geometry of thionyl chloride is best described as [Question ID = 688]
1. T-shaped [Option ID = 2752]
2. Tetrahedral [Option ID = 2751]
3. Trigonal pyramidal [Option ID = 2749]
4. Trigonal planar [Option ID = 2750]
Correct Answer :-
Trigonal pyramidal [Option ID = 2749]
In a face-center cubic (FCC) type of crystal lattice, the number of atoms belonging exclusively to each unit cell within the lattice
is/are: [Question ID = 754]
1. 2 [Option ID = 3014]
2. 1 [Option ID = 3013]
3. 3 [Option ID = 3015]
4. 4 [Option ID = 3016]
Correct Answer :-
4 [Option ID = 3016]
18)
19)
20)
21)
22)
23)
24)
Among the following, the weakest oxidizing agent is [Question ID = 675]
1. Mg (s) [Option ID = 2698]
2. [Option ID = 2699]
3. [Option ID = 2700]
4. [Option ID = 2697]
Correct Answer :-
Mg (s) [Option ID = 2698]
For a polymer, which of the following statement/s is/are true? [Question ID = 759]
1. Weight average molecular weight is almost always higher than the number average molecular weight [Option ID = 3035]
2. Formation of a polypeptide from its monomers (amino acids) is an example of addition polymerization [Option ID = 3034]
3. All of these [Option ID = 3036]
4. Vinyl polymerization is an example of condensation polymerization. [Option ID = 3033]
Correct Answer :-
Weight average molecular weight is almost always higher than the number average molecular weight [Option ID = 3035]
Quantum dots are [Question ID = 762]
1. Three dimensional [Option ID = 3048]
2. One dimensional [Option ID = 3046]
3. Two dimensional [Option ID = 3047]
4. Zero dimensional [Option ID = 3045]
Correct Answer :-
Zero dimensional [Option ID = 3045]
The unit of rate constant for a third order reaction is: [Question ID = 749]
1. [Option ID = 2993]
2. [Option ID = 2995]
3. [Option ID = 2996]
4. [Option ID = 2994]
Correct Answer :-
[Option ID = 2996]
All the following elements have at least one isotope that is not radioactive EXCEPT [Question ID = 673]
1. Pb [Option ID = 2690]
2. O [Option ID = 2689]
3. Sn [Option ID = 2691]
4. No [Option ID = 2692]
Correct Answer :-
No [Option ID = 2692]
The conditions for a species to follow Bose-Einstein statistics are; [Question ID = 736]
1. Particles are indistinguishable, with no restriction on filling up of energy levels [Option ID = 2944]
2. Particles are indistinguishable, with a restriction on filling up of energy levels [Option ID = 2943]
3. Particles are distinguishable, with a restriction on filling up of energy levels [Option ID = 2941]
4. Particles are distinguishable, with no restriction on filling up of energy levels [Option ID = 2942]
Correct Answer :-
Particles are indistinguishable, with no restriction on filling up of energy levels [Option ID = 2944]
In the kinetic theory of collisions, the SI unit of collision number, in terms of m (meter) and s (second), is:
[Question ID = 761]
25)
26)
27)
28)
29)
1. m-
2
s
-1
[Option ID = 3042]
2. m
4
s
-1
[Option ID = 3041]
3. m
2
s
-1
[Option ID = 3043]
4. None of these [Option ID = 3044]
Correct Answer :-
None of these [Option ID = 3044]
Correct characteristics of the functional groups of adenine in DNA base pair are [Question ID = 706]
1. [Option ID = 2824]
2. [Option ID = 2821]
3. [Option ID = 2823]
4. [Option ID = 2822]
Correct Answer :-
[Option ID = 2822]
The carbon monoxide molecule has an internuclear distance of 1.13 Angstroms. What is the moment of Inertia of this molecule?
[Question ID = 740]
1. [Option ID = 2960]
2. [Option ID = 2957]
3. [Option ID = 2958]
4. [Option ID = 2959]
Correct Answer :-
[Option ID = 2957]
Which of the following represent/s non-linear optical technique? [Question ID = 744]
1. Second Harmonic generation [Option ID = 2974]
2. Two-photon photoluminescence [Option ID = 2975]
3. Four-wave mixing [Option ID = 2973]
4. All of these [Option ID = 2976]
Correct Answer :-
All of these [Option ID = 2976]
. Which of the following does not affect the intensity of spectral lines of a sample? [Question ID = 743]
1. Path length of a sample [Option ID = 2972]
2. Population of energy states [Option ID = 2970]
3. Heisenberg?s Uncertainty principle [Option ID = 2971]
4. Concentration of a sample [Option ID = 2969]
Correct Answer :-
Heisenberg?s Uncertainty principle [Option ID = 2971]
[Question ID = 716]
30)
31)
1. [Option ID = 2862]
2. [Option ID = 2863]
3. [Option ID = 2861]
4. [Option ID = 2864]
Correct Answer :-
[Option ID = 2864]
[Question ID = 712]
1. [Option ID = 2845]
2. [Option ID = 2846]
3. [Option ID = 2848]
4. [Option ID = 2847]
Correct Answer :-
[Option ID = 2847]
[Question ID = 686]
1. [Option ID = 2743]
2. [Option ID = 2742]
3. [Option ID = 2741]
32)
33)
4. [Option ID = 2744]
Correct Answer :-
[Option ID = 2743]
[Question ID = 5633]
1. [Option ID = 22523]
2. [Option ID = 22524]
3. [Option ID = 22526]
4. [Option ID = 22525]
Correct Answer :-
[Option ID = 22523]
[Question ID = 730]
1. [Option ID = 2918]
2. [Option ID = 2920]
3. [Option ID = 2917]
4. [Option ID = 2919]
Correct Answer :-
34)
35)
36)
[Option ID = 2919]
[Question ID = 714]
1. [Option ID = 2856]
2. [Option ID = 2853]
3. [Option ID = 2855]
4. [Option ID = 2854]
Correct Answer :-
[Option ID = 2854]
[Question ID = 669]
1. 2 [Option ID = 2673]
2. 18 [Option ID = 2676]
3. 5 [Option ID = 2674]
4. 10 [Option ID = 2675]
Correct Answer :-
5 [Option ID = 2674]
[Question ID = 689]
1. 2 [Option ID = 2754]
2. 0 [Option ID = 2756]
3. 1 [Option ID = 2755]
37)
38)
4. 3 [Option ID = 2753]
Correct Answer :-
1 [Option ID = 2755]
[Question ID = 718]
1. C) [Option ID = 2871]
2. B) [Option ID = 2870]
3. A) [Option ID = 2869]
4. D) both (B) and (C) [Option ID = 2872]
Correct Answer :-
A) [Option ID = 2869]
[Question ID = 709]
1. [Option ID = 2836]
2. [Option ID = 2833]
3. [Option ID = 2835]
4. [Option ID = 2834]
39)
40)
41)
42)
Correct Answer :-
[Option ID = 2836]
[Question ID = 705]
1. (2Z, 4Z)-3-chlorohexa-2, 4-diene-1,6-diol. [Option ID = 2819]
2. (2E, 4E)-3-chlorohexa-2, 4-diene-1,6-diol. [Option ID = 2817]
3. (2Z, 4E)-3-chlorohexa-2, 4-diene-1,6-diol. [Option ID = 2818]
4. (2Z, 4E)-3-chlorohexa-2, 4-diene-1,6-diol. [Option ID = 2820]
Correct Answer :-
(2Z, 4E)-3-chlorohexa-2, 4-diene-1,6-diol. [Option ID = 2820]
[Question ID = 680]
1. The presence of other complexing ligands in solution affects the equilibrium concentration of metal-EDTA complexes [Option ID = 2719]
2. Metal-EDTA complexes have an equilibrium concentration independent of pH [Option ID = 2718]
3. Metal-EDTA complexes are often 2:1 in stoichiometry [Option ID = 2717]
4. Metal-EDTA complexes are less stable than the corresponding metal-ammine complexes [Option ID = 2720]
Correct Answer :-
The presence of other complexing ligands in solution affects the equilibrium concentration of metal-EDTA complexes [Option ID = 2719]
[Question ID = 671]
1. 150 g [Option ID = 2682]
2. 300 g [Option ID = 2684]
3. 120 g [Option ID = 2681]
4. 180 g [Option ID = 2683]
Correct Answer :-
300 g [Option ID = 2684]
[Question ID = 756]
1. [Option ID = 3021]
2. [Option ID = 3022]
3. [Option ID = 3024]
4. [Option ID = 3023]
Correct Answer :-
43)
44)
45)
[Option ID = 3024]
[Question ID = 750]
1. [Option ID = 2999]
2. [Option ID = 3000]
3. [Option ID = 2997]
4. [Option ID = 2998]
Correct Answer :-
[Option ID = 2997]
[Question ID = 715]
1. [Option ID = 2857]
2. [Option ID = 2860]
3. [Option ID = 2858]
4. [Option ID = 2859]
Correct Answer :-
[Option ID = 2860]
46)
47)
[Question ID = 719]
1. [Option ID = 2873]
2. [Option ID = 2876]
3. [Option ID = 2874]
4. [Option ID = 2875]
Correct Answer :-
[Question ID = 674]
1. Evaporating some water to decrease the volume of the solution. [Option ID = 2696]
2. [Option ID = 2695]
3. [Option ID = 2694]
4. [Option ID = 2693]
Correct Answer :-
[Option ID = 2693]
[Question ID = 728]
1. [Option ID = 2910]
2. [Option ID = 2911]
3. [Option ID = 2909]
4. [Option ID = 2912]
48)
49)
50)
51)
Correct Answer :-
[Option ID = 2912]
[Question ID = 697]
1. III only [Option ID = 2788]
2. II only [Option ID = 2787]
3. I only [Option ID = 2785]
4. I, II and III [Option ID = 2786]
Correct Answer :-
I, II and III [Option ID = 2786]
[Question ID = 700]
1. [Option ID = 2798]
2. [Option ID = 2797]
3. [Option ID = 2800]
4. [Option ID = 2799]
Correct Answer :-
[Option ID = 2799]
[Question ID = 678]
1. Cr
2+
has a low cationic charge [Option ID = 2711]
2. F has -1 anionic charge and highly electronegative [Option ID = 2709]
3. Spin-orbit coupling in Cr
2+
[Option ID = 2712]
4. The Jahn-Teller effect [Option ID = 2710]
Correct Answer :-
The Jahn-Teller effect [Option ID = 2710]
[Question ID = 723]
1. [Option ID = 2889]
52)
53)
2. [Option ID = 2890]
3. [Option ID = 2891]
4. [Option ID = 2892]
Correct Answer :-
[Option ID = 2889]
[Question ID = 727]
1. [Option ID = 2907]
2. [Option ID = 2906]
3. [Option ID = 2905]
4. [Option ID = 2908]
Correct Answer :-
[Option ID = 2905]
[Question ID = 707]
54)
55)
56)
1. N9 [Option ID = 2828]
2. N7 [Option ID = 2827]
3. N1 [Option ID = 2825]
4. N3 [Option ID = 2826]
Correct Answer :-
N7 [Option ID = 2827]
[Question ID = 672]
1. Bicapped prism [Option ID = 2688]
2. Square pyramidal [Option ID = 2686]
3. Trigonal planar [Option ID = 2685]
4. Bent [Option ID = 2687]
Correct Answer :-
Square pyramidal [Option ID = 2686]
[Question ID = 704]
1. [Option ID = 2816]
2. [Option ID = 2814]
3. [Option ID = 2813]
4. [Option ID = 2815]
Correct Answer :-
[Option ID = 2813]
57)
58)
[Question ID = 708]
1. [Option ID = 2829]
2. [Option ID = 2832]
3. [Option ID = 2831]
4. [Option ID = 2830]
Correct Answer :-
[Option ID = 2831]
[Question ID = 717]
1. [Option ID = 2866]
2. [Option ID = 2865]
3. [Option ID = 2867]
4. [Option ID = 2868]
Correct Answer :-
[Option ID = 2866]
59)
[Question ID = 711]
1. [Option ID = 2843]
2. [Option ID = 2842]
3. [Option ID = 2844]
4. [Option ID = 2841]
Correct Answer :-
[Option ID = 2842]
[Question ID = 710]
1. [Option ID = 2838]
2. [Option ID = 2839]
3. [Option ID = 2837]
4. [Option ID = 2840]
Correct Answer :-
60)
61)
62)
[Option ID = 2838]
[Question ID = 682]
1. 0.30 M [Option ID = 2728]
2. 0.25 M [Option ID = 2727]
3. 0.60 M [Option ID = 2725]
4. 0.10 M [Option ID = 2726]
Correct Answer :-
0.30 M [Option ID = 2728]
[Question ID = 772]
1. [Option ID = 3087]
2. [Option ID = 3088]
3. [Option ID = 3086]
4. [Option ID = 3085]
Correct Answer :-
[Option ID = 3086]
[Question ID = 733]
1. [Option ID = 2932]
63)
64)
65)
2. [Option ID = 2929]
3. [Option ID = 2930]
4. [Option ID = 2931]
Correct Answer :-
[Option ID = 2929]
[Question ID = 767]
1. 120 [Option ID = 3065]
2. 280 [Option ID = 3066]
3. 300 [Option ID = 3068]
4. 20 [Option ID = 3067]
Correct Answer :-
20 [Option ID = 3067]
[Question ID = 734]
1. [Option ID = 2936]
2. [Option ID = 2933]
3. [Option ID = 2934]
4. [Option ID = 2935]
Correct Answer :-
[Option ID = 2935]
[Question ID = 676]
FirstRanker.com - FirstRanker's Choice
1)
2)
3)
4)
5)
DU PhD in Chemistry
Topic:- DU_J18_PHD_CHEM
Which of the following statements about sulfur dioxide is true?
[Question ID = 677]
1. It forms a S-S dimer in condensed phase [Option ID = 2707]
2. Its anhydride of sulfuric acid [Option ID = 2706]
3. Its O-S-O angle is 180
0
[Option ID = 2708]
4. It is a product of the combustion of fossil fuels that contain sulfur [Option ID = 2705]
Correct Answer :-
It is a product of the combustion of fossil fuels that contain sulfur [Option ID = 2705]
Which of the following is a strong acid in pure liquid HF
[Question ID = 683]
1. H
2
O [Option ID = 2731]
2. NaF [Option ID = 2729]
3. CH
3
COOH [Option ID = 2730]
4. SbF
5
[Option ID = 2732]
Correct Answer :-
SbF
5
[Option ID = 2732]
Each of the following molecules can act as a chelating agent EXCEPT [Question ID = 679]
1. [Option ID = 2716]
2. [Option ID = 2714]
3. [Option ID = 2713]
4. [Option ID = 2715]
Correct Answer :-
[Option ID = 2714]
What is correct about h-index?
[Question ID = 758]
1. Alternative of impact factor [Option ID = 3031]
2. Based on most quoted papers [Option ID = 3030]
3. Quantify scientific productivity [Option ID = 3029]
4. All of these [Option ID = 3032]
Correct Answer :-
All of these [Option ID = 3032]
The hyperfine electron spin resonance (e.s.r.) spectrum of the benzene radical has how many lines? [Question ID = 748]
1. 12 [Option ID = 2992]
2. 7 [Option ID = 2990]
3. 1 [Option ID = 2991]
4. 6 [Option ID = 2989]
Correct Answer :-
7 [Option ID = 2990]
6)
7)
8)
9)
10)
11)
12)
The energy changes involving the core electrons of an atom or molecule are expressed in which region of the electromagnetic
spectrum? [Question ID = 742]
1. Ultraviolet and Visible region [Option ID = 2967]
2. X-ray region [Option ID = 2968]
3. Radiofrequency region [Option ID = 2966]
4. Infra-red region [Option ID = 2965]
Correct Answer :-
X-ray region [Option ID = 2968]
Find out the expected intensity ratio of M and M+1 signal for the Naphthalene molecular ion [Question ID = 726]
1. 99:1.1 [Option ID = 2903]
2. 1.1:99 [Option ID = 2904]
3. 9:01 [Option ID = 2901]
4. 1:9 [Option ID = 2902]
Correct Answer :-
9:01 [Option ID = 2901]
Cobalt-60 is used in radiation therapy of cancer and can be produced by the bombardment of Cobalt-59 with [Question ID = 692]
1. Alpha particles [Option ID = 2765]
2. Beta particles [Option ID = 2767]
3. Neutrons [Option ID = 2766]
4. Gamma rays [Option ID = 2768]
Correct Answer :-
Neutrons [Option ID = 2766]
The standard emf of galvanic cell involving 3 moles of electrons in its redox reaction is 0.59 V. The equilibrium constant for the
reaction of the cell is- [Question ID = 763]
1. [Option ID = 3051]
2. [Option ID = 3052]
3. [Option ID = 3049]
4. [Option ID = 3050]
Correct Answer :-
[Option ID = 3052]
A characteristic common to polymers that can be made to conduct electricity such as polyacetylene, polypyrrole is: [Question ID =
685]
1. Conjugation throughout the polymeric chain. [Option ID = 2740]
2. A high degree of cross linking [Option ID = 2738]
3. A very low glass transition temperature [Option ID = 2737]
4. Presence of stereogenic centers of the same configuration [Option ID = 2739]
Correct Answer :-
Conjugation throughout the polymeric chain. [Option ID = 2740]
Impact factor is [Question ID = 768]
1. Ratio between citations and recent citable items publish [Option ID = 3071]
2. All of these [Option ID = 3072]
3. Addition of citations and recent citable items publish [Option ID = 3069]
4. Ratio between recent citable items publish and citations [Option ID = 3070]
Correct Answer :-
Ratio between recent citable items publish and citations [Option ID = 3070]
On the basis of oxidation-reduction potential, which of the following is most likely to occur? [Question ID = 693]
13)
14)
15)
16)
17)
1. [Option ID = 2770]
2. [Option ID = 2772]
3. [Option ID = 2771]
4. [Option ID = 2769]
Correct Answer :-
[Option ID = 2769]
How many diastereoisomers are possible for the compound 2, 4 ?diphenylcyclobutane-1, 3 di carboxylic acids. [Question ID = 725]
1. 6 [Option ID = 2899]
2. 5 [Option ID = 2898]
3. 8 [Option ID = 2900]
4. 4 [Option ID = 2897]
Correct Answer :-
5 [Option ID = 2898]
An increase in equivalent conductance of a strong electrolyte with dilution is mainly due to- [Question ID = 764]
1. increase in ionic mobility of ions [Option ID = 3055]
2. increase in number of ions [Option ID = 3054]
3. 100% ionization of electrolyte at normal dilution [Option ID = 3056]
4. increase in both i.e. number of ions and ionic mobility of ions. [Option ID = 3053]
Correct Answer :-
increase in ionic mobility of ions [Option ID = 3055]
The solid state structures of the principal allotropes of elemental boron are made up of which of the following structural units
[Question ID = 699]
1. [Option ID = 2796]
2. [Option ID = 2795]
3. [Option ID = 2794]
4. [Option ID = 2793]
Correct Answer :-
[Option ID = 2793]
The molecular geometry of thionyl chloride is best described as [Question ID = 688]
1. T-shaped [Option ID = 2752]
2. Tetrahedral [Option ID = 2751]
3. Trigonal pyramidal [Option ID = 2749]
4. Trigonal planar [Option ID = 2750]
Correct Answer :-
Trigonal pyramidal [Option ID = 2749]
In a face-center cubic (FCC) type of crystal lattice, the number of atoms belonging exclusively to each unit cell within the lattice
is/are: [Question ID = 754]
1. 2 [Option ID = 3014]
2. 1 [Option ID = 3013]
3. 3 [Option ID = 3015]
4. 4 [Option ID = 3016]
Correct Answer :-
4 [Option ID = 3016]
18)
19)
20)
21)
22)
23)
24)
Among the following, the weakest oxidizing agent is [Question ID = 675]
1. Mg (s) [Option ID = 2698]
2. [Option ID = 2699]
3. [Option ID = 2700]
4. [Option ID = 2697]
Correct Answer :-
Mg (s) [Option ID = 2698]
For a polymer, which of the following statement/s is/are true? [Question ID = 759]
1. Weight average molecular weight is almost always higher than the number average molecular weight [Option ID = 3035]
2. Formation of a polypeptide from its monomers (amino acids) is an example of addition polymerization [Option ID = 3034]
3. All of these [Option ID = 3036]
4. Vinyl polymerization is an example of condensation polymerization. [Option ID = 3033]
Correct Answer :-
Weight average molecular weight is almost always higher than the number average molecular weight [Option ID = 3035]
Quantum dots are [Question ID = 762]
1. Three dimensional [Option ID = 3048]
2. One dimensional [Option ID = 3046]
3. Two dimensional [Option ID = 3047]
4. Zero dimensional [Option ID = 3045]
Correct Answer :-
Zero dimensional [Option ID = 3045]
The unit of rate constant for a third order reaction is: [Question ID = 749]
1. [Option ID = 2993]
2. [Option ID = 2995]
3. [Option ID = 2996]
4. [Option ID = 2994]
Correct Answer :-
[Option ID = 2996]
All the following elements have at least one isotope that is not radioactive EXCEPT [Question ID = 673]
1. Pb [Option ID = 2690]
2. O [Option ID = 2689]
3. Sn [Option ID = 2691]
4. No [Option ID = 2692]
Correct Answer :-
No [Option ID = 2692]
The conditions for a species to follow Bose-Einstein statistics are; [Question ID = 736]
1. Particles are indistinguishable, with no restriction on filling up of energy levels [Option ID = 2944]
2. Particles are indistinguishable, with a restriction on filling up of energy levels [Option ID = 2943]
3. Particles are distinguishable, with a restriction on filling up of energy levels [Option ID = 2941]
4. Particles are distinguishable, with no restriction on filling up of energy levels [Option ID = 2942]
Correct Answer :-
Particles are indistinguishable, with no restriction on filling up of energy levels [Option ID = 2944]
In the kinetic theory of collisions, the SI unit of collision number, in terms of m (meter) and s (second), is:
[Question ID = 761]
25)
26)
27)
28)
29)
1. m-
2
s
-1
[Option ID = 3042]
2. m
4
s
-1
[Option ID = 3041]
3. m
2
s
-1
[Option ID = 3043]
4. None of these [Option ID = 3044]
Correct Answer :-
None of these [Option ID = 3044]
Correct characteristics of the functional groups of adenine in DNA base pair are [Question ID = 706]
1. [Option ID = 2824]
2. [Option ID = 2821]
3. [Option ID = 2823]
4. [Option ID = 2822]
Correct Answer :-
[Option ID = 2822]
The carbon monoxide molecule has an internuclear distance of 1.13 Angstroms. What is the moment of Inertia of this molecule?
[Question ID = 740]
1. [Option ID = 2960]
2. [Option ID = 2957]
3. [Option ID = 2958]
4. [Option ID = 2959]
Correct Answer :-
[Option ID = 2957]
Which of the following represent/s non-linear optical technique? [Question ID = 744]
1. Second Harmonic generation [Option ID = 2974]
2. Two-photon photoluminescence [Option ID = 2975]
3. Four-wave mixing [Option ID = 2973]
4. All of these [Option ID = 2976]
Correct Answer :-
All of these [Option ID = 2976]
. Which of the following does not affect the intensity of spectral lines of a sample? [Question ID = 743]
1. Path length of a sample [Option ID = 2972]
2. Population of energy states [Option ID = 2970]
3. Heisenberg?s Uncertainty principle [Option ID = 2971]
4. Concentration of a sample [Option ID = 2969]
Correct Answer :-
Heisenberg?s Uncertainty principle [Option ID = 2971]
[Question ID = 716]
30)
31)
1. [Option ID = 2862]
2. [Option ID = 2863]
3. [Option ID = 2861]
4. [Option ID = 2864]
Correct Answer :-
[Option ID = 2864]
[Question ID = 712]
1. [Option ID = 2845]
2. [Option ID = 2846]
3. [Option ID = 2848]
4. [Option ID = 2847]
Correct Answer :-
[Option ID = 2847]
[Question ID = 686]
1. [Option ID = 2743]
2. [Option ID = 2742]
3. [Option ID = 2741]
32)
33)
4. [Option ID = 2744]
Correct Answer :-
[Option ID = 2743]
[Question ID = 5633]
1. [Option ID = 22523]
2. [Option ID = 22524]
3. [Option ID = 22526]
4. [Option ID = 22525]
Correct Answer :-
[Option ID = 22523]
[Question ID = 730]
1. [Option ID = 2918]
2. [Option ID = 2920]
3. [Option ID = 2917]
4. [Option ID = 2919]
Correct Answer :-
34)
35)
36)
[Option ID = 2919]
[Question ID = 714]
1. [Option ID = 2856]
2. [Option ID = 2853]
3. [Option ID = 2855]
4. [Option ID = 2854]
Correct Answer :-
[Option ID = 2854]
[Question ID = 669]
1. 2 [Option ID = 2673]
2. 18 [Option ID = 2676]
3. 5 [Option ID = 2674]
4. 10 [Option ID = 2675]
Correct Answer :-
5 [Option ID = 2674]
[Question ID = 689]
1. 2 [Option ID = 2754]
2. 0 [Option ID = 2756]
3. 1 [Option ID = 2755]
37)
38)
4. 3 [Option ID = 2753]
Correct Answer :-
1 [Option ID = 2755]
[Question ID = 718]
1. C) [Option ID = 2871]
2. B) [Option ID = 2870]
3. A) [Option ID = 2869]
4. D) both (B) and (C) [Option ID = 2872]
Correct Answer :-
A) [Option ID = 2869]
[Question ID = 709]
1. [Option ID = 2836]
2. [Option ID = 2833]
3. [Option ID = 2835]
4. [Option ID = 2834]
39)
40)
41)
42)
Correct Answer :-
[Option ID = 2836]
[Question ID = 705]
1. (2Z, 4Z)-3-chlorohexa-2, 4-diene-1,6-diol. [Option ID = 2819]
2. (2E, 4E)-3-chlorohexa-2, 4-diene-1,6-diol. [Option ID = 2817]
3. (2Z, 4E)-3-chlorohexa-2, 4-diene-1,6-diol. [Option ID = 2818]
4. (2Z, 4E)-3-chlorohexa-2, 4-diene-1,6-diol. [Option ID = 2820]
Correct Answer :-
(2Z, 4E)-3-chlorohexa-2, 4-diene-1,6-diol. [Option ID = 2820]
[Question ID = 680]
1. The presence of other complexing ligands in solution affects the equilibrium concentration of metal-EDTA complexes [Option ID = 2719]
2. Metal-EDTA complexes have an equilibrium concentration independent of pH [Option ID = 2718]
3. Metal-EDTA complexes are often 2:1 in stoichiometry [Option ID = 2717]
4. Metal-EDTA complexes are less stable than the corresponding metal-ammine complexes [Option ID = 2720]
Correct Answer :-
The presence of other complexing ligands in solution affects the equilibrium concentration of metal-EDTA complexes [Option ID = 2719]
[Question ID = 671]
1. 150 g [Option ID = 2682]
2. 300 g [Option ID = 2684]
3. 120 g [Option ID = 2681]
4. 180 g [Option ID = 2683]
Correct Answer :-
300 g [Option ID = 2684]
[Question ID = 756]
1. [Option ID = 3021]
2. [Option ID = 3022]
3. [Option ID = 3024]
4. [Option ID = 3023]
Correct Answer :-
43)
44)
45)
[Option ID = 3024]
[Question ID = 750]
1. [Option ID = 2999]
2. [Option ID = 3000]
3. [Option ID = 2997]
4. [Option ID = 2998]
Correct Answer :-
[Option ID = 2997]
[Question ID = 715]
1. [Option ID = 2857]
2. [Option ID = 2860]
3. [Option ID = 2858]
4. [Option ID = 2859]
Correct Answer :-
[Option ID = 2860]
46)
47)
[Question ID = 719]
1. [Option ID = 2873]
2. [Option ID = 2876]
3. [Option ID = 2874]
4. [Option ID = 2875]
Correct Answer :-
[Question ID = 674]
1. Evaporating some water to decrease the volume of the solution. [Option ID = 2696]
2. [Option ID = 2695]
3. [Option ID = 2694]
4. [Option ID = 2693]
Correct Answer :-
[Option ID = 2693]
[Question ID = 728]
1. [Option ID = 2910]
2. [Option ID = 2911]
3. [Option ID = 2909]
4. [Option ID = 2912]
48)
49)
50)
51)
Correct Answer :-
[Option ID = 2912]
[Question ID = 697]
1. III only [Option ID = 2788]
2. II only [Option ID = 2787]
3. I only [Option ID = 2785]
4. I, II and III [Option ID = 2786]
Correct Answer :-
I, II and III [Option ID = 2786]
[Question ID = 700]
1. [Option ID = 2798]
2. [Option ID = 2797]
3. [Option ID = 2800]
4. [Option ID = 2799]
Correct Answer :-
[Option ID = 2799]
[Question ID = 678]
1. Cr
2+
has a low cationic charge [Option ID = 2711]
2. F has -1 anionic charge and highly electronegative [Option ID = 2709]
3. Spin-orbit coupling in Cr
2+
[Option ID = 2712]
4. The Jahn-Teller effect [Option ID = 2710]
Correct Answer :-
The Jahn-Teller effect [Option ID = 2710]
[Question ID = 723]
1. [Option ID = 2889]
52)
53)
2. [Option ID = 2890]
3. [Option ID = 2891]
4. [Option ID = 2892]
Correct Answer :-
[Option ID = 2889]
[Question ID = 727]
1. [Option ID = 2907]
2. [Option ID = 2906]
3. [Option ID = 2905]
4. [Option ID = 2908]
Correct Answer :-
[Option ID = 2905]
[Question ID = 707]
54)
55)
56)
1. N9 [Option ID = 2828]
2. N7 [Option ID = 2827]
3. N1 [Option ID = 2825]
4. N3 [Option ID = 2826]
Correct Answer :-
N7 [Option ID = 2827]
[Question ID = 672]
1. Bicapped prism [Option ID = 2688]
2. Square pyramidal [Option ID = 2686]
3. Trigonal planar [Option ID = 2685]
4. Bent [Option ID = 2687]
Correct Answer :-
Square pyramidal [Option ID = 2686]
[Question ID = 704]
1. [Option ID = 2816]
2. [Option ID = 2814]
3. [Option ID = 2813]
4. [Option ID = 2815]
Correct Answer :-
[Option ID = 2813]
57)
58)
[Question ID = 708]
1. [Option ID = 2829]
2. [Option ID = 2832]
3. [Option ID = 2831]
4. [Option ID = 2830]
Correct Answer :-
[Option ID = 2831]
[Question ID = 717]
1. [Option ID = 2866]
2. [Option ID = 2865]
3. [Option ID = 2867]
4. [Option ID = 2868]
Correct Answer :-
[Option ID = 2866]
59)
[Question ID = 711]
1. [Option ID = 2843]
2. [Option ID = 2842]
3. [Option ID = 2844]
4. [Option ID = 2841]
Correct Answer :-
[Option ID = 2842]
[Question ID = 710]
1. [Option ID = 2838]
2. [Option ID = 2839]
3. [Option ID = 2837]
4. [Option ID = 2840]
Correct Answer :-
60)
61)
62)
[Option ID = 2838]
[Question ID = 682]
1. 0.30 M [Option ID = 2728]
2. 0.25 M [Option ID = 2727]
3. 0.60 M [Option ID = 2725]
4. 0.10 M [Option ID = 2726]
Correct Answer :-
0.30 M [Option ID = 2728]
[Question ID = 772]
1. [Option ID = 3087]
2. [Option ID = 3088]
3. [Option ID = 3086]
4. [Option ID = 3085]
Correct Answer :-
[Option ID = 3086]
[Question ID = 733]
1. [Option ID = 2932]
63)
64)
65)
2. [Option ID = 2929]
3. [Option ID = 2930]
4. [Option ID = 2931]
Correct Answer :-
[Option ID = 2929]
[Question ID = 767]
1. 120 [Option ID = 3065]
2. 280 [Option ID = 3066]
3. 300 [Option ID = 3068]
4. 20 [Option ID = 3067]
Correct Answer :-
20 [Option ID = 3067]
[Question ID = 734]
1. [Option ID = 2936]
2. [Option ID = 2933]
3. [Option ID = 2934]
4. [Option ID = 2935]
Correct Answer :-
[Option ID = 2935]
[Question ID = 676]
66)
67)
68)
1. [Option ID = 2702]
2. [Option ID = 2703]
3. [Option ID = 2704]
4. [Option ID = 2701]
Correct Answer :-
[Option ID = 2703]
[Question ID = 729]
1. (2Z, 4Z)-3, 4-dibromo hepta-2, 4-diene [Option ID = 2914]
2. (2E, 4E)-3, 4-dibromo hepta-2, 4-diene [Option ID = 2915]
3. (2E, 4Z)-3, 4-dibromo hepta-2, 4-diene [Option ID = 2916]
4. (2Z, 4E)-3, 4-dibromo hepta-2, 4-diene [Option ID = 2913]
Correct Answer :-
(2E, 4Z)-3, 4-dibromo hepta-2, 4-diene [Option ID = 2916]
[Question ID = 765]
1. [Option ID = 3060]
2. [Option ID = 3057]
3. [Option ID = 3058]
4. [Option ID = 3059]
Correct Answer :-
[Option ID = 3059]
[Question ID = 724]
1. [Option ID = 2893]
FirstRanker.com - FirstRanker's Choice
1)
2)
3)
4)
5)
DU PhD in Chemistry
Topic:- DU_J18_PHD_CHEM
Which of the following statements about sulfur dioxide is true?
[Question ID = 677]
1. It forms a S-S dimer in condensed phase [Option ID = 2707]
2. Its anhydride of sulfuric acid [Option ID = 2706]
3. Its O-S-O angle is 180
0
[Option ID = 2708]
4. It is a product of the combustion of fossil fuels that contain sulfur [Option ID = 2705]
Correct Answer :-
It is a product of the combustion of fossil fuels that contain sulfur [Option ID = 2705]
Which of the following is a strong acid in pure liquid HF
[Question ID = 683]
1. H
2
O [Option ID = 2731]
2. NaF [Option ID = 2729]
3. CH
3
COOH [Option ID = 2730]
4. SbF
5
[Option ID = 2732]
Correct Answer :-
SbF
5
[Option ID = 2732]
Each of the following molecules can act as a chelating agent EXCEPT [Question ID = 679]
1. [Option ID = 2716]
2. [Option ID = 2714]
3. [Option ID = 2713]
4. [Option ID = 2715]
Correct Answer :-
[Option ID = 2714]
What is correct about h-index?
[Question ID = 758]
1. Alternative of impact factor [Option ID = 3031]
2. Based on most quoted papers [Option ID = 3030]
3. Quantify scientific productivity [Option ID = 3029]
4. All of these [Option ID = 3032]
Correct Answer :-
All of these [Option ID = 3032]
The hyperfine electron spin resonance (e.s.r.) spectrum of the benzene radical has how many lines? [Question ID = 748]
1. 12 [Option ID = 2992]
2. 7 [Option ID = 2990]
3. 1 [Option ID = 2991]
4. 6 [Option ID = 2989]
Correct Answer :-
7 [Option ID = 2990]
6)
7)
8)
9)
10)
11)
12)
The energy changes involving the core electrons of an atom or molecule are expressed in which region of the electromagnetic
spectrum? [Question ID = 742]
1. Ultraviolet and Visible region [Option ID = 2967]
2. X-ray region [Option ID = 2968]
3. Radiofrequency region [Option ID = 2966]
4. Infra-red region [Option ID = 2965]
Correct Answer :-
X-ray region [Option ID = 2968]
Find out the expected intensity ratio of M and M+1 signal for the Naphthalene molecular ion [Question ID = 726]
1. 99:1.1 [Option ID = 2903]
2. 1.1:99 [Option ID = 2904]
3. 9:01 [Option ID = 2901]
4. 1:9 [Option ID = 2902]
Correct Answer :-
9:01 [Option ID = 2901]
Cobalt-60 is used in radiation therapy of cancer and can be produced by the bombardment of Cobalt-59 with [Question ID = 692]
1. Alpha particles [Option ID = 2765]
2. Beta particles [Option ID = 2767]
3. Neutrons [Option ID = 2766]
4. Gamma rays [Option ID = 2768]
Correct Answer :-
Neutrons [Option ID = 2766]
The standard emf of galvanic cell involving 3 moles of electrons in its redox reaction is 0.59 V. The equilibrium constant for the
reaction of the cell is- [Question ID = 763]
1. [Option ID = 3051]
2. [Option ID = 3052]
3. [Option ID = 3049]
4. [Option ID = 3050]
Correct Answer :-
[Option ID = 3052]
A characteristic common to polymers that can be made to conduct electricity such as polyacetylene, polypyrrole is: [Question ID =
685]
1. Conjugation throughout the polymeric chain. [Option ID = 2740]
2. A high degree of cross linking [Option ID = 2738]
3. A very low glass transition temperature [Option ID = 2737]
4. Presence of stereogenic centers of the same configuration [Option ID = 2739]
Correct Answer :-
Conjugation throughout the polymeric chain. [Option ID = 2740]
Impact factor is [Question ID = 768]
1. Ratio between citations and recent citable items publish [Option ID = 3071]
2. All of these [Option ID = 3072]
3. Addition of citations and recent citable items publish [Option ID = 3069]
4. Ratio between recent citable items publish and citations [Option ID = 3070]
Correct Answer :-
Ratio between recent citable items publish and citations [Option ID = 3070]
On the basis of oxidation-reduction potential, which of the following is most likely to occur? [Question ID = 693]
13)
14)
15)
16)
17)
1. [Option ID = 2770]
2. [Option ID = 2772]
3. [Option ID = 2771]
4. [Option ID = 2769]
Correct Answer :-
[Option ID = 2769]
How many diastereoisomers are possible for the compound 2, 4 ?diphenylcyclobutane-1, 3 di carboxylic acids. [Question ID = 725]
1. 6 [Option ID = 2899]
2. 5 [Option ID = 2898]
3. 8 [Option ID = 2900]
4. 4 [Option ID = 2897]
Correct Answer :-
5 [Option ID = 2898]
An increase in equivalent conductance of a strong electrolyte with dilution is mainly due to- [Question ID = 764]
1. increase in ionic mobility of ions [Option ID = 3055]
2. increase in number of ions [Option ID = 3054]
3. 100% ionization of electrolyte at normal dilution [Option ID = 3056]
4. increase in both i.e. number of ions and ionic mobility of ions. [Option ID = 3053]
Correct Answer :-
increase in ionic mobility of ions [Option ID = 3055]
The solid state structures of the principal allotropes of elemental boron are made up of which of the following structural units
[Question ID = 699]
1. [Option ID = 2796]
2. [Option ID = 2795]
3. [Option ID = 2794]
4. [Option ID = 2793]
Correct Answer :-
[Option ID = 2793]
The molecular geometry of thionyl chloride is best described as [Question ID = 688]
1. T-shaped [Option ID = 2752]
2. Tetrahedral [Option ID = 2751]
3. Trigonal pyramidal [Option ID = 2749]
4. Trigonal planar [Option ID = 2750]
Correct Answer :-
Trigonal pyramidal [Option ID = 2749]
In a face-center cubic (FCC) type of crystal lattice, the number of atoms belonging exclusively to each unit cell within the lattice
is/are: [Question ID = 754]
1. 2 [Option ID = 3014]
2. 1 [Option ID = 3013]
3. 3 [Option ID = 3015]
4. 4 [Option ID = 3016]
Correct Answer :-
4 [Option ID = 3016]
18)
19)
20)
21)
22)
23)
24)
Among the following, the weakest oxidizing agent is [Question ID = 675]
1. Mg (s) [Option ID = 2698]
2. [Option ID = 2699]
3. [Option ID = 2700]
4. [Option ID = 2697]
Correct Answer :-
Mg (s) [Option ID = 2698]
For a polymer, which of the following statement/s is/are true? [Question ID = 759]
1. Weight average molecular weight is almost always higher than the number average molecular weight [Option ID = 3035]
2. Formation of a polypeptide from its monomers (amino acids) is an example of addition polymerization [Option ID = 3034]
3. All of these [Option ID = 3036]
4. Vinyl polymerization is an example of condensation polymerization. [Option ID = 3033]
Correct Answer :-
Weight average molecular weight is almost always higher than the number average molecular weight [Option ID = 3035]
Quantum dots are [Question ID = 762]
1. Three dimensional [Option ID = 3048]
2. One dimensional [Option ID = 3046]
3. Two dimensional [Option ID = 3047]
4. Zero dimensional [Option ID = 3045]
Correct Answer :-
Zero dimensional [Option ID = 3045]
The unit of rate constant for a third order reaction is: [Question ID = 749]
1. [Option ID = 2993]
2. [Option ID = 2995]
3. [Option ID = 2996]
4. [Option ID = 2994]
Correct Answer :-
[Option ID = 2996]
All the following elements have at least one isotope that is not radioactive EXCEPT [Question ID = 673]
1. Pb [Option ID = 2690]
2. O [Option ID = 2689]
3. Sn [Option ID = 2691]
4. No [Option ID = 2692]
Correct Answer :-
No [Option ID = 2692]
The conditions for a species to follow Bose-Einstein statistics are; [Question ID = 736]
1. Particles are indistinguishable, with no restriction on filling up of energy levels [Option ID = 2944]
2. Particles are indistinguishable, with a restriction on filling up of energy levels [Option ID = 2943]
3. Particles are distinguishable, with a restriction on filling up of energy levels [Option ID = 2941]
4. Particles are distinguishable, with no restriction on filling up of energy levels [Option ID = 2942]
Correct Answer :-
Particles are indistinguishable, with no restriction on filling up of energy levels [Option ID = 2944]
In the kinetic theory of collisions, the SI unit of collision number, in terms of m (meter) and s (second), is:
[Question ID = 761]
25)
26)
27)
28)
29)
1. m-
2
s
-1
[Option ID = 3042]
2. m
4
s
-1
[Option ID = 3041]
3. m
2
s
-1
[Option ID = 3043]
4. None of these [Option ID = 3044]
Correct Answer :-
None of these [Option ID = 3044]
Correct characteristics of the functional groups of adenine in DNA base pair are [Question ID = 706]
1. [Option ID = 2824]
2. [Option ID = 2821]
3. [Option ID = 2823]
4. [Option ID = 2822]
Correct Answer :-
[Option ID = 2822]
The carbon monoxide molecule has an internuclear distance of 1.13 Angstroms. What is the moment of Inertia of this molecule?
[Question ID = 740]
1. [Option ID = 2960]
2. [Option ID = 2957]
3. [Option ID = 2958]
4. [Option ID = 2959]
Correct Answer :-
[Option ID = 2957]
Which of the following represent/s non-linear optical technique? [Question ID = 744]
1. Second Harmonic generation [Option ID = 2974]
2. Two-photon photoluminescence [Option ID = 2975]
3. Four-wave mixing [Option ID = 2973]
4. All of these [Option ID = 2976]
Correct Answer :-
All of these [Option ID = 2976]
. Which of the following does not affect the intensity of spectral lines of a sample? [Question ID = 743]
1. Path length of a sample [Option ID = 2972]
2. Population of energy states [Option ID = 2970]
3. Heisenberg?s Uncertainty principle [Option ID = 2971]
4. Concentration of a sample [Option ID = 2969]
Correct Answer :-
Heisenberg?s Uncertainty principle [Option ID = 2971]
[Question ID = 716]
30)
31)
1. [Option ID = 2862]
2. [Option ID = 2863]
3. [Option ID = 2861]
4. [Option ID = 2864]
Correct Answer :-
[Option ID = 2864]
[Question ID = 712]
1. [Option ID = 2845]
2. [Option ID = 2846]
3. [Option ID = 2848]
4. [Option ID = 2847]
Correct Answer :-
[Option ID = 2847]
[Question ID = 686]
1. [Option ID = 2743]
2. [Option ID = 2742]
3. [Option ID = 2741]
32)
33)
4. [Option ID = 2744]
Correct Answer :-
[Option ID = 2743]
[Question ID = 5633]
1. [Option ID = 22523]
2. [Option ID = 22524]
3. [Option ID = 22526]
4. [Option ID = 22525]
Correct Answer :-
[Option ID = 22523]
[Question ID = 730]
1. [Option ID = 2918]
2. [Option ID = 2920]
3. [Option ID = 2917]
4. [Option ID = 2919]
Correct Answer :-
34)
35)
36)
[Option ID = 2919]
[Question ID = 714]
1. [Option ID = 2856]
2. [Option ID = 2853]
3. [Option ID = 2855]
4. [Option ID = 2854]
Correct Answer :-
[Option ID = 2854]
[Question ID = 669]
1. 2 [Option ID = 2673]
2. 18 [Option ID = 2676]
3. 5 [Option ID = 2674]
4. 10 [Option ID = 2675]
Correct Answer :-
5 [Option ID = 2674]
[Question ID = 689]
1. 2 [Option ID = 2754]
2. 0 [Option ID = 2756]
3. 1 [Option ID = 2755]
37)
38)
4. 3 [Option ID = 2753]
Correct Answer :-
1 [Option ID = 2755]
[Question ID = 718]
1. C) [Option ID = 2871]
2. B) [Option ID = 2870]
3. A) [Option ID = 2869]
4. D) both (B) and (C) [Option ID = 2872]
Correct Answer :-
A) [Option ID = 2869]
[Question ID = 709]
1. [Option ID = 2836]
2. [Option ID = 2833]
3. [Option ID = 2835]
4. [Option ID = 2834]
39)
40)
41)
42)
Correct Answer :-
[Option ID = 2836]
[Question ID = 705]
1. (2Z, 4Z)-3-chlorohexa-2, 4-diene-1,6-diol. [Option ID = 2819]
2. (2E, 4E)-3-chlorohexa-2, 4-diene-1,6-diol. [Option ID = 2817]
3. (2Z, 4E)-3-chlorohexa-2, 4-diene-1,6-diol. [Option ID = 2818]
4. (2Z, 4E)-3-chlorohexa-2, 4-diene-1,6-diol. [Option ID = 2820]
Correct Answer :-
(2Z, 4E)-3-chlorohexa-2, 4-diene-1,6-diol. [Option ID = 2820]
[Question ID = 680]
1. The presence of other complexing ligands in solution affects the equilibrium concentration of metal-EDTA complexes [Option ID = 2719]
2. Metal-EDTA complexes have an equilibrium concentration independent of pH [Option ID = 2718]
3. Metal-EDTA complexes are often 2:1 in stoichiometry [Option ID = 2717]
4. Metal-EDTA complexes are less stable than the corresponding metal-ammine complexes [Option ID = 2720]
Correct Answer :-
The presence of other complexing ligands in solution affects the equilibrium concentration of metal-EDTA complexes [Option ID = 2719]
[Question ID = 671]
1. 150 g [Option ID = 2682]
2. 300 g [Option ID = 2684]
3. 120 g [Option ID = 2681]
4. 180 g [Option ID = 2683]
Correct Answer :-
300 g [Option ID = 2684]
[Question ID = 756]
1. [Option ID = 3021]
2. [Option ID = 3022]
3. [Option ID = 3024]
4. [Option ID = 3023]
Correct Answer :-
43)
44)
45)
[Option ID = 3024]
[Question ID = 750]
1. [Option ID = 2999]
2. [Option ID = 3000]
3. [Option ID = 2997]
4. [Option ID = 2998]
Correct Answer :-
[Option ID = 2997]
[Question ID = 715]
1. [Option ID = 2857]
2. [Option ID = 2860]
3. [Option ID = 2858]
4. [Option ID = 2859]
Correct Answer :-
[Option ID = 2860]
46)
47)
[Question ID = 719]
1. [Option ID = 2873]
2. [Option ID = 2876]
3. [Option ID = 2874]
4. [Option ID = 2875]
Correct Answer :-
[Question ID = 674]
1. Evaporating some water to decrease the volume of the solution. [Option ID = 2696]
2. [Option ID = 2695]
3. [Option ID = 2694]
4. [Option ID = 2693]
Correct Answer :-
[Option ID = 2693]
[Question ID = 728]
1. [Option ID = 2910]
2. [Option ID = 2911]
3. [Option ID = 2909]
4. [Option ID = 2912]
48)
49)
50)
51)
Correct Answer :-
[Option ID = 2912]
[Question ID = 697]
1. III only [Option ID = 2788]
2. II only [Option ID = 2787]
3. I only [Option ID = 2785]
4. I, II and III [Option ID = 2786]
Correct Answer :-
I, II and III [Option ID = 2786]
[Question ID = 700]
1. [Option ID = 2798]
2. [Option ID = 2797]
3. [Option ID = 2800]
4. [Option ID = 2799]
Correct Answer :-
[Option ID = 2799]
[Question ID = 678]
1. Cr
2+
has a low cationic charge [Option ID = 2711]
2. F has -1 anionic charge and highly electronegative [Option ID = 2709]
3. Spin-orbit coupling in Cr
2+
[Option ID = 2712]
4. The Jahn-Teller effect [Option ID = 2710]
Correct Answer :-
The Jahn-Teller effect [Option ID = 2710]
[Question ID = 723]
1. [Option ID = 2889]
52)
53)
2. [Option ID = 2890]
3. [Option ID = 2891]
4. [Option ID = 2892]
Correct Answer :-
[Option ID = 2889]
[Question ID = 727]
1. [Option ID = 2907]
2. [Option ID = 2906]
3. [Option ID = 2905]
4. [Option ID = 2908]
Correct Answer :-
[Option ID = 2905]
[Question ID = 707]
54)
55)
56)
1. N9 [Option ID = 2828]
2. N7 [Option ID = 2827]
3. N1 [Option ID = 2825]
4. N3 [Option ID = 2826]
Correct Answer :-
N7 [Option ID = 2827]
[Question ID = 672]
1. Bicapped prism [Option ID = 2688]
2. Square pyramidal [Option ID = 2686]
3. Trigonal planar [Option ID = 2685]
4. Bent [Option ID = 2687]
Correct Answer :-
Square pyramidal [Option ID = 2686]
[Question ID = 704]
1. [Option ID = 2816]
2. [Option ID = 2814]
3. [Option ID = 2813]
4. [Option ID = 2815]
Correct Answer :-
[Option ID = 2813]
57)
58)
[Question ID = 708]
1. [Option ID = 2829]
2. [Option ID = 2832]
3. [Option ID = 2831]
4. [Option ID = 2830]
Correct Answer :-
[Option ID = 2831]
[Question ID = 717]
1. [Option ID = 2866]
2. [Option ID = 2865]
3. [Option ID = 2867]
4. [Option ID = 2868]
Correct Answer :-
[Option ID = 2866]
59)
[Question ID = 711]
1. [Option ID = 2843]
2. [Option ID = 2842]
3. [Option ID = 2844]
4. [Option ID = 2841]
Correct Answer :-
[Option ID = 2842]
[Question ID = 710]
1. [Option ID = 2838]
2. [Option ID = 2839]
3. [Option ID = 2837]
4. [Option ID = 2840]
Correct Answer :-
60)
61)
62)
[Option ID = 2838]
[Question ID = 682]
1. 0.30 M [Option ID = 2728]
2. 0.25 M [Option ID = 2727]
3. 0.60 M [Option ID = 2725]
4. 0.10 M [Option ID = 2726]
Correct Answer :-
0.30 M [Option ID = 2728]
[Question ID = 772]
1. [Option ID = 3087]
2. [Option ID = 3088]
3. [Option ID = 3086]
4. [Option ID = 3085]
Correct Answer :-
[Option ID = 3086]
[Question ID = 733]
1. [Option ID = 2932]
63)
64)
65)
2. [Option ID = 2929]
3. [Option ID = 2930]
4. [Option ID = 2931]
Correct Answer :-
[Option ID = 2929]
[Question ID = 767]
1. 120 [Option ID = 3065]
2. 280 [Option ID = 3066]
3. 300 [Option ID = 3068]
4. 20 [Option ID = 3067]
Correct Answer :-
20 [Option ID = 3067]
[Question ID = 734]
1. [Option ID = 2936]
2. [Option ID = 2933]
3. [Option ID = 2934]
4. [Option ID = 2935]
Correct Answer :-
[Option ID = 2935]
[Question ID = 676]
66)
67)
68)
1. [Option ID = 2702]
2. [Option ID = 2703]
3. [Option ID = 2704]
4. [Option ID = 2701]
Correct Answer :-
[Option ID = 2703]
[Question ID = 729]
1. (2Z, 4Z)-3, 4-dibromo hepta-2, 4-diene [Option ID = 2914]
2. (2E, 4E)-3, 4-dibromo hepta-2, 4-diene [Option ID = 2915]
3. (2E, 4Z)-3, 4-dibromo hepta-2, 4-diene [Option ID = 2916]
4. (2Z, 4E)-3, 4-dibromo hepta-2, 4-diene [Option ID = 2913]
Correct Answer :-
(2E, 4Z)-3, 4-dibromo hepta-2, 4-diene [Option ID = 2916]
[Question ID = 765]
1. [Option ID = 3060]
2. [Option ID = 3057]
3. [Option ID = 3058]
4. [Option ID = 3059]
Correct Answer :-
[Option ID = 3059]
[Question ID = 724]
1. [Option ID = 2893]
69)
2. [Option ID = 2895]
3. [Option ID = 2896]
4. [Option ID = 2894]
Correct Answer :-
[Option ID = 2893]
[Question ID = 721]
1. [Option ID = 2881]
2. [Option ID = 2884]
3. [Option ID = 2882]
4. [Option ID = 2883]
Correct Answer :-
[Option ID = 2882]
FirstRanker.com - FirstRanker's Choice
1)
2)
3)
4)
5)
DU PhD in Chemistry
Topic:- DU_J18_PHD_CHEM
Which of the following statements about sulfur dioxide is true?
[Question ID = 677]
1. It forms a S-S dimer in condensed phase [Option ID = 2707]
2. Its anhydride of sulfuric acid [Option ID = 2706]
3. Its O-S-O angle is 180
0
[Option ID = 2708]
4. It is a product of the combustion of fossil fuels that contain sulfur [Option ID = 2705]
Correct Answer :-
It is a product of the combustion of fossil fuels that contain sulfur [Option ID = 2705]
Which of the following is a strong acid in pure liquid HF
[Question ID = 683]
1. H
2
O [Option ID = 2731]
2. NaF [Option ID = 2729]
3. CH
3
COOH [Option ID = 2730]
4. SbF
5
[Option ID = 2732]
Correct Answer :-
SbF
5
[Option ID = 2732]
Each of the following molecules can act as a chelating agent EXCEPT [Question ID = 679]
1. [Option ID = 2716]
2. [Option ID = 2714]
3. [Option ID = 2713]
4. [Option ID = 2715]
Correct Answer :-
[Option ID = 2714]
What is correct about h-index?
[Question ID = 758]
1. Alternative of impact factor [Option ID = 3031]
2. Based on most quoted papers [Option ID = 3030]
3. Quantify scientific productivity [Option ID = 3029]
4. All of these [Option ID = 3032]
Correct Answer :-
All of these [Option ID = 3032]
The hyperfine electron spin resonance (e.s.r.) spectrum of the benzene radical has how many lines? [Question ID = 748]
1. 12 [Option ID = 2992]
2. 7 [Option ID = 2990]
3. 1 [Option ID = 2991]
4. 6 [Option ID = 2989]
Correct Answer :-
7 [Option ID = 2990]
6)
7)
8)
9)
10)
11)
12)
The energy changes involving the core electrons of an atom or molecule are expressed in which region of the electromagnetic
spectrum? [Question ID = 742]
1. Ultraviolet and Visible region [Option ID = 2967]
2. X-ray region [Option ID = 2968]
3. Radiofrequency region [Option ID = 2966]
4. Infra-red region [Option ID = 2965]
Correct Answer :-
X-ray region [Option ID = 2968]
Find out the expected intensity ratio of M and M+1 signal for the Naphthalene molecular ion [Question ID = 726]
1. 99:1.1 [Option ID = 2903]
2. 1.1:99 [Option ID = 2904]
3. 9:01 [Option ID = 2901]
4. 1:9 [Option ID = 2902]
Correct Answer :-
9:01 [Option ID = 2901]
Cobalt-60 is used in radiation therapy of cancer and can be produced by the bombardment of Cobalt-59 with [Question ID = 692]
1. Alpha particles [Option ID = 2765]
2. Beta particles [Option ID = 2767]
3. Neutrons [Option ID = 2766]
4. Gamma rays [Option ID = 2768]
Correct Answer :-
Neutrons [Option ID = 2766]
The standard emf of galvanic cell involving 3 moles of electrons in its redox reaction is 0.59 V. The equilibrium constant for the
reaction of the cell is- [Question ID = 763]
1. [Option ID = 3051]
2. [Option ID = 3052]
3. [Option ID = 3049]
4. [Option ID = 3050]
Correct Answer :-
[Option ID = 3052]
A characteristic common to polymers that can be made to conduct electricity such as polyacetylene, polypyrrole is: [Question ID =
685]
1. Conjugation throughout the polymeric chain. [Option ID = 2740]
2. A high degree of cross linking [Option ID = 2738]
3. A very low glass transition temperature [Option ID = 2737]
4. Presence of stereogenic centers of the same configuration [Option ID = 2739]
Correct Answer :-
Conjugation throughout the polymeric chain. [Option ID = 2740]
Impact factor is [Question ID = 768]
1. Ratio between citations and recent citable items publish [Option ID = 3071]
2. All of these [Option ID = 3072]
3. Addition of citations and recent citable items publish [Option ID = 3069]
4. Ratio between recent citable items publish and citations [Option ID = 3070]
Correct Answer :-
Ratio between recent citable items publish and citations [Option ID = 3070]
On the basis of oxidation-reduction potential, which of the following is most likely to occur? [Question ID = 693]
13)
14)
15)
16)
17)
1. [Option ID = 2770]
2. [Option ID = 2772]
3. [Option ID = 2771]
4. [Option ID = 2769]
Correct Answer :-
[Option ID = 2769]
How many diastereoisomers are possible for the compound 2, 4 ?diphenylcyclobutane-1, 3 di carboxylic acids. [Question ID = 725]
1. 6 [Option ID = 2899]
2. 5 [Option ID = 2898]
3. 8 [Option ID = 2900]
4. 4 [Option ID = 2897]
Correct Answer :-
5 [Option ID = 2898]
An increase in equivalent conductance of a strong electrolyte with dilution is mainly due to- [Question ID = 764]
1. increase in ionic mobility of ions [Option ID = 3055]
2. increase in number of ions [Option ID = 3054]
3. 100% ionization of electrolyte at normal dilution [Option ID = 3056]
4. increase in both i.e. number of ions and ionic mobility of ions. [Option ID = 3053]
Correct Answer :-
increase in ionic mobility of ions [Option ID = 3055]
The solid state structures of the principal allotropes of elemental boron are made up of which of the following structural units
[Question ID = 699]
1. [Option ID = 2796]
2. [Option ID = 2795]
3. [Option ID = 2794]
4. [Option ID = 2793]
Correct Answer :-
[Option ID = 2793]
The molecular geometry of thionyl chloride is best described as [Question ID = 688]
1. T-shaped [Option ID = 2752]
2. Tetrahedral [Option ID = 2751]
3. Trigonal pyramidal [Option ID = 2749]
4. Trigonal planar [Option ID = 2750]
Correct Answer :-
Trigonal pyramidal [Option ID = 2749]
In a face-center cubic (FCC) type of crystal lattice, the number of atoms belonging exclusively to each unit cell within the lattice
is/are: [Question ID = 754]
1. 2 [Option ID = 3014]
2. 1 [Option ID = 3013]
3. 3 [Option ID = 3015]
4. 4 [Option ID = 3016]
Correct Answer :-
4 [Option ID = 3016]
18)
19)
20)
21)
22)
23)
24)
Among the following, the weakest oxidizing agent is [Question ID = 675]
1. Mg (s) [Option ID = 2698]
2. [Option ID = 2699]
3. [Option ID = 2700]
4. [Option ID = 2697]
Correct Answer :-
Mg (s) [Option ID = 2698]
For a polymer, which of the following statement/s is/are true? [Question ID = 759]
1. Weight average molecular weight is almost always higher than the number average molecular weight [Option ID = 3035]
2. Formation of a polypeptide from its monomers (amino acids) is an example of addition polymerization [Option ID = 3034]
3. All of these [Option ID = 3036]
4. Vinyl polymerization is an example of condensation polymerization. [Option ID = 3033]
Correct Answer :-
Weight average molecular weight is almost always higher than the number average molecular weight [Option ID = 3035]
Quantum dots are [Question ID = 762]
1. Three dimensional [Option ID = 3048]
2. One dimensional [Option ID = 3046]
3. Two dimensional [Option ID = 3047]
4. Zero dimensional [Option ID = 3045]
Correct Answer :-
Zero dimensional [Option ID = 3045]
The unit of rate constant for a third order reaction is: [Question ID = 749]
1. [Option ID = 2993]
2. [Option ID = 2995]
3. [Option ID = 2996]
4. [Option ID = 2994]
Correct Answer :-
[Option ID = 2996]
All the following elements have at least one isotope that is not radioactive EXCEPT [Question ID = 673]
1. Pb [Option ID = 2690]
2. O [Option ID = 2689]
3. Sn [Option ID = 2691]
4. No [Option ID = 2692]
Correct Answer :-
No [Option ID = 2692]
The conditions for a species to follow Bose-Einstein statistics are; [Question ID = 736]
1. Particles are indistinguishable, with no restriction on filling up of energy levels [Option ID = 2944]
2. Particles are indistinguishable, with a restriction on filling up of energy levels [Option ID = 2943]
3. Particles are distinguishable, with a restriction on filling up of energy levels [Option ID = 2941]
4. Particles are distinguishable, with no restriction on filling up of energy levels [Option ID = 2942]
Correct Answer :-
Particles are indistinguishable, with no restriction on filling up of energy levels [Option ID = 2944]
In the kinetic theory of collisions, the SI unit of collision number, in terms of m (meter) and s (second), is:
[Question ID = 761]
25)
26)
27)
28)
29)
1. m-
2
s
-1
[Option ID = 3042]
2. m
4
s
-1
[Option ID = 3041]
3. m
2
s
-1
[Option ID = 3043]
4. None of these [Option ID = 3044]
Correct Answer :-
None of these [Option ID = 3044]
Correct characteristics of the functional groups of adenine in DNA base pair are [Question ID = 706]
1. [Option ID = 2824]
2. [Option ID = 2821]
3. [Option ID = 2823]
4. [Option ID = 2822]
Correct Answer :-
[Option ID = 2822]
The carbon monoxide molecule has an internuclear distance of 1.13 Angstroms. What is the moment of Inertia of this molecule?
[Question ID = 740]
1. [Option ID = 2960]
2. [Option ID = 2957]
3. [Option ID = 2958]
4. [Option ID = 2959]
Correct Answer :-
[Option ID = 2957]
Which of the following represent/s non-linear optical technique? [Question ID = 744]
1. Second Harmonic generation [Option ID = 2974]
2. Two-photon photoluminescence [Option ID = 2975]
3. Four-wave mixing [Option ID = 2973]
4. All of these [Option ID = 2976]
Correct Answer :-
All of these [Option ID = 2976]
. Which of the following does not affect the intensity of spectral lines of a sample? [Question ID = 743]
1. Path length of a sample [Option ID = 2972]
2. Population of energy states [Option ID = 2970]
3. Heisenberg?s Uncertainty principle [Option ID = 2971]
4. Concentration of a sample [Option ID = 2969]
Correct Answer :-
Heisenberg?s Uncertainty principle [Option ID = 2971]
[Question ID = 716]
30)
31)
1. [Option ID = 2862]
2. [Option ID = 2863]
3. [Option ID = 2861]
4. [Option ID = 2864]
Correct Answer :-
[Option ID = 2864]
[Question ID = 712]
1. [Option ID = 2845]
2. [Option ID = 2846]
3. [Option ID = 2848]
4. [Option ID = 2847]
Correct Answer :-
[Option ID = 2847]
[Question ID = 686]
1. [Option ID = 2743]
2. [Option ID = 2742]
3. [Option ID = 2741]
32)
33)
4. [Option ID = 2744]
Correct Answer :-
[Option ID = 2743]
[Question ID = 5633]
1. [Option ID = 22523]
2. [Option ID = 22524]
3. [Option ID = 22526]
4. [Option ID = 22525]
Correct Answer :-
[Option ID = 22523]
[Question ID = 730]
1. [Option ID = 2918]
2. [Option ID = 2920]
3. [Option ID = 2917]
4. [Option ID = 2919]
Correct Answer :-
34)
35)
36)
[Option ID = 2919]
[Question ID = 714]
1. [Option ID = 2856]
2. [Option ID = 2853]
3. [Option ID = 2855]
4. [Option ID = 2854]
Correct Answer :-
[Option ID = 2854]
[Question ID = 669]
1. 2 [Option ID = 2673]
2. 18 [Option ID = 2676]
3. 5 [Option ID = 2674]
4. 10 [Option ID = 2675]
Correct Answer :-
5 [Option ID = 2674]
[Question ID = 689]
1. 2 [Option ID = 2754]
2. 0 [Option ID = 2756]
3. 1 [Option ID = 2755]
37)
38)
4. 3 [Option ID = 2753]
Correct Answer :-
1 [Option ID = 2755]
[Question ID = 718]
1. C) [Option ID = 2871]
2. B) [Option ID = 2870]
3. A) [Option ID = 2869]
4. D) both (B) and (C) [Option ID = 2872]
Correct Answer :-
A) [Option ID = 2869]
[Question ID = 709]
1. [Option ID = 2836]
2. [Option ID = 2833]
3. [Option ID = 2835]
4. [Option ID = 2834]
39)
40)
41)
42)
Correct Answer :-
[Option ID = 2836]
[Question ID = 705]
1. (2Z, 4Z)-3-chlorohexa-2, 4-diene-1,6-diol. [Option ID = 2819]
2. (2E, 4E)-3-chlorohexa-2, 4-diene-1,6-diol. [Option ID = 2817]
3. (2Z, 4E)-3-chlorohexa-2, 4-diene-1,6-diol. [Option ID = 2818]
4. (2Z, 4E)-3-chlorohexa-2, 4-diene-1,6-diol. [Option ID = 2820]
Correct Answer :-
(2Z, 4E)-3-chlorohexa-2, 4-diene-1,6-diol. [Option ID = 2820]
[Question ID = 680]
1. The presence of other complexing ligands in solution affects the equilibrium concentration of metal-EDTA complexes [Option ID = 2719]
2. Metal-EDTA complexes have an equilibrium concentration independent of pH [Option ID = 2718]
3. Metal-EDTA complexes are often 2:1 in stoichiometry [Option ID = 2717]
4. Metal-EDTA complexes are less stable than the corresponding metal-ammine complexes [Option ID = 2720]
Correct Answer :-
The presence of other complexing ligands in solution affects the equilibrium concentration of metal-EDTA complexes [Option ID = 2719]
[Question ID = 671]
1. 150 g [Option ID = 2682]
2. 300 g [Option ID = 2684]
3. 120 g [Option ID = 2681]
4. 180 g [Option ID = 2683]
Correct Answer :-
300 g [Option ID = 2684]
[Question ID = 756]
1. [Option ID = 3021]
2. [Option ID = 3022]
3. [Option ID = 3024]
4. [Option ID = 3023]
Correct Answer :-
43)
44)
45)
[Option ID = 3024]
[Question ID = 750]
1. [Option ID = 2999]
2. [Option ID = 3000]
3. [Option ID = 2997]
4. [Option ID = 2998]
Correct Answer :-
[Option ID = 2997]
[Question ID = 715]
1. [Option ID = 2857]
2. [Option ID = 2860]
3. [Option ID = 2858]
4. [Option ID = 2859]
Correct Answer :-
[Option ID = 2860]
46)
47)
[Question ID = 719]
1. [Option ID = 2873]
2. [Option ID = 2876]
3. [Option ID = 2874]
4. [Option ID = 2875]
Correct Answer :-
[Question ID = 674]
1. Evaporating some water to decrease the volume of the solution. [Option ID = 2696]
2. [Option ID = 2695]
3. [Option ID = 2694]
4. [Option ID = 2693]
Correct Answer :-
[Option ID = 2693]
[Question ID = 728]
1. [Option ID = 2910]
2. [Option ID = 2911]
3. [Option ID = 2909]
4. [Option ID = 2912]
48)
49)
50)
51)
Correct Answer :-
[Option ID = 2912]
[Question ID = 697]
1. III only [Option ID = 2788]
2. II only [Option ID = 2787]
3. I only [Option ID = 2785]
4. I, II and III [Option ID = 2786]
Correct Answer :-
I, II and III [Option ID = 2786]
[Question ID = 700]
1. [Option ID = 2798]
2. [Option ID = 2797]
3. [Option ID = 2800]
4. [Option ID = 2799]
Correct Answer :-
[Option ID = 2799]
[Question ID = 678]
1. Cr
2+
has a low cationic charge [Option ID = 2711]
2. F has -1 anionic charge and highly electronegative [Option ID = 2709]
3. Spin-orbit coupling in Cr
2+
[Option ID = 2712]
4. The Jahn-Teller effect [Option ID = 2710]
Correct Answer :-
The Jahn-Teller effect [Option ID = 2710]
[Question ID = 723]
1. [Option ID = 2889]
52)
53)
2. [Option ID = 2890]
3. [Option ID = 2891]
4. [Option ID = 2892]
Correct Answer :-
[Option ID = 2889]
[Question ID = 727]
1. [Option ID = 2907]
2. [Option ID = 2906]
3. [Option ID = 2905]
4. [Option ID = 2908]
Correct Answer :-
[Option ID = 2905]
[Question ID = 707]
54)
55)
56)
1. N9 [Option ID = 2828]
2. N7 [Option ID = 2827]
3. N1 [Option ID = 2825]
4. N3 [Option ID = 2826]
Correct Answer :-
N7 [Option ID = 2827]
[Question ID = 672]
1. Bicapped prism [Option ID = 2688]
2. Square pyramidal [Option ID = 2686]
3. Trigonal planar [Option ID = 2685]
4. Bent [Option ID = 2687]
Correct Answer :-
Square pyramidal [Option ID = 2686]
[Question ID = 704]
1. [Option ID = 2816]
2. [Option ID = 2814]
3. [Option ID = 2813]
4. [Option ID = 2815]
Correct Answer :-
[Option ID = 2813]
57)
58)
[Question ID = 708]
1. [Option ID = 2829]
2. [Option ID = 2832]
3. [Option ID = 2831]
4. [Option ID = 2830]
Correct Answer :-
[Option ID = 2831]
[Question ID = 717]
1. [Option ID = 2866]
2. [Option ID = 2865]
3. [Option ID = 2867]
4. [Option ID = 2868]
Correct Answer :-
[Option ID = 2866]
59)
[Question ID = 711]
1. [Option ID = 2843]
2. [Option ID = 2842]
3. [Option ID = 2844]
4. [Option ID = 2841]
Correct Answer :-
[Option ID = 2842]
[Question ID = 710]
1. [Option ID = 2838]
2. [Option ID = 2839]
3. [Option ID = 2837]
4. [Option ID = 2840]
Correct Answer :-
60)
61)
62)
[Option ID = 2838]
[Question ID = 682]
1. 0.30 M [Option ID = 2728]
2. 0.25 M [Option ID = 2727]
3. 0.60 M [Option ID = 2725]
4. 0.10 M [Option ID = 2726]
Correct Answer :-
0.30 M [Option ID = 2728]
[Question ID = 772]
1. [Option ID = 3087]
2. [Option ID = 3088]
3. [Option ID = 3086]
4. [Option ID = 3085]
Correct Answer :-
[Option ID = 3086]
[Question ID = 733]
1. [Option ID = 2932]
63)
64)
65)
2. [Option ID = 2929]
3. [Option ID = 2930]
4. [Option ID = 2931]
Correct Answer :-
[Option ID = 2929]
[Question ID = 767]
1. 120 [Option ID = 3065]
2. 280 [Option ID = 3066]
3. 300 [Option ID = 3068]
4. 20 [Option ID = 3067]
Correct Answer :-
20 [Option ID = 3067]
[Question ID = 734]
1. [Option ID = 2936]
2. [Option ID = 2933]
3. [Option ID = 2934]
4. [Option ID = 2935]
Correct Answer :-
[Option ID = 2935]
[Question ID = 676]
66)
67)
68)
1. [Option ID = 2702]
2. [Option ID = 2703]
3. [Option ID = 2704]
4. [Option ID = 2701]
Correct Answer :-
[Option ID = 2703]
[Question ID = 729]
1. (2Z, 4Z)-3, 4-dibromo hepta-2, 4-diene [Option ID = 2914]
2. (2E, 4E)-3, 4-dibromo hepta-2, 4-diene [Option ID = 2915]
3. (2E, 4Z)-3, 4-dibromo hepta-2, 4-diene [Option ID = 2916]
4. (2Z, 4E)-3, 4-dibromo hepta-2, 4-diene [Option ID = 2913]
Correct Answer :-
(2E, 4Z)-3, 4-dibromo hepta-2, 4-diene [Option ID = 2916]
[Question ID = 765]
1. [Option ID = 3060]
2. [Option ID = 3057]
3. [Option ID = 3058]
4. [Option ID = 3059]
Correct Answer :-
[Option ID = 3059]
[Question ID = 724]
1. [Option ID = 2893]
69)
2. [Option ID = 2895]
3. [Option ID = 2896]
4. [Option ID = 2894]
Correct Answer :-
[Option ID = 2893]
[Question ID = 721]
1. [Option ID = 2881]
2. [Option ID = 2884]
3. [Option ID = 2882]
4. [Option ID = 2883]
Correct Answer :-
[Option ID = 2882]
70)
71)
72)
73)
[Question ID = 766]
1. 512 times [Option ID = 3064]
2. 256 times [Option ID = 3063]
3. 128 times [Option ID = 3062]
4. 64 times [Option ID = 3061]
Correct Answer :-
512 times [Option ID = 3064]
[Question ID = 670]
1. 2 [Option ID = 2678]
2. 5 [Option ID = 2680]
3. 1 [Option ID = 2677]
4. 3 [Option ID = 2679]
Correct Answer :-
5 [Option ID = 2680]
[Question ID = 731]
1. [Option ID = 2924]
2. [Option ID = 2923]
3. [Option ID = 2922]
4. [Option ID = 2921]
Correct Answer :-
[Option ID = 2922]
[Question ID = 690]
1. [Option ID = 2760]
2. [Option ID = 2759]
3. [Option ID = 2757]
4. [Option ID = 2758]
FirstRanker.com - FirstRanker's Choice
1)
2)
3)
4)
5)
DU PhD in Chemistry
Topic:- DU_J18_PHD_CHEM
Which of the following statements about sulfur dioxide is true?
[Question ID = 677]
1. It forms a S-S dimer in condensed phase [Option ID = 2707]
2. Its anhydride of sulfuric acid [Option ID = 2706]
3. Its O-S-O angle is 180
0
[Option ID = 2708]
4. It is a product of the combustion of fossil fuels that contain sulfur [Option ID = 2705]
Correct Answer :-
It is a product of the combustion of fossil fuels that contain sulfur [Option ID = 2705]
Which of the following is a strong acid in pure liquid HF
[Question ID = 683]
1. H
2
O [Option ID = 2731]
2. NaF [Option ID = 2729]
3. CH
3
COOH [Option ID = 2730]
4. SbF
5
[Option ID = 2732]
Correct Answer :-
SbF
5
[Option ID = 2732]
Each of the following molecules can act as a chelating agent EXCEPT [Question ID = 679]
1. [Option ID = 2716]
2. [Option ID = 2714]
3. [Option ID = 2713]
4. [Option ID = 2715]
Correct Answer :-
[Option ID = 2714]
What is correct about h-index?
[Question ID = 758]
1. Alternative of impact factor [Option ID = 3031]
2. Based on most quoted papers [Option ID = 3030]
3. Quantify scientific productivity [Option ID = 3029]
4. All of these [Option ID = 3032]
Correct Answer :-
All of these [Option ID = 3032]
The hyperfine electron spin resonance (e.s.r.) spectrum of the benzene radical has how many lines? [Question ID = 748]
1. 12 [Option ID = 2992]
2. 7 [Option ID = 2990]
3. 1 [Option ID = 2991]
4. 6 [Option ID = 2989]
Correct Answer :-
7 [Option ID = 2990]
6)
7)
8)
9)
10)
11)
12)
The energy changes involving the core electrons of an atom or molecule are expressed in which region of the electromagnetic
spectrum? [Question ID = 742]
1. Ultraviolet and Visible region [Option ID = 2967]
2. X-ray region [Option ID = 2968]
3. Radiofrequency region [Option ID = 2966]
4. Infra-red region [Option ID = 2965]
Correct Answer :-
X-ray region [Option ID = 2968]
Find out the expected intensity ratio of M and M+1 signal for the Naphthalene molecular ion [Question ID = 726]
1. 99:1.1 [Option ID = 2903]
2. 1.1:99 [Option ID = 2904]
3. 9:01 [Option ID = 2901]
4. 1:9 [Option ID = 2902]
Correct Answer :-
9:01 [Option ID = 2901]
Cobalt-60 is used in radiation therapy of cancer and can be produced by the bombardment of Cobalt-59 with [Question ID = 692]
1. Alpha particles [Option ID = 2765]
2. Beta particles [Option ID = 2767]
3. Neutrons [Option ID = 2766]
4. Gamma rays [Option ID = 2768]
Correct Answer :-
Neutrons [Option ID = 2766]
The standard emf of galvanic cell involving 3 moles of electrons in its redox reaction is 0.59 V. The equilibrium constant for the
reaction of the cell is- [Question ID = 763]
1. [Option ID = 3051]
2. [Option ID = 3052]
3. [Option ID = 3049]
4. [Option ID = 3050]
Correct Answer :-
[Option ID = 3052]
A characteristic common to polymers that can be made to conduct electricity such as polyacetylene, polypyrrole is: [Question ID =
685]
1. Conjugation throughout the polymeric chain. [Option ID = 2740]
2. A high degree of cross linking [Option ID = 2738]
3. A very low glass transition temperature [Option ID = 2737]
4. Presence of stereogenic centers of the same configuration [Option ID = 2739]
Correct Answer :-
Conjugation throughout the polymeric chain. [Option ID = 2740]
Impact factor is [Question ID = 768]
1. Ratio between citations and recent citable items publish [Option ID = 3071]
2. All of these [Option ID = 3072]
3. Addition of citations and recent citable items publish [Option ID = 3069]
4. Ratio between recent citable items publish and citations [Option ID = 3070]
Correct Answer :-
Ratio between recent citable items publish and citations [Option ID = 3070]
On the basis of oxidation-reduction potential, which of the following is most likely to occur? [Question ID = 693]
13)
14)
15)
16)
17)
1. [Option ID = 2770]
2. [Option ID = 2772]
3. [Option ID = 2771]
4. [Option ID = 2769]
Correct Answer :-
[Option ID = 2769]
How many diastereoisomers are possible for the compound 2, 4 ?diphenylcyclobutane-1, 3 di carboxylic acids. [Question ID = 725]
1. 6 [Option ID = 2899]
2. 5 [Option ID = 2898]
3. 8 [Option ID = 2900]
4. 4 [Option ID = 2897]
Correct Answer :-
5 [Option ID = 2898]
An increase in equivalent conductance of a strong electrolyte with dilution is mainly due to- [Question ID = 764]
1. increase in ionic mobility of ions [Option ID = 3055]
2. increase in number of ions [Option ID = 3054]
3. 100% ionization of electrolyte at normal dilution [Option ID = 3056]
4. increase in both i.e. number of ions and ionic mobility of ions. [Option ID = 3053]
Correct Answer :-
increase in ionic mobility of ions [Option ID = 3055]
The solid state structures of the principal allotropes of elemental boron are made up of which of the following structural units
[Question ID = 699]
1. [Option ID = 2796]
2. [Option ID = 2795]
3. [Option ID = 2794]
4. [Option ID = 2793]
Correct Answer :-
[Option ID = 2793]
The molecular geometry of thionyl chloride is best described as [Question ID = 688]
1. T-shaped [Option ID = 2752]
2. Tetrahedral [Option ID = 2751]
3. Trigonal pyramidal [Option ID = 2749]
4. Trigonal planar [Option ID = 2750]
Correct Answer :-
Trigonal pyramidal [Option ID = 2749]
In a face-center cubic (FCC) type of crystal lattice, the number of atoms belonging exclusively to each unit cell within the lattice
is/are: [Question ID = 754]
1. 2 [Option ID = 3014]
2. 1 [Option ID = 3013]
3. 3 [Option ID = 3015]
4. 4 [Option ID = 3016]
Correct Answer :-
4 [Option ID = 3016]
18)
19)
20)
21)
22)
23)
24)
Among the following, the weakest oxidizing agent is [Question ID = 675]
1. Mg (s) [Option ID = 2698]
2. [Option ID = 2699]
3. [Option ID = 2700]
4. [Option ID = 2697]
Correct Answer :-
Mg (s) [Option ID = 2698]
For a polymer, which of the following statement/s is/are true? [Question ID = 759]
1. Weight average molecular weight is almost always higher than the number average molecular weight [Option ID = 3035]
2. Formation of a polypeptide from its monomers (amino acids) is an example of addition polymerization [Option ID = 3034]
3. All of these [Option ID = 3036]
4. Vinyl polymerization is an example of condensation polymerization. [Option ID = 3033]
Correct Answer :-
Weight average molecular weight is almost always higher than the number average molecular weight [Option ID = 3035]
Quantum dots are [Question ID = 762]
1. Three dimensional [Option ID = 3048]
2. One dimensional [Option ID = 3046]
3. Two dimensional [Option ID = 3047]
4. Zero dimensional [Option ID = 3045]
Correct Answer :-
Zero dimensional [Option ID = 3045]
The unit of rate constant for a third order reaction is: [Question ID = 749]
1. [Option ID = 2993]
2. [Option ID = 2995]
3. [Option ID = 2996]
4. [Option ID = 2994]
Correct Answer :-
[Option ID = 2996]
All the following elements have at least one isotope that is not radioactive EXCEPT [Question ID = 673]
1. Pb [Option ID = 2690]
2. O [Option ID = 2689]
3. Sn [Option ID = 2691]
4. No [Option ID = 2692]
Correct Answer :-
No [Option ID = 2692]
The conditions for a species to follow Bose-Einstein statistics are; [Question ID = 736]
1. Particles are indistinguishable, with no restriction on filling up of energy levels [Option ID = 2944]
2. Particles are indistinguishable, with a restriction on filling up of energy levels [Option ID = 2943]
3. Particles are distinguishable, with a restriction on filling up of energy levels [Option ID = 2941]
4. Particles are distinguishable, with no restriction on filling up of energy levels [Option ID = 2942]
Correct Answer :-
Particles are indistinguishable, with no restriction on filling up of energy levels [Option ID = 2944]
In the kinetic theory of collisions, the SI unit of collision number, in terms of m (meter) and s (second), is:
[Question ID = 761]
25)
26)
27)
28)
29)
1. m-
2
s
-1
[Option ID = 3042]
2. m
4
s
-1
[Option ID = 3041]
3. m
2
s
-1
[Option ID = 3043]
4. None of these [Option ID = 3044]
Correct Answer :-
None of these [Option ID = 3044]
Correct characteristics of the functional groups of adenine in DNA base pair are [Question ID = 706]
1. [Option ID = 2824]
2. [Option ID = 2821]
3. [Option ID = 2823]
4. [Option ID = 2822]
Correct Answer :-
[Option ID = 2822]
The carbon monoxide molecule has an internuclear distance of 1.13 Angstroms. What is the moment of Inertia of this molecule?
[Question ID = 740]
1. [Option ID = 2960]
2. [Option ID = 2957]
3. [Option ID = 2958]
4. [Option ID = 2959]
Correct Answer :-
[Option ID = 2957]
Which of the following represent/s non-linear optical technique? [Question ID = 744]
1. Second Harmonic generation [Option ID = 2974]
2. Two-photon photoluminescence [Option ID = 2975]
3. Four-wave mixing [Option ID = 2973]
4. All of these [Option ID = 2976]
Correct Answer :-
All of these [Option ID = 2976]
. Which of the following does not affect the intensity of spectral lines of a sample? [Question ID = 743]
1. Path length of a sample [Option ID = 2972]
2. Population of energy states [Option ID = 2970]
3. Heisenberg?s Uncertainty principle [Option ID = 2971]
4. Concentration of a sample [Option ID = 2969]
Correct Answer :-
Heisenberg?s Uncertainty principle [Option ID = 2971]
[Question ID = 716]
30)
31)
1. [Option ID = 2862]
2. [Option ID = 2863]
3. [Option ID = 2861]
4. [Option ID = 2864]
Correct Answer :-
[Option ID = 2864]
[Question ID = 712]
1. [Option ID = 2845]
2. [Option ID = 2846]
3. [Option ID = 2848]
4. [Option ID = 2847]
Correct Answer :-
[Option ID = 2847]
[Question ID = 686]
1. [Option ID = 2743]
2. [Option ID = 2742]
3. [Option ID = 2741]
32)
33)
4. [Option ID = 2744]
Correct Answer :-
[Option ID = 2743]
[Question ID = 5633]
1. [Option ID = 22523]
2. [Option ID = 22524]
3. [Option ID = 22526]
4. [Option ID = 22525]
Correct Answer :-
[Option ID = 22523]
[Question ID = 730]
1. [Option ID = 2918]
2. [Option ID = 2920]
3. [Option ID = 2917]
4. [Option ID = 2919]
Correct Answer :-
34)
35)
36)
[Option ID = 2919]
[Question ID = 714]
1. [Option ID = 2856]
2. [Option ID = 2853]
3. [Option ID = 2855]
4. [Option ID = 2854]
Correct Answer :-
[Option ID = 2854]
[Question ID = 669]
1. 2 [Option ID = 2673]
2. 18 [Option ID = 2676]
3. 5 [Option ID = 2674]
4. 10 [Option ID = 2675]
Correct Answer :-
5 [Option ID = 2674]
[Question ID = 689]
1. 2 [Option ID = 2754]
2. 0 [Option ID = 2756]
3. 1 [Option ID = 2755]
37)
38)
4. 3 [Option ID = 2753]
Correct Answer :-
1 [Option ID = 2755]
[Question ID = 718]
1. C) [Option ID = 2871]
2. B) [Option ID = 2870]
3. A) [Option ID = 2869]
4. D) both (B) and (C) [Option ID = 2872]
Correct Answer :-
A) [Option ID = 2869]
[Question ID = 709]
1. [Option ID = 2836]
2. [Option ID = 2833]
3. [Option ID = 2835]
4. [Option ID = 2834]
39)
40)
41)
42)
Correct Answer :-
[Option ID = 2836]
[Question ID = 705]
1. (2Z, 4Z)-3-chlorohexa-2, 4-diene-1,6-diol. [Option ID = 2819]
2. (2E, 4E)-3-chlorohexa-2, 4-diene-1,6-diol. [Option ID = 2817]
3. (2Z, 4E)-3-chlorohexa-2, 4-diene-1,6-diol. [Option ID = 2818]
4. (2Z, 4E)-3-chlorohexa-2, 4-diene-1,6-diol. [Option ID = 2820]
Correct Answer :-
(2Z, 4E)-3-chlorohexa-2, 4-diene-1,6-diol. [Option ID = 2820]
[Question ID = 680]
1. The presence of other complexing ligands in solution affects the equilibrium concentration of metal-EDTA complexes [Option ID = 2719]
2. Metal-EDTA complexes have an equilibrium concentration independent of pH [Option ID = 2718]
3. Metal-EDTA complexes are often 2:1 in stoichiometry [Option ID = 2717]
4. Metal-EDTA complexes are less stable than the corresponding metal-ammine complexes [Option ID = 2720]
Correct Answer :-
The presence of other complexing ligands in solution affects the equilibrium concentration of metal-EDTA complexes [Option ID = 2719]
[Question ID = 671]
1. 150 g [Option ID = 2682]
2. 300 g [Option ID = 2684]
3. 120 g [Option ID = 2681]
4. 180 g [Option ID = 2683]
Correct Answer :-
300 g [Option ID = 2684]
[Question ID = 756]
1. [Option ID = 3021]
2. [Option ID = 3022]
3. [Option ID = 3024]
4. [Option ID = 3023]
Correct Answer :-
43)
44)
45)
[Option ID = 3024]
[Question ID = 750]
1. [Option ID = 2999]
2. [Option ID = 3000]
3. [Option ID = 2997]
4. [Option ID = 2998]
Correct Answer :-
[Option ID = 2997]
[Question ID = 715]
1. [Option ID = 2857]
2. [Option ID = 2860]
3. [Option ID = 2858]
4. [Option ID = 2859]
Correct Answer :-
[Option ID = 2860]
46)
47)
[Question ID = 719]
1. [Option ID = 2873]
2. [Option ID = 2876]
3. [Option ID = 2874]
4. [Option ID = 2875]
Correct Answer :-
[Question ID = 674]
1. Evaporating some water to decrease the volume of the solution. [Option ID = 2696]
2. [Option ID = 2695]
3. [Option ID = 2694]
4. [Option ID = 2693]
Correct Answer :-
[Option ID = 2693]
[Question ID = 728]
1. [Option ID = 2910]
2. [Option ID = 2911]
3. [Option ID = 2909]
4. [Option ID = 2912]
48)
49)
50)
51)
Correct Answer :-
[Option ID = 2912]
[Question ID = 697]
1. III only [Option ID = 2788]
2. II only [Option ID = 2787]
3. I only [Option ID = 2785]
4. I, II and III [Option ID = 2786]
Correct Answer :-
I, II and III [Option ID = 2786]
[Question ID = 700]
1. [Option ID = 2798]
2. [Option ID = 2797]
3. [Option ID = 2800]
4. [Option ID = 2799]
Correct Answer :-
[Option ID = 2799]
[Question ID = 678]
1. Cr
2+
has a low cationic charge [Option ID = 2711]
2. F has -1 anionic charge and highly electronegative [Option ID = 2709]
3. Spin-orbit coupling in Cr
2+
[Option ID = 2712]
4. The Jahn-Teller effect [Option ID = 2710]
Correct Answer :-
The Jahn-Teller effect [Option ID = 2710]
[Question ID = 723]
1. [Option ID = 2889]
52)
53)
2. [Option ID = 2890]
3. [Option ID = 2891]
4. [Option ID = 2892]
Correct Answer :-
[Option ID = 2889]
[Question ID = 727]
1. [Option ID = 2907]
2. [Option ID = 2906]
3. [Option ID = 2905]
4. [Option ID = 2908]
Correct Answer :-
[Option ID = 2905]
[Question ID = 707]
54)
55)
56)
1. N9 [Option ID = 2828]
2. N7 [Option ID = 2827]
3. N1 [Option ID = 2825]
4. N3 [Option ID = 2826]
Correct Answer :-
N7 [Option ID = 2827]
[Question ID = 672]
1. Bicapped prism [Option ID = 2688]
2. Square pyramidal [Option ID = 2686]
3. Trigonal planar [Option ID = 2685]
4. Bent [Option ID = 2687]
Correct Answer :-
Square pyramidal [Option ID = 2686]
[Question ID = 704]
1. [Option ID = 2816]
2. [Option ID = 2814]
3. [Option ID = 2813]
4. [Option ID = 2815]
Correct Answer :-
[Option ID = 2813]
57)
58)
[Question ID = 708]
1. [Option ID = 2829]
2. [Option ID = 2832]
3. [Option ID = 2831]
4. [Option ID = 2830]
Correct Answer :-
[Option ID = 2831]
[Question ID = 717]
1. [Option ID = 2866]
2. [Option ID = 2865]
3. [Option ID = 2867]
4. [Option ID = 2868]
Correct Answer :-
[Option ID = 2866]
59)
[Question ID = 711]
1. [Option ID = 2843]
2. [Option ID = 2842]
3. [Option ID = 2844]
4. [Option ID = 2841]
Correct Answer :-
[Option ID = 2842]
[Question ID = 710]
1. [Option ID = 2838]
2. [Option ID = 2839]
3. [Option ID = 2837]
4. [Option ID = 2840]
Correct Answer :-
60)
61)
62)
[Option ID = 2838]
[Question ID = 682]
1. 0.30 M [Option ID = 2728]
2. 0.25 M [Option ID = 2727]
3. 0.60 M [Option ID = 2725]
4. 0.10 M [Option ID = 2726]
Correct Answer :-
0.30 M [Option ID = 2728]
[Question ID = 772]
1. [Option ID = 3087]
2. [Option ID = 3088]
3. [Option ID = 3086]
4. [Option ID = 3085]
Correct Answer :-
[Option ID = 3086]
[Question ID = 733]
1. [Option ID = 2932]
63)
64)
65)
2. [Option ID = 2929]
3. [Option ID = 2930]
4. [Option ID = 2931]
Correct Answer :-
[Option ID = 2929]
[Question ID = 767]
1. 120 [Option ID = 3065]
2. 280 [Option ID = 3066]
3. 300 [Option ID = 3068]
4. 20 [Option ID = 3067]
Correct Answer :-
20 [Option ID = 3067]
[Question ID = 734]
1. [Option ID = 2936]
2. [Option ID = 2933]
3. [Option ID = 2934]
4. [Option ID = 2935]
Correct Answer :-
[Option ID = 2935]
[Question ID = 676]
66)
67)
68)
1. [Option ID = 2702]
2. [Option ID = 2703]
3. [Option ID = 2704]
4. [Option ID = 2701]
Correct Answer :-
[Option ID = 2703]
[Question ID = 729]
1. (2Z, 4Z)-3, 4-dibromo hepta-2, 4-diene [Option ID = 2914]
2. (2E, 4E)-3, 4-dibromo hepta-2, 4-diene [Option ID = 2915]
3. (2E, 4Z)-3, 4-dibromo hepta-2, 4-diene [Option ID = 2916]
4. (2Z, 4E)-3, 4-dibromo hepta-2, 4-diene [Option ID = 2913]
Correct Answer :-
(2E, 4Z)-3, 4-dibromo hepta-2, 4-diene [Option ID = 2916]
[Question ID = 765]
1. [Option ID = 3060]
2. [Option ID = 3057]
3. [Option ID = 3058]
4. [Option ID = 3059]
Correct Answer :-
[Option ID = 3059]
[Question ID = 724]
1. [Option ID = 2893]
69)
2. [Option ID = 2895]
3. [Option ID = 2896]
4. [Option ID = 2894]
Correct Answer :-
[Option ID = 2893]
[Question ID = 721]
1. [Option ID = 2881]
2. [Option ID = 2884]
3. [Option ID = 2882]
4. [Option ID = 2883]
Correct Answer :-
[Option ID = 2882]
70)
71)
72)
73)
[Question ID = 766]
1. 512 times [Option ID = 3064]
2. 256 times [Option ID = 3063]
3. 128 times [Option ID = 3062]
4. 64 times [Option ID = 3061]
Correct Answer :-
512 times [Option ID = 3064]
[Question ID = 670]
1. 2 [Option ID = 2678]
2. 5 [Option ID = 2680]
3. 1 [Option ID = 2677]
4. 3 [Option ID = 2679]
Correct Answer :-
5 [Option ID = 2680]
[Question ID = 731]
1. [Option ID = 2924]
2. [Option ID = 2923]
3. [Option ID = 2922]
4. [Option ID = 2921]
Correct Answer :-
[Option ID = 2922]
[Question ID = 690]
1. [Option ID = 2760]
2. [Option ID = 2759]
3. [Option ID = 2757]
4. [Option ID = 2758]
74)
75)
Correct Answer :-
[Option ID = 2758]
[Question ID = 720]
1. [Option ID = 2879]
2. [Option ID = 2878]
3. [Option ID = 2880]
4. [Option ID = 2877]
Correct Answer :-
[Option ID = 2877]
[Question ID = 722]
1. [Option ID = 2886]
2. [Option ID = 2888]
3. [Option ID = 2885]
4. [Option ID = 2887]
Correct Answer :-
FirstRanker.com - FirstRanker's Choice
1)
2)
3)
4)
5)
DU PhD in Chemistry
Topic:- DU_J18_PHD_CHEM
Which of the following statements about sulfur dioxide is true?
[Question ID = 677]
1. It forms a S-S dimer in condensed phase [Option ID = 2707]
2. Its anhydride of sulfuric acid [Option ID = 2706]
3. Its O-S-O angle is 180
0
[Option ID = 2708]
4. It is a product of the combustion of fossil fuels that contain sulfur [Option ID = 2705]
Correct Answer :-
It is a product of the combustion of fossil fuels that contain sulfur [Option ID = 2705]
Which of the following is a strong acid in pure liquid HF
[Question ID = 683]
1. H
2
O [Option ID = 2731]
2. NaF [Option ID = 2729]
3. CH
3
COOH [Option ID = 2730]
4. SbF
5
[Option ID = 2732]
Correct Answer :-
SbF
5
[Option ID = 2732]
Each of the following molecules can act as a chelating agent EXCEPT [Question ID = 679]
1. [Option ID = 2716]
2. [Option ID = 2714]
3. [Option ID = 2713]
4. [Option ID = 2715]
Correct Answer :-
[Option ID = 2714]
What is correct about h-index?
[Question ID = 758]
1. Alternative of impact factor [Option ID = 3031]
2. Based on most quoted papers [Option ID = 3030]
3. Quantify scientific productivity [Option ID = 3029]
4. All of these [Option ID = 3032]
Correct Answer :-
All of these [Option ID = 3032]
The hyperfine electron spin resonance (e.s.r.) spectrum of the benzene radical has how many lines? [Question ID = 748]
1. 12 [Option ID = 2992]
2. 7 [Option ID = 2990]
3. 1 [Option ID = 2991]
4. 6 [Option ID = 2989]
Correct Answer :-
7 [Option ID = 2990]
6)
7)
8)
9)
10)
11)
12)
The energy changes involving the core electrons of an atom or molecule are expressed in which region of the electromagnetic
spectrum? [Question ID = 742]
1. Ultraviolet and Visible region [Option ID = 2967]
2. X-ray region [Option ID = 2968]
3. Radiofrequency region [Option ID = 2966]
4. Infra-red region [Option ID = 2965]
Correct Answer :-
X-ray region [Option ID = 2968]
Find out the expected intensity ratio of M and M+1 signal for the Naphthalene molecular ion [Question ID = 726]
1. 99:1.1 [Option ID = 2903]
2. 1.1:99 [Option ID = 2904]
3. 9:01 [Option ID = 2901]
4. 1:9 [Option ID = 2902]
Correct Answer :-
9:01 [Option ID = 2901]
Cobalt-60 is used in radiation therapy of cancer and can be produced by the bombardment of Cobalt-59 with [Question ID = 692]
1. Alpha particles [Option ID = 2765]
2. Beta particles [Option ID = 2767]
3. Neutrons [Option ID = 2766]
4. Gamma rays [Option ID = 2768]
Correct Answer :-
Neutrons [Option ID = 2766]
The standard emf of galvanic cell involving 3 moles of electrons in its redox reaction is 0.59 V. The equilibrium constant for the
reaction of the cell is- [Question ID = 763]
1. [Option ID = 3051]
2. [Option ID = 3052]
3. [Option ID = 3049]
4. [Option ID = 3050]
Correct Answer :-
[Option ID = 3052]
A characteristic common to polymers that can be made to conduct electricity such as polyacetylene, polypyrrole is: [Question ID =
685]
1. Conjugation throughout the polymeric chain. [Option ID = 2740]
2. A high degree of cross linking [Option ID = 2738]
3. A very low glass transition temperature [Option ID = 2737]
4. Presence of stereogenic centers of the same configuration [Option ID = 2739]
Correct Answer :-
Conjugation throughout the polymeric chain. [Option ID = 2740]
Impact factor is [Question ID = 768]
1. Ratio between citations and recent citable items publish [Option ID = 3071]
2. All of these [Option ID = 3072]
3. Addition of citations and recent citable items publish [Option ID = 3069]
4. Ratio between recent citable items publish and citations [Option ID = 3070]
Correct Answer :-
Ratio between recent citable items publish and citations [Option ID = 3070]
On the basis of oxidation-reduction potential, which of the following is most likely to occur? [Question ID = 693]
13)
14)
15)
16)
17)
1. [Option ID = 2770]
2. [Option ID = 2772]
3. [Option ID = 2771]
4. [Option ID = 2769]
Correct Answer :-
[Option ID = 2769]
How many diastereoisomers are possible for the compound 2, 4 ?diphenylcyclobutane-1, 3 di carboxylic acids. [Question ID = 725]
1. 6 [Option ID = 2899]
2. 5 [Option ID = 2898]
3. 8 [Option ID = 2900]
4. 4 [Option ID = 2897]
Correct Answer :-
5 [Option ID = 2898]
An increase in equivalent conductance of a strong electrolyte with dilution is mainly due to- [Question ID = 764]
1. increase in ionic mobility of ions [Option ID = 3055]
2. increase in number of ions [Option ID = 3054]
3. 100% ionization of electrolyte at normal dilution [Option ID = 3056]
4. increase in both i.e. number of ions and ionic mobility of ions. [Option ID = 3053]
Correct Answer :-
increase in ionic mobility of ions [Option ID = 3055]
The solid state structures of the principal allotropes of elemental boron are made up of which of the following structural units
[Question ID = 699]
1. [Option ID = 2796]
2. [Option ID = 2795]
3. [Option ID = 2794]
4. [Option ID = 2793]
Correct Answer :-
[Option ID = 2793]
The molecular geometry of thionyl chloride is best described as [Question ID = 688]
1. T-shaped [Option ID = 2752]
2. Tetrahedral [Option ID = 2751]
3. Trigonal pyramidal [Option ID = 2749]
4. Trigonal planar [Option ID = 2750]
Correct Answer :-
Trigonal pyramidal [Option ID = 2749]
In a face-center cubic (FCC) type of crystal lattice, the number of atoms belonging exclusively to each unit cell within the lattice
is/are: [Question ID = 754]
1. 2 [Option ID = 3014]
2. 1 [Option ID = 3013]
3. 3 [Option ID = 3015]
4. 4 [Option ID = 3016]
Correct Answer :-
4 [Option ID = 3016]
18)
19)
20)
21)
22)
23)
24)
Among the following, the weakest oxidizing agent is [Question ID = 675]
1. Mg (s) [Option ID = 2698]
2. [Option ID = 2699]
3. [Option ID = 2700]
4. [Option ID = 2697]
Correct Answer :-
Mg (s) [Option ID = 2698]
For a polymer, which of the following statement/s is/are true? [Question ID = 759]
1. Weight average molecular weight is almost always higher than the number average molecular weight [Option ID = 3035]
2. Formation of a polypeptide from its monomers (amino acids) is an example of addition polymerization [Option ID = 3034]
3. All of these [Option ID = 3036]
4. Vinyl polymerization is an example of condensation polymerization. [Option ID = 3033]
Correct Answer :-
Weight average molecular weight is almost always higher than the number average molecular weight [Option ID = 3035]
Quantum dots are [Question ID = 762]
1. Three dimensional [Option ID = 3048]
2. One dimensional [Option ID = 3046]
3. Two dimensional [Option ID = 3047]
4. Zero dimensional [Option ID = 3045]
Correct Answer :-
Zero dimensional [Option ID = 3045]
The unit of rate constant for a third order reaction is: [Question ID = 749]
1. [Option ID = 2993]
2. [Option ID = 2995]
3. [Option ID = 2996]
4. [Option ID = 2994]
Correct Answer :-
[Option ID = 2996]
All the following elements have at least one isotope that is not radioactive EXCEPT [Question ID = 673]
1. Pb [Option ID = 2690]
2. O [Option ID = 2689]
3. Sn [Option ID = 2691]
4. No [Option ID = 2692]
Correct Answer :-
No [Option ID = 2692]
The conditions for a species to follow Bose-Einstein statistics are; [Question ID = 736]
1. Particles are indistinguishable, with no restriction on filling up of energy levels [Option ID = 2944]
2. Particles are indistinguishable, with a restriction on filling up of energy levels [Option ID = 2943]
3. Particles are distinguishable, with a restriction on filling up of energy levels [Option ID = 2941]
4. Particles are distinguishable, with no restriction on filling up of energy levels [Option ID = 2942]
Correct Answer :-
Particles are indistinguishable, with no restriction on filling up of energy levels [Option ID = 2944]
In the kinetic theory of collisions, the SI unit of collision number, in terms of m (meter) and s (second), is:
[Question ID = 761]
25)
26)
27)
28)
29)
1. m-
2
s
-1
[Option ID = 3042]
2. m
4
s
-1
[Option ID = 3041]
3. m
2
s
-1
[Option ID = 3043]
4. None of these [Option ID = 3044]
Correct Answer :-
None of these [Option ID = 3044]
Correct characteristics of the functional groups of adenine in DNA base pair are [Question ID = 706]
1. [Option ID = 2824]
2. [Option ID = 2821]
3. [Option ID = 2823]
4. [Option ID = 2822]
Correct Answer :-
[Option ID = 2822]
The carbon monoxide molecule has an internuclear distance of 1.13 Angstroms. What is the moment of Inertia of this molecule?
[Question ID = 740]
1. [Option ID = 2960]
2. [Option ID = 2957]
3. [Option ID = 2958]
4. [Option ID = 2959]
Correct Answer :-
[Option ID = 2957]
Which of the following represent/s non-linear optical technique? [Question ID = 744]
1. Second Harmonic generation [Option ID = 2974]
2. Two-photon photoluminescence [Option ID = 2975]
3. Four-wave mixing [Option ID = 2973]
4. All of these [Option ID = 2976]
Correct Answer :-
All of these [Option ID = 2976]
. Which of the following does not affect the intensity of spectral lines of a sample? [Question ID = 743]
1. Path length of a sample [Option ID = 2972]
2. Population of energy states [Option ID = 2970]
3. Heisenberg?s Uncertainty principle [Option ID = 2971]
4. Concentration of a sample [Option ID = 2969]
Correct Answer :-
Heisenberg?s Uncertainty principle [Option ID = 2971]
[Question ID = 716]
30)
31)
1. [Option ID = 2862]
2. [Option ID = 2863]
3. [Option ID = 2861]
4. [Option ID = 2864]
Correct Answer :-
[Option ID = 2864]
[Question ID = 712]
1. [Option ID = 2845]
2. [Option ID = 2846]
3. [Option ID = 2848]
4. [Option ID = 2847]
Correct Answer :-
[Option ID = 2847]
[Question ID = 686]
1. [Option ID = 2743]
2. [Option ID = 2742]
3. [Option ID = 2741]
32)
33)
4. [Option ID = 2744]
Correct Answer :-
[Option ID = 2743]
[Question ID = 5633]
1. [Option ID = 22523]
2. [Option ID = 22524]
3. [Option ID = 22526]
4. [Option ID = 22525]
Correct Answer :-
[Option ID = 22523]
[Question ID = 730]
1. [Option ID = 2918]
2. [Option ID = 2920]
3. [Option ID = 2917]
4. [Option ID = 2919]
Correct Answer :-
34)
35)
36)
[Option ID = 2919]
[Question ID = 714]
1. [Option ID = 2856]
2. [Option ID = 2853]
3. [Option ID = 2855]
4. [Option ID = 2854]
Correct Answer :-
[Option ID = 2854]
[Question ID = 669]
1. 2 [Option ID = 2673]
2. 18 [Option ID = 2676]
3. 5 [Option ID = 2674]
4. 10 [Option ID = 2675]
Correct Answer :-
5 [Option ID = 2674]
[Question ID = 689]
1. 2 [Option ID = 2754]
2. 0 [Option ID = 2756]
3. 1 [Option ID = 2755]
37)
38)
4. 3 [Option ID = 2753]
Correct Answer :-
1 [Option ID = 2755]
[Question ID = 718]
1. C) [Option ID = 2871]
2. B) [Option ID = 2870]
3. A) [Option ID = 2869]
4. D) both (B) and (C) [Option ID = 2872]
Correct Answer :-
A) [Option ID = 2869]
[Question ID = 709]
1. [Option ID = 2836]
2. [Option ID = 2833]
3. [Option ID = 2835]
4. [Option ID = 2834]
39)
40)
41)
42)
Correct Answer :-
[Option ID = 2836]
[Question ID = 705]
1. (2Z, 4Z)-3-chlorohexa-2, 4-diene-1,6-diol. [Option ID = 2819]
2. (2E, 4E)-3-chlorohexa-2, 4-diene-1,6-diol. [Option ID = 2817]
3. (2Z, 4E)-3-chlorohexa-2, 4-diene-1,6-diol. [Option ID = 2818]
4. (2Z, 4E)-3-chlorohexa-2, 4-diene-1,6-diol. [Option ID = 2820]
Correct Answer :-
(2Z, 4E)-3-chlorohexa-2, 4-diene-1,6-diol. [Option ID = 2820]
[Question ID = 680]
1. The presence of other complexing ligands in solution affects the equilibrium concentration of metal-EDTA complexes [Option ID = 2719]
2. Metal-EDTA complexes have an equilibrium concentration independent of pH [Option ID = 2718]
3. Metal-EDTA complexes are often 2:1 in stoichiometry [Option ID = 2717]
4. Metal-EDTA complexes are less stable than the corresponding metal-ammine complexes [Option ID = 2720]
Correct Answer :-
The presence of other complexing ligands in solution affects the equilibrium concentration of metal-EDTA complexes [Option ID = 2719]
[Question ID = 671]
1. 150 g [Option ID = 2682]
2. 300 g [Option ID = 2684]
3. 120 g [Option ID = 2681]
4. 180 g [Option ID = 2683]
Correct Answer :-
300 g [Option ID = 2684]
[Question ID = 756]
1. [Option ID = 3021]
2. [Option ID = 3022]
3. [Option ID = 3024]
4. [Option ID = 3023]
Correct Answer :-
43)
44)
45)
[Option ID = 3024]
[Question ID = 750]
1. [Option ID = 2999]
2. [Option ID = 3000]
3. [Option ID = 2997]
4. [Option ID = 2998]
Correct Answer :-
[Option ID = 2997]
[Question ID = 715]
1. [Option ID = 2857]
2. [Option ID = 2860]
3. [Option ID = 2858]
4. [Option ID = 2859]
Correct Answer :-
[Option ID = 2860]
46)
47)
[Question ID = 719]
1. [Option ID = 2873]
2. [Option ID = 2876]
3. [Option ID = 2874]
4. [Option ID = 2875]
Correct Answer :-
[Question ID = 674]
1. Evaporating some water to decrease the volume of the solution. [Option ID = 2696]
2. [Option ID = 2695]
3. [Option ID = 2694]
4. [Option ID = 2693]
Correct Answer :-
[Option ID = 2693]
[Question ID = 728]
1. [Option ID = 2910]
2. [Option ID = 2911]
3. [Option ID = 2909]
4. [Option ID = 2912]
48)
49)
50)
51)
Correct Answer :-
[Option ID = 2912]
[Question ID = 697]
1. III only [Option ID = 2788]
2. II only [Option ID = 2787]
3. I only [Option ID = 2785]
4. I, II and III [Option ID = 2786]
Correct Answer :-
I, II and III [Option ID = 2786]
[Question ID = 700]
1. [Option ID = 2798]
2. [Option ID = 2797]
3. [Option ID = 2800]
4. [Option ID = 2799]
Correct Answer :-
[Option ID = 2799]
[Question ID = 678]
1. Cr
2+
has a low cationic charge [Option ID = 2711]
2. F has -1 anionic charge and highly electronegative [Option ID = 2709]
3. Spin-orbit coupling in Cr
2+
[Option ID = 2712]
4. The Jahn-Teller effect [Option ID = 2710]
Correct Answer :-
The Jahn-Teller effect [Option ID = 2710]
[Question ID = 723]
1. [Option ID = 2889]
52)
53)
2. [Option ID = 2890]
3. [Option ID = 2891]
4. [Option ID = 2892]
Correct Answer :-
[Option ID = 2889]
[Question ID = 727]
1. [Option ID = 2907]
2. [Option ID = 2906]
3. [Option ID = 2905]
4. [Option ID = 2908]
Correct Answer :-
[Option ID = 2905]
[Question ID = 707]
54)
55)
56)
1. N9 [Option ID = 2828]
2. N7 [Option ID = 2827]
3. N1 [Option ID = 2825]
4. N3 [Option ID = 2826]
Correct Answer :-
N7 [Option ID = 2827]
[Question ID = 672]
1. Bicapped prism [Option ID = 2688]
2. Square pyramidal [Option ID = 2686]
3. Trigonal planar [Option ID = 2685]
4. Bent [Option ID = 2687]
Correct Answer :-
Square pyramidal [Option ID = 2686]
[Question ID = 704]
1. [Option ID = 2816]
2. [Option ID = 2814]
3. [Option ID = 2813]
4. [Option ID = 2815]
Correct Answer :-
[Option ID = 2813]
57)
58)
[Question ID = 708]
1. [Option ID = 2829]
2. [Option ID = 2832]
3. [Option ID = 2831]
4. [Option ID = 2830]
Correct Answer :-
[Option ID = 2831]
[Question ID = 717]
1. [Option ID = 2866]
2. [Option ID = 2865]
3. [Option ID = 2867]
4. [Option ID = 2868]
Correct Answer :-
[Option ID = 2866]
59)
[Question ID = 711]
1. [Option ID = 2843]
2. [Option ID = 2842]
3. [Option ID = 2844]
4. [Option ID = 2841]
Correct Answer :-
[Option ID = 2842]
[Question ID = 710]
1. [Option ID = 2838]
2. [Option ID = 2839]
3. [Option ID = 2837]
4. [Option ID = 2840]
Correct Answer :-
60)
61)
62)
[Option ID = 2838]
[Question ID = 682]
1. 0.30 M [Option ID = 2728]
2. 0.25 M [Option ID = 2727]
3. 0.60 M [Option ID = 2725]
4. 0.10 M [Option ID = 2726]
Correct Answer :-
0.30 M [Option ID = 2728]
[Question ID = 772]
1. [Option ID = 3087]
2. [Option ID = 3088]
3. [Option ID = 3086]
4. [Option ID = 3085]
Correct Answer :-
[Option ID = 3086]
[Question ID = 733]
1. [Option ID = 2932]
63)
64)
65)
2. [Option ID = 2929]
3. [Option ID = 2930]
4. [Option ID = 2931]
Correct Answer :-
[Option ID = 2929]
[Question ID = 767]
1. 120 [Option ID = 3065]
2. 280 [Option ID = 3066]
3. 300 [Option ID = 3068]
4. 20 [Option ID = 3067]
Correct Answer :-
20 [Option ID = 3067]
[Question ID = 734]
1. [Option ID = 2936]
2. [Option ID = 2933]
3. [Option ID = 2934]
4. [Option ID = 2935]
Correct Answer :-
[Option ID = 2935]
[Question ID = 676]
66)
67)
68)
1. [Option ID = 2702]
2. [Option ID = 2703]
3. [Option ID = 2704]
4. [Option ID = 2701]
Correct Answer :-
[Option ID = 2703]
[Question ID = 729]
1. (2Z, 4Z)-3, 4-dibromo hepta-2, 4-diene [Option ID = 2914]
2. (2E, 4E)-3, 4-dibromo hepta-2, 4-diene [Option ID = 2915]
3. (2E, 4Z)-3, 4-dibromo hepta-2, 4-diene [Option ID = 2916]
4. (2Z, 4E)-3, 4-dibromo hepta-2, 4-diene [Option ID = 2913]
Correct Answer :-
(2E, 4Z)-3, 4-dibromo hepta-2, 4-diene [Option ID = 2916]
[Question ID = 765]
1. [Option ID = 3060]
2. [Option ID = 3057]
3. [Option ID = 3058]
4. [Option ID = 3059]
Correct Answer :-
[Option ID = 3059]
[Question ID = 724]
1. [Option ID = 2893]
69)
2. [Option ID = 2895]
3. [Option ID = 2896]
4. [Option ID = 2894]
Correct Answer :-
[Option ID = 2893]
[Question ID = 721]
1. [Option ID = 2881]
2. [Option ID = 2884]
3. [Option ID = 2882]
4. [Option ID = 2883]
Correct Answer :-
[Option ID = 2882]
70)
71)
72)
73)
[Question ID = 766]
1. 512 times [Option ID = 3064]
2. 256 times [Option ID = 3063]
3. 128 times [Option ID = 3062]
4. 64 times [Option ID = 3061]
Correct Answer :-
512 times [Option ID = 3064]
[Question ID = 670]
1. 2 [Option ID = 2678]
2. 5 [Option ID = 2680]
3. 1 [Option ID = 2677]
4. 3 [Option ID = 2679]
Correct Answer :-
5 [Option ID = 2680]
[Question ID = 731]
1. [Option ID = 2924]
2. [Option ID = 2923]
3. [Option ID = 2922]
4. [Option ID = 2921]
Correct Answer :-
[Option ID = 2922]
[Question ID = 690]
1. [Option ID = 2760]
2. [Option ID = 2759]
3. [Option ID = 2757]
4. [Option ID = 2758]
74)
75)
Correct Answer :-
[Option ID = 2758]
[Question ID = 720]
1. [Option ID = 2879]
2. [Option ID = 2878]
3. [Option ID = 2880]
4. [Option ID = 2877]
Correct Answer :-
[Option ID = 2877]
[Question ID = 722]
1. [Option ID = 2886]
2. [Option ID = 2888]
3. [Option ID = 2885]
4. [Option ID = 2887]
Correct Answer :-
76)
77)
78)
[Option ID = 2887]
[Question ID = 755]
1. [Option ID = 3019]
2. [Option ID = 3018]
3. [Option ID = 3020]
4. [Option ID = 3017]
Correct Answer :-
[Option ID = 3019]
[Question ID = 695]
1. Ba [Option ID = 2779]
2. BaO [Option ID = 2780]
3. BaOH [Option ID = 2778]
4. BaH
2
[Option ID = 2777]
Correct Answer :-
BaO [Option ID = 2780]
[Question ID = 703]
1. [Option ID = 2811]
2. [Option ID = 2809]
3. [Option ID = 2812]
4. [Option ID = 2810]
Correct Answer :-
[Option ID = 2811]
FirstRanker.com - FirstRanker's Choice
1)
2)
3)
4)
5)
DU PhD in Chemistry
Topic:- DU_J18_PHD_CHEM
Which of the following statements about sulfur dioxide is true?
[Question ID = 677]
1. It forms a S-S dimer in condensed phase [Option ID = 2707]
2. Its anhydride of sulfuric acid [Option ID = 2706]
3. Its O-S-O angle is 180
0
[Option ID = 2708]
4. It is a product of the combustion of fossil fuels that contain sulfur [Option ID = 2705]
Correct Answer :-
It is a product of the combustion of fossil fuels that contain sulfur [Option ID = 2705]
Which of the following is a strong acid in pure liquid HF
[Question ID = 683]
1. H
2
O [Option ID = 2731]
2. NaF [Option ID = 2729]
3. CH
3
COOH [Option ID = 2730]
4. SbF
5
[Option ID = 2732]
Correct Answer :-
SbF
5
[Option ID = 2732]
Each of the following molecules can act as a chelating agent EXCEPT [Question ID = 679]
1. [Option ID = 2716]
2. [Option ID = 2714]
3. [Option ID = 2713]
4. [Option ID = 2715]
Correct Answer :-
[Option ID = 2714]
What is correct about h-index?
[Question ID = 758]
1. Alternative of impact factor [Option ID = 3031]
2. Based on most quoted papers [Option ID = 3030]
3. Quantify scientific productivity [Option ID = 3029]
4. All of these [Option ID = 3032]
Correct Answer :-
All of these [Option ID = 3032]
The hyperfine electron spin resonance (e.s.r.) spectrum of the benzene radical has how many lines? [Question ID = 748]
1. 12 [Option ID = 2992]
2. 7 [Option ID = 2990]
3. 1 [Option ID = 2991]
4. 6 [Option ID = 2989]
Correct Answer :-
7 [Option ID = 2990]
6)
7)
8)
9)
10)
11)
12)
The energy changes involving the core electrons of an atom or molecule are expressed in which region of the electromagnetic
spectrum? [Question ID = 742]
1. Ultraviolet and Visible region [Option ID = 2967]
2. X-ray region [Option ID = 2968]
3. Radiofrequency region [Option ID = 2966]
4. Infra-red region [Option ID = 2965]
Correct Answer :-
X-ray region [Option ID = 2968]
Find out the expected intensity ratio of M and M+1 signal for the Naphthalene molecular ion [Question ID = 726]
1. 99:1.1 [Option ID = 2903]
2. 1.1:99 [Option ID = 2904]
3. 9:01 [Option ID = 2901]
4. 1:9 [Option ID = 2902]
Correct Answer :-
9:01 [Option ID = 2901]
Cobalt-60 is used in radiation therapy of cancer and can be produced by the bombardment of Cobalt-59 with [Question ID = 692]
1. Alpha particles [Option ID = 2765]
2. Beta particles [Option ID = 2767]
3. Neutrons [Option ID = 2766]
4. Gamma rays [Option ID = 2768]
Correct Answer :-
Neutrons [Option ID = 2766]
The standard emf of galvanic cell involving 3 moles of electrons in its redox reaction is 0.59 V. The equilibrium constant for the
reaction of the cell is- [Question ID = 763]
1. [Option ID = 3051]
2. [Option ID = 3052]
3. [Option ID = 3049]
4. [Option ID = 3050]
Correct Answer :-
[Option ID = 3052]
A characteristic common to polymers that can be made to conduct electricity such as polyacetylene, polypyrrole is: [Question ID =
685]
1. Conjugation throughout the polymeric chain. [Option ID = 2740]
2. A high degree of cross linking [Option ID = 2738]
3. A very low glass transition temperature [Option ID = 2737]
4. Presence of stereogenic centers of the same configuration [Option ID = 2739]
Correct Answer :-
Conjugation throughout the polymeric chain. [Option ID = 2740]
Impact factor is [Question ID = 768]
1. Ratio between citations and recent citable items publish [Option ID = 3071]
2. All of these [Option ID = 3072]
3. Addition of citations and recent citable items publish [Option ID = 3069]
4. Ratio between recent citable items publish and citations [Option ID = 3070]
Correct Answer :-
Ratio between recent citable items publish and citations [Option ID = 3070]
On the basis of oxidation-reduction potential, which of the following is most likely to occur? [Question ID = 693]
13)
14)
15)
16)
17)
1. [Option ID = 2770]
2. [Option ID = 2772]
3. [Option ID = 2771]
4. [Option ID = 2769]
Correct Answer :-
[Option ID = 2769]
How many diastereoisomers are possible for the compound 2, 4 ?diphenylcyclobutane-1, 3 di carboxylic acids. [Question ID = 725]
1. 6 [Option ID = 2899]
2. 5 [Option ID = 2898]
3. 8 [Option ID = 2900]
4. 4 [Option ID = 2897]
Correct Answer :-
5 [Option ID = 2898]
An increase in equivalent conductance of a strong electrolyte with dilution is mainly due to- [Question ID = 764]
1. increase in ionic mobility of ions [Option ID = 3055]
2. increase in number of ions [Option ID = 3054]
3. 100% ionization of electrolyte at normal dilution [Option ID = 3056]
4. increase in both i.e. number of ions and ionic mobility of ions. [Option ID = 3053]
Correct Answer :-
increase in ionic mobility of ions [Option ID = 3055]
The solid state structures of the principal allotropes of elemental boron are made up of which of the following structural units
[Question ID = 699]
1. [Option ID = 2796]
2. [Option ID = 2795]
3. [Option ID = 2794]
4. [Option ID = 2793]
Correct Answer :-
[Option ID = 2793]
The molecular geometry of thionyl chloride is best described as [Question ID = 688]
1. T-shaped [Option ID = 2752]
2. Tetrahedral [Option ID = 2751]
3. Trigonal pyramidal [Option ID = 2749]
4. Trigonal planar [Option ID = 2750]
Correct Answer :-
Trigonal pyramidal [Option ID = 2749]
In a face-center cubic (FCC) type of crystal lattice, the number of atoms belonging exclusively to each unit cell within the lattice
is/are: [Question ID = 754]
1. 2 [Option ID = 3014]
2. 1 [Option ID = 3013]
3. 3 [Option ID = 3015]
4. 4 [Option ID = 3016]
Correct Answer :-
4 [Option ID = 3016]
18)
19)
20)
21)
22)
23)
24)
Among the following, the weakest oxidizing agent is [Question ID = 675]
1. Mg (s) [Option ID = 2698]
2. [Option ID = 2699]
3. [Option ID = 2700]
4. [Option ID = 2697]
Correct Answer :-
Mg (s) [Option ID = 2698]
For a polymer, which of the following statement/s is/are true? [Question ID = 759]
1. Weight average molecular weight is almost always higher than the number average molecular weight [Option ID = 3035]
2. Formation of a polypeptide from its monomers (amino acids) is an example of addition polymerization [Option ID = 3034]
3. All of these [Option ID = 3036]
4. Vinyl polymerization is an example of condensation polymerization. [Option ID = 3033]
Correct Answer :-
Weight average molecular weight is almost always higher than the number average molecular weight [Option ID = 3035]
Quantum dots are [Question ID = 762]
1. Three dimensional [Option ID = 3048]
2. One dimensional [Option ID = 3046]
3. Two dimensional [Option ID = 3047]
4. Zero dimensional [Option ID = 3045]
Correct Answer :-
Zero dimensional [Option ID = 3045]
The unit of rate constant for a third order reaction is: [Question ID = 749]
1. [Option ID = 2993]
2. [Option ID = 2995]
3. [Option ID = 2996]
4. [Option ID = 2994]
Correct Answer :-
[Option ID = 2996]
All the following elements have at least one isotope that is not radioactive EXCEPT [Question ID = 673]
1. Pb [Option ID = 2690]
2. O [Option ID = 2689]
3. Sn [Option ID = 2691]
4. No [Option ID = 2692]
Correct Answer :-
No [Option ID = 2692]
The conditions for a species to follow Bose-Einstein statistics are; [Question ID = 736]
1. Particles are indistinguishable, with no restriction on filling up of energy levels [Option ID = 2944]
2. Particles are indistinguishable, with a restriction on filling up of energy levels [Option ID = 2943]
3. Particles are distinguishable, with a restriction on filling up of energy levels [Option ID = 2941]
4. Particles are distinguishable, with no restriction on filling up of energy levels [Option ID = 2942]
Correct Answer :-
Particles are indistinguishable, with no restriction on filling up of energy levels [Option ID = 2944]
In the kinetic theory of collisions, the SI unit of collision number, in terms of m (meter) and s (second), is:
[Question ID = 761]
25)
26)
27)
28)
29)
1. m-
2
s
-1
[Option ID = 3042]
2. m
4
s
-1
[Option ID = 3041]
3. m
2
s
-1
[Option ID = 3043]
4. None of these [Option ID = 3044]
Correct Answer :-
None of these [Option ID = 3044]
Correct characteristics of the functional groups of adenine in DNA base pair are [Question ID = 706]
1. [Option ID = 2824]
2. [Option ID = 2821]
3. [Option ID = 2823]
4. [Option ID = 2822]
Correct Answer :-
[Option ID = 2822]
The carbon monoxide molecule has an internuclear distance of 1.13 Angstroms. What is the moment of Inertia of this molecule?
[Question ID = 740]
1. [Option ID = 2960]
2. [Option ID = 2957]
3. [Option ID = 2958]
4. [Option ID = 2959]
Correct Answer :-
[Option ID = 2957]
Which of the following represent/s non-linear optical technique? [Question ID = 744]
1. Second Harmonic generation [Option ID = 2974]
2. Two-photon photoluminescence [Option ID = 2975]
3. Four-wave mixing [Option ID = 2973]
4. All of these [Option ID = 2976]
Correct Answer :-
All of these [Option ID = 2976]
. Which of the following does not affect the intensity of spectral lines of a sample? [Question ID = 743]
1. Path length of a sample [Option ID = 2972]
2. Population of energy states [Option ID = 2970]
3. Heisenberg?s Uncertainty principle [Option ID = 2971]
4. Concentration of a sample [Option ID = 2969]
Correct Answer :-
Heisenberg?s Uncertainty principle [Option ID = 2971]
[Question ID = 716]
30)
31)
1. [Option ID = 2862]
2. [Option ID = 2863]
3. [Option ID = 2861]
4. [Option ID = 2864]
Correct Answer :-
[Option ID = 2864]
[Question ID = 712]
1. [Option ID = 2845]
2. [Option ID = 2846]
3. [Option ID = 2848]
4. [Option ID = 2847]
Correct Answer :-
[Option ID = 2847]
[Question ID = 686]
1. [Option ID = 2743]
2. [Option ID = 2742]
3. [Option ID = 2741]
32)
33)
4. [Option ID = 2744]
Correct Answer :-
[Option ID = 2743]
[Question ID = 5633]
1. [Option ID = 22523]
2. [Option ID = 22524]
3. [Option ID = 22526]
4. [Option ID = 22525]
Correct Answer :-
[Option ID = 22523]
[Question ID = 730]
1. [Option ID = 2918]
2. [Option ID = 2920]
3. [Option ID = 2917]
4. [Option ID = 2919]
Correct Answer :-
34)
35)
36)
[Option ID = 2919]
[Question ID = 714]
1. [Option ID = 2856]
2. [Option ID = 2853]
3. [Option ID = 2855]
4. [Option ID = 2854]
Correct Answer :-
[Option ID = 2854]
[Question ID = 669]
1. 2 [Option ID = 2673]
2. 18 [Option ID = 2676]
3. 5 [Option ID = 2674]
4. 10 [Option ID = 2675]
Correct Answer :-
5 [Option ID = 2674]
[Question ID = 689]
1. 2 [Option ID = 2754]
2. 0 [Option ID = 2756]
3. 1 [Option ID = 2755]
37)
38)
4. 3 [Option ID = 2753]
Correct Answer :-
1 [Option ID = 2755]
[Question ID = 718]
1. C) [Option ID = 2871]
2. B) [Option ID = 2870]
3. A) [Option ID = 2869]
4. D) both (B) and (C) [Option ID = 2872]
Correct Answer :-
A) [Option ID = 2869]
[Question ID = 709]
1. [Option ID = 2836]
2. [Option ID = 2833]
3. [Option ID = 2835]
4. [Option ID = 2834]
39)
40)
41)
42)
Correct Answer :-
[Option ID = 2836]
[Question ID = 705]
1. (2Z, 4Z)-3-chlorohexa-2, 4-diene-1,6-diol. [Option ID = 2819]
2. (2E, 4E)-3-chlorohexa-2, 4-diene-1,6-diol. [Option ID = 2817]
3. (2Z, 4E)-3-chlorohexa-2, 4-diene-1,6-diol. [Option ID = 2818]
4. (2Z, 4E)-3-chlorohexa-2, 4-diene-1,6-diol. [Option ID = 2820]
Correct Answer :-
(2Z, 4E)-3-chlorohexa-2, 4-diene-1,6-diol. [Option ID = 2820]
[Question ID = 680]
1. The presence of other complexing ligands in solution affects the equilibrium concentration of metal-EDTA complexes [Option ID = 2719]
2. Metal-EDTA complexes have an equilibrium concentration independent of pH [Option ID = 2718]
3. Metal-EDTA complexes are often 2:1 in stoichiometry [Option ID = 2717]
4. Metal-EDTA complexes are less stable than the corresponding metal-ammine complexes [Option ID = 2720]
Correct Answer :-
The presence of other complexing ligands in solution affects the equilibrium concentration of metal-EDTA complexes [Option ID = 2719]
[Question ID = 671]
1. 150 g [Option ID = 2682]
2. 300 g [Option ID = 2684]
3. 120 g [Option ID = 2681]
4. 180 g [Option ID = 2683]
Correct Answer :-
300 g [Option ID = 2684]
[Question ID = 756]
1. [Option ID = 3021]
2. [Option ID = 3022]
3. [Option ID = 3024]
4. [Option ID = 3023]
Correct Answer :-
43)
44)
45)
[Option ID = 3024]
[Question ID = 750]
1. [Option ID = 2999]
2. [Option ID = 3000]
3. [Option ID = 2997]
4. [Option ID = 2998]
Correct Answer :-
[Option ID = 2997]
[Question ID = 715]
1. [Option ID = 2857]
2. [Option ID = 2860]
3. [Option ID = 2858]
4. [Option ID = 2859]
Correct Answer :-
[Option ID = 2860]
46)
47)
[Question ID = 719]
1. [Option ID = 2873]
2. [Option ID = 2876]
3. [Option ID = 2874]
4. [Option ID = 2875]
Correct Answer :-
[Question ID = 674]
1. Evaporating some water to decrease the volume of the solution. [Option ID = 2696]
2. [Option ID = 2695]
3. [Option ID = 2694]
4. [Option ID = 2693]
Correct Answer :-
[Option ID = 2693]
[Question ID = 728]
1. [Option ID = 2910]
2. [Option ID = 2911]
3. [Option ID = 2909]
4. [Option ID = 2912]
48)
49)
50)
51)
Correct Answer :-
[Option ID = 2912]
[Question ID = 697]
1. III only [Option ID = 2788]
2. II only [Option ID = 2787]
3. I only [Option ID = 2785]
4. I, II and III [Option ID = 2786]
Correct Answer :-
I, II and III [Option ID = 2786]
[Question ID = 700]
1. [Option ID = 2798]
2. [Option ID = 2797]
3. [Option ID = 2800]
4. [Option ID = 2799]
Correct Answer :-
[Option ID = 2799]
[Question ID = 678]
1. Cr
2+
has a low cationic charge [Option ID = 2711]
2. F has -1 anionic charge and highly electronegative [Option ID = 2709]
3. Spin-orbit coupling in Cr
2+
[Option ID = 2712]
4. The Jahn-Teller effect [Option ID = 2710]
Correct Answer :-
The Jahn-Teller effect [Option ID = 2710]
[Question ID = 723]
1. [Option ID = 2889]
52)
53)
2. [Option ID = 2890]
3. [Option ID = 2891]
4. [Option ID = 2892]
Correct Answer :-
[Option ID = 2889]
[Question ID = 727]
1. [Option ID = 2907]
2. [Option ID = 2906]
3. [Option ID = 2905]
4. [Option ID = 2908]
Correct Answer :-
[Option ID = 2905]
[Question ID = 707]
54)
55)
56)
1. N9 [Option ID = 2828]
2. N7 [Option ID = 2827]
3. N1 [Option ID = 2825]
4. N3 [Option ID = 2826]
Correct Answer :-
N7 [Option ID = 2827]
[Question ID = 672]
1. Bicapped prism [Option ID = 2688]
2. Square pyramidal [Option ID = 2686]
3. Trigonal planar [Option ID = 2685]
4. Bent [Option ID = 2687]
Correct Answer :-
Square pyramidal [Option ID = 2686]
[Question ID = 704]
1. [Option ID = 2816]
2. [Option ID = 2814]
3. [Option ID = 2813]
4. [Option ID = 2815]
Correct Answer :-
[Option ID = 2813]
57)
58)
[Question ID = 708]
1. [Option ID = 2829]
2. [Option ID = 2832]
3. [Option ID = 2831]
4. [Option ID = 2830]
Correct Answer :-
[Option ID = 2831]
[Question ID = 717]
1. [Option ID = 2866]
2. [Option ID = 2865]
3. [Option ID = 2867]
4. [Option ID = 2868]
Correct Answer :-
[Option ID = 2866]
59)
[Question ID = 711]
1. [Option ID = 2843]
2. [Option ID = 2842]
3. [Option ID = 2844]
4. [Option ID = 2841]
Correct Answer :-
[Option ID = 2842]
[Question ID = 710]
1. [Option ID = 2838]
2. [Option ID = 2839]
3. [Option ID = 2837]
4. [Option ID = 2840]
Correct Answer :-
60)
61)
62)
[Option ID = 2838]
[Question ID = 682]
1. 0.30 M [Option ID = 2728]
2. 0.25 M [Option ID = 2727]
3. 0.60 M [Option ID = 2725]
4. 0.10 M [Option ID = 2726]
Correct Answer :-
0.30 M [Option ID = 2728]
[Question ID = 772]
1. [Option ID = 3087]
2. [Option ID = 3088]
3. [Option ID = 3086]
4. [Option ID = 3085]
Correct Answer :-
[Option ID = 3086]
[Question ID = 733]
1. [Option ID = 2932]
63)
64)
65)
2. [Option ID = 2929]
3. [Option ID = 2930]
4. [Option ID = 2931]
Correct Answer :-
[Option ID = 2929]
[Question ID = 767]
1. 120 [Option ID = 3065]
2. 280 [Option ID = 3066]
3. 300 [Option ID = 3068]
4. 20 [Option ID = 3067]
Correct Answer :-
20 [Option ID = 3067]
[Question ID = 734]
1. [Option ID = 2936]
2. [Option ID = 2933]
3. [Option ID = 2934]
4. [Option ID = 2935]
Correct Answer :-
[Option ID = 2935]
[Question ID = 676]
66)
67)
68)
1. [Option ID = 2702]
2. [Option ID = 2703]
3. [Option ID = 2704]
4. [Option ID = 2701]
Correct Answer :-
[Option ID = 2703]
[Question ID = 729]
1. (2Z, 4Z)-3, 4-dibromo hepta-2, 4-diene [Option ID = 2914]
2. (2E, 4E)-3, 4-dibromo hepta-2, 4-diene [Option ID = 2915]
3. (2E, 4Z)-3, 4-dibromo hepta-2, 4-diene [Option ID = 2916]
4. (2Z, 4E)-3, 4-dibromo hepta-2, 4-diene [Option ID = 2913]
Correct Answer :-
(2E, 4Z)-3, 4-dibromo hepta-2, 4-diene [Option ID = 2916]
[Question ID = 765]
1. [Option ID = 3060]
2. [Option ID = 3057]
3. [Option ID = 3058]
4. [Option ID = 3059]
Correct Answer :-
[Option ID = 3059]
[Question ID = 724]
1. [Option ID = 2893]
69)
2. [Option ID = 2895]
3. [Option ID = 2896]
4. [Option ID = 2894]
Correct Answer :-
[Option ID = 2893]
[Question ID = 721]
1. [Option ID = 2881]
2. [Option ID = 2884]
3. [Option ID = 2882]
4. [Option ID = 2883]
Correct Answer :-
[Option ID = 2882]
70)
71)
72)
73)
[Question ID = 766]
1. 512 times [Option ID = 3064]
2. 256 times [Option ID = 3063]
3. 128 times [Option ID = 3062]
4. 64 times [Option ID = 3061]
Correct Answer :-
512 times [Option ID = 3064]
[Question ID = 670]
1. 2 [Option ID = 2678]
2. 5 [Option ID = 2680]
3. 1 [Option ID = 2677]
4. 3 [Option ID = 2679]
Correct Answer :-
5 [Option ID = 2680]
[Question ID = 731]
1. [Option ID = 2924]
2. [Option ID = 2923]
3. [Option ID = 2922]
4. [Option ID = 2921]
Correct Answer :-
[Option ID = 2922]
[Question ID = 690]
1. [Option ID = 2760]
2. [Option ID = 2759]
3. [Option ID = 2757]
4. [Option ID = 2758]
74)
75)
Correct Answer :-
[Option ID = 2758]
[Question ID = 720]
1. [Option ID = 2879]
2. [Option ID = 2878]
3. [Option ID = 2880]
4. [Option ID = 2877]
Correct Answer :-
[Option ID = 2877]
[Question ID = 722]
1. [Option ID = 2886]
2. [Option ID = 2888]
3. [Option ID = 2885]
4. [Option ID = 2887]
Correct Answer :-
76)
77)
78)
[Option ID = 2887]
[Question ID = 755]
1. [Option ID = 3019]
2. [Option ID = 3018]
3. [Option ID = 3020]
4. [Option ID = 3017]
Correct Answer :-
[Option ID = 3019]
[Question ID = 695]
1. Ba [Option ID = 2779]
2. BaO [Option ID = 2780]
3. BaOH [Option ID = 2778]
4. BaH
2
[Option ID = 2777]
Correct Answer :-
BaO [Option ID = 2780]
[Question ID = 703]
1. [Option ID = 2811]
2. [Option ID = 2809]
3. [Option ID = 2812]
4. [Option ID = 2810]
Correct Answer :-
[Option ID = 2811]
79)
80)
81)
82)
83)
[Option ID = 2810]
[Question ID = 732]
1. [Option ID = 2927]
2. [Option ID = 2928]
3. [Option ID = 2926]
4. [Option ID = 2925]
Correct Answer :-
[Option ID = 2926]
A monoatomic gas following Fermi-Dirac statistics begins to follow Maxwell-Boltzmann statistics at: [Question ID = 735]
1. Low Temperature and low density [Option ID = 2937]
2. High Temperature and high density [Option ID = 2940]
3. Low Temperature and high density [Option ID = 2938]
4. High Temperature and low density [Option ID = 2939]
Correct Answer :-
High Temperature and low density [Option ID = 2939]
The Dulong and Petit?s Law says that the molar heat capacity of elements is: [Question ID = 741]
1. [Option ID = 2964]
2. [Option ID = 2961]
3. [Option ID = 2963]
4. [Option ID = 2962]
Correct Answer :-
[Option ID = 2961]
What is the most common natural form in which fluorine is found on earth?
[Question ID = 684]
1. As a fluoride ion in various minerals [Option ID = 2735]
2. As weak acid HF (aq) [Option ID = 2734]
3. In various fluorocarbon compounds in the atmosphere. [Option ID = 2736]
4. As XeF
2
(s)

[Option ID = 2733]
Correct Answer :-
As a fluoride ion in various minerals [Option ID = 2735]
What is the correct form of Stirling?s approximation?
[Question ID = 738]
1. [Option ID = 2950]
2. [Option ID = 2951]
FirstRanker.com - FirstRanker's Choice
1)
2)
3)
4)
5)
DU PhD in Chemistry
Topic:- DU_J18_PHD_CHEM
Which of the following statements about sulfur dioxide is true?
[Question ID = 677]
1. It forms a S-S dimer in condensed phase [Option ID = 2707]
2. Its anhydride of sulfuric acid [Option ID = 2706]
3. Its O-S-O angle is 180
0
[Option ID = 2708]
4. It is a product of the combustion of fossil fuels that contain sulfur [Option ID = 2705]
Correct Answer :-
It is a product of the combustion of fossil fuels that contain sulfur [Option ID = 2705]
Which of the following is a strong acid in pure liquid HF
[Question ID = 683]
1. H
2
O [Option ID = 2731]
2. NaF [Option ID = 2729]
3. CH
3
COOH [Option ID = 2730]
4. SbF
5
[Option ID = 2732]
Correct Answer :-
SbF
5
[Option ID = 2732]
Each of the following molecules can act as a chelating agent EXCEPT [Question ID = 679]
1. [Option ID = 2716]
2. [Option ID = 2714]
3. [Option ID = 2713]
4. [Option ID = 2715]
Correct Answer :-
[Option ID = 2714]
What is correct about h-index?
[Question ID = 758]
1. Alternative of impact factor [Option ID = 3031]
2. Based on most quoted papers [Option ID = 3030]
3. Quantify scientific productivity [Option ID = 3029]
4. All of these [Option ID = 3032]
Correct Answer :-
All of these [Option ID = 3032]
The hyperfine electron spin resonance (e.s.r.) spectrum of the benzene radical has how many lines? [Question ID = 748]
1. 12 [Option ID = 2992]
2. 7 [Option ID = 2990]
3. 1 [Option ID = 2991]
4. 6 [Option ID = 2989]
Correct Answer :-
7 [Option ID = 2990]
6)
7)
8)
9)
10)
11)
12)
The energy changes involving the core electrons of an atom or molecule are expressed in which region of the electromagnetic
spectrum? [Question ID = 742]
1. Ultraviolet and Visible region [Option ID = 2967]
2. X-ray region [Option ID = 2968]
3. Radiofrequency region [Option ID = 2966]
4. Infra-red region [Option ID = 2965]
Correct Answer :-
X-ray region [Option ID = 2968]
Find out the expected intensity ratio of M and M+1 signal for the Naphthalene molecular ion [Question ID = 726]
1. 99:1.1 [Option ID = 2903]
2. 1.1:99 [Option ID = 2904]
3. 9:01 [Option ID = 2901]
4. 1:9 [Option ID = 2902]
Correct Answer :-
9:01 [Option ID = 2901]
Cobalt-60 is used in radiation therapy of cancer and can be produced by the bombardment of Cobalt-59 with [Question ID = 692]
1. Alpha particles [Option ID = 2765]
2. Beta particles [Option ID = 2767]
3. Neutrons [Option ID = 2766]
4. Gamma rays [Option ID = 2768]
Correct Answer :-
Neutrons [Option ID = 2766]
The standard emf of galvanic cell involving 3 moles of electrons in its redox reaction is 0.59 V. The equilibrium constant for the
reaction of the cell is- [Question ID = 763]
1. [Option ID = 3051]
2. [Option ID = 3052]
3. [Option ID = 3049]
4. [Option ID = 3050]
Correct Answer :-
[Option ID = 3052]
A characteristic common to polymers that can be made to conduct electricity such as polyacetylene, polypyrrole is: [Question ID =
685]
1. Conjugation throughout the polymeric chain. [Option ID = 2740]
2. A high degree of cross linking [Option ID = 2738]
3. A very low glass transition temperature [Option ID = 2737]
4. Presence of stereogenic centers of the same configuration [Option ID = 2739]
Correct Answer :-
Conjugation throughout the polymeric chain. [Option ID = 2740]
Impact factor is [Question ID = 768]
1. Ratio between citations and recent citable items publish [Option ID = 3071]
2. All of these [Option ID = 3072]
3. Addition of citations and recent citable items publish [Option ID = 3069]
4. Ratio between recent citable items publish and citations [Option ID = 3070]
Correct Answer :-
Ratio between recent citable items publish and citations [Option ID = 3070]
On the basis of oxidation-reduction potential, which of the following is most likely to occur? [Question ID = 693]
13)
14)
15)
16)
17)
1. [Option ID = 2770]
2. [Option ID = 2772]
3. [Option ID = 2771]
4. [Option ID = 2769]
Correct Answer :-
[Option ID = 2769]
How many diastereoisomers are possible for the compound 2, 4 ?diphenylcyclobutane-1, 3 di carboxylic acids. [Question ID = 725]
1. 6 [Option ID = 2899]
2. 5 [Option ID = 2898]
3. 8 [Option ID = 2900]
4. 4 [Option ID = 2897]
Correct Answer :-
5 [Option ID = 2898]
An increase in equivalent conductance of a strong electrolyte with dilution is mainly due to- [Question ID = 764]
1. increase in ionic mobility of ions [Option ID = 3055]
2. increase in number of ions [Option ID = 3054]
3. 100% ionization of electrolyte at normal dilution [Option ID = 3056]
4. increase in both i.e. number of ions and ionic mobility of ions. [Option ID = 3053]
Correct Answer :-
increase in ionic mobility of ions [Option ID = 3055]
The solid state structures of the principal allotropes of elemental boron are made up of which of the following structural units
[Question ID = 699]
1. [Option ID = 2796]
2. [Option ID = 2795]
3. [Option ID = 2794]
4. [Option ID = 2793]
Correct Answer :-
[Option ID = 2793]
The molecular geometry of thionyl chloride is best described as [Question ID = 688]
1. T-shaped [Option ID = 2752]
2. Tetrahedral [Option ID = 2751]
3. Trigonal pyramidal [Option ID = 2749]
4. Trigonal planar [Option ID = 2750]
Correct Answer :-
Trigonal pyramidal [Option ID = 2749]
In a face-center cubic (FCC) type of crystal lattice, the number of atoms belonging exclusively to each unit cell within the lattice
is/are: [Question ID = 754]
1. 2 [Option ID = 3014]
2. 1 [Option ID = 3013]
3. 3 [Option ID = 3015]
4. 4 [Option ID = 3016]
Correct Answer :-
4 [Option ID = 3016]
18)
19)
20)
21)
22)
23)
24)
Among the following, the weakest oxidizing agent is [Question ID = 675]
1. Mg (s) [Option ID = 2698]
2. [Option ID = 2699]
3. [Option ID = 2700]
4. [Option ID = 2697]
Correct Answer :-
Mg (s) [Option ID = 2698]
For a polymer, which of the following statement/s is/are true? [Question ID = 759]
1. Weight average molecular weight is almost always higher than the number average molecular weight [Option ID = 3035]
2. Formation of a polypeptide from its monomers (amino acids) is an example of addition polymerization [Option ID = 3034]
3. All of these [Option ID = 3036]
4. Vinyl polymerization is an example of condensation polymerization. [Option ID = 3033]
Correct Answer :-
Weight average molecular weight is almost always higher than the number average molecular weight [Option ID = 3035]
Quantum dots are [Question ID = 762]
1. Three dimensional [Option ID = 3048]
2. One dimensional [Option ID = 3046]
3. Two dimensional [Option ID = 3047]
4. Zero dimensional [Option ID = 3045]
Correct Answer :-
Zero dimensional [Option ID = 3045]
The unit of rate constant for a third order reaction is: [Question ID = 749]
1. [Option ID = 2993]
2. [Option ID = 2995]
3. [Option ID = 2996]
4. [Option ID = 2994]
Correct Answer :-
[Option ID = 2996]
All the following elements have at least one isotope that is not radioactive EXCEPT [Question ID = 673]
1. Pb [Option ID = 2690]
2. O [Option ID = 2689]
3. Sn [Option ID = 2691]
4. No [Option ID = 2692]
Correct Answer :-
No [Option ID = 2692]
The conditions for a species to follow Bose-Einstein statistics are; [Question ID = 736]
1. Particles are indistinguishable, with no restriction on filling up of energy levels [Option ID = 2944]
2. Particles are indistinguishable, with a restriction on filling up of energy levels [Option ID = 2943]
3. Particles are distinguishable, with a restriction on filling up of energy levels [Option ID = 2941]
4. Particles are distinguishable, with no restriction on filling up of energy levels [Option ID = 2942]
Correct Answer :-
Particles are indistinguishable, with no restriction on filling up of energy levels [Option ID = 2944]
In the kinetic theory of collisions, the SI unit of collision number, in terms of m (meter) and s (second), is:
[Question ID = 761]
25)
26)
27)
28)
29)
1. m-
2
s
-1
[Option ID = 3042]
2. m
4
s
-1
[Option ID = 3041]
3. m
2
s
-1
[Option ID = 3043]
4. None of these [Option ID = 3044]
Correct Answer :-
None of these [Option ID = 3044]
Correct characteristics of the functional groups of adenine in DNA base pair are [Question ID = 706]
1. [Option ID = 2824]
2. [Option ID = 2821]
3. [Option ID = 2823]
4. [Option ID = 2822]
Correct Answer :-
[Option ID = 2822]
The carbon monoxide molecule has an internuclear distance of 1.13 Angstroms. What is the moment of Inertia of this molecule?
[Question ID = 740]
1. [Option ID = 2960]
2. [Option ID = 2957]
3. [Option ID = 2958]
4. [Option ID = 2959]
Correct Answer :-
[Option ID = 2957]
Which of the following represent/s non-linear optical technique? [Question ID = 744]
1. Second Harmonic generation [Option ID = 2974]
2. Two-photon photoluminescence [Option ID = 2975]
3. Four-wave mixing [Option ID = 2973]
4. All of these [Option ID = 2976]
Correct Answer :-
All of these [Option ID = 2976]
. Which of the following does not affect the intensity of spectral lines of a sample? [Question ID = 743]
1. Path length of a sample [Option ID = 2972]
2. Population of energy states [Option ID = 2970]
3. Heisenberg?s Uncertainty principle [Option ID = 2971]
4. Concentration of a sample [Option ID = 2969]
Correct Answer :-
Heisenberg?s Uncertainty principle [Option ID = 2971]
[Question ID = 716]
30)
31)
1. [Option ID = 2862]
2. [Option ID = 2863]
3. [Option ID = 2861]
4. [Option ID = 2864]
Correct Answer :-
[Option ID = 2864]
[Question ID = 712]
1. [Option ID = 2845]
2. [Option ID = 2846]
3. [Option ID = 2848]
4. [Option ID = 2847]
Correct Answer :-
[Option ID = 2847]
[Question ID = 686]
1. [Option ID = 2743]
2. [Option ID = 2742]
3. [Option ID = 2741]
32)
33)
4. [Option ID = 2744]
Correct Answer :-
[Option ID = 2743]
[Question ID = 5633]
1. [Option ID = 22523]
2. [Option ID = 22524]
3. [Option ID = 22526]
4. [Option ID = 22525]
Correct Answer :-
[Option ID = 22523]
[Question ID = 730]
1. [Option ID = 2918]
2. [Option ID = 2920]
3. [Option ID = 2917]
4. [Option ID = 2919]
Correct Answer :-
34)
35)
36)
[Option ID = 2919]
[Question ID = 714]
1. [Option ID = 2856]
2. [Option ID = 2853]
3. [Option ID = 2855]
4. [Option ID = 2854]
Correct Answer :-
[Option ID = 2854]
[Question ID = 669]
1. 2 [Option ID = 2673]
2. 18 [Option ID = 2676]
3. 5 [Option ID = 2674]
4. 10 [Option ID = 2675]
Correct Answer :-
5 [Option ID = 2674]
[Question ID = 689]
1. 2 [Option ID = 2754]
2. 0 [Option ID = 2756]
3. 1 [Option ID = 2755]
37)
38)
4. 3 [Option ID = 2753]
Correct Answer :-
1 [Option ID = 2755]
[Question ID = 718]
1. C) [Option ID = 2871]
2. B) [Option ID = 2870]
3. A) [Option ID = 2869]
4. D) both (B) and (C) [Option ID = 2872]
Correct Answer :-
A) [Option ID = 2869]
[Question ID = 709]
1. [Option ID = 2836]
2. [Option ID = 2833]
3. [Option ID = 2835]
4. [Option ID = 2834]
39)
40)
41)
42)
Correct Answer :-
[Option ID = 2836]
[Question ID = 705]
1. (2Z, 4Z)-3-chlorohexa-2, 4-diene-1,6-diol. [Option ID = 2819]
2. (2E, 4E)-3-chlorohexa-2, 4-diene-1,6-diol. [Option ID = 2817]
3. (2Z, 4E)-3-chlorohexa-2, 4-diene-1,6-diol. [Option ID = 2818]
4. (2Z, 4E)-3-chlorohexa-2, 4-diene-1,6-diol. [Option ID = 2820]
Correct Answer :-
(2Z, 4E)-3-chlorohexa-2, 4-diene-1,6-diol. [Option ID = 2820]
[Question ID = 680]
1. The presence of other complexing ligands in solution affects the equilibrium concentration of metal-EDTA complexes [Option ID = 2719]
2. Metal-EDTA complexes have an equilibrium concentration independent of pH [Option ID = 2718]
3. Metal-EDTA complexes are often 2:1 in stoichiometry [Option ID = 2717]
4. Metal-EDTA complexes are less stable than the corresponding metal-ammine complexes [Option ID = 2720]
Correct Answer :-
The presence of other complexing ligands in solution affects the equilibrium concentration of metal-EDTA complexes [Option ID = 2719]
[Question ID = 671]
1. 150 g [Option ID = 2682]
2. 300 g [Option ID = 2684]
3. 120 g [Option ID = 2681]
4. 180 g [Option ID = 2683]
Correct Answer :-
300 g [Option ID = 2684]
[Question ID = 756]
1. [Option ID = 3021]
2. [Option ID = 3022]
3. [Option ID = 3024]
4. [Option ID = 3023]
Correct Answer :-
43)
44)
45)
[Option ID = 3024]
[Question ID = 750]
1. [Option ID = 2999]
2. [Option ID = 3000]
3. [Option ID = 2997]
4. [Option ID = 2998]
Correct Answer :-
[Option ID = 2997]
[Question ID = 715]
1. [Option ID = 2857]
2. [Option ID = 2860]
3. [Option ID = 2858]
4. [Option ID = 2859]
Correct Answer :-
[Option ID = 2860]
46)
47)
[Question ID = 719]
1. [Option ID = 2873]
2. [Option ID = 2876]
3. [Option ID = 2874]
4. [Option ID = 2875]
Correct Answer :-
[Question ID = 674]
1. Evaporating some water to decrease the volume of the solution. [Option ID = 2696]
2. [Option ID = 2695]
3. [Option ID = 2694]
4. [Option ID = 2693]
Correct Answer :-
[Option ID = 2693]
[Question ID = 728]
1. [Option ID = 2910]
2. [Option ID = 2911]
3. [Option ID = 2909]
4. [Option ID = 2912]
48)
49)
50)
51)
Correct Answer :-
[Option ID = 2912]
[Question ID = 697]
1. III only [Option ID = 2788]
2. II only [Option ID = 2787]
3. I only [Option ID = 2785]
4. I, II and III [Option ID = 2786]
Correct Answer :-
I, II and III [Option ID = 2786]
[Question ID = 700]
1. [Option ID = 2798]
2. [Option ID = 2797]
3. [Option ID = 2800]
4. [Option ID = 2799]
Correct Answer :-
[Option ID = 2799]
[Question ID = 678]
1. Cr
2+
has a low cationic charge [Option ID = 2711]
2. F has -1 anionic charge and highly electronegative [Option ID = 2709]
3. Spin-orbit coupling in Cr
2+
[Option ID = 2712]
4. The Jahn-Teller effect [Option ID = 2710]
Correct Answer :-
The Jahn-Teller effect [Option ID = 2710]
[Question ID = 723]
1. [Option ID = 2889]
52)
53)
2. [Option ID = 2890]
3. [Option ID = 2891]
4. [Option ID = 2892]
Correct Answer :-
[Option ID = 2889]
[Question ID = 727]
1. [Option ID = 2907]
2. [Option ID = 2906]
3. [Option ID = 2905]
4. [Option ID = 2908]
Correct Answer :-
[Option ID = 2905]
[Question ID = 707]
54)
55)
56)
1. N9 [Option ID = 2828]
2. N7 [Option ID = 2827]
3. N1 [Option ID = 2825]
4. N3 [Option ID = 2826]
Correct Answer :-
N7 [Option ID = 2827]
[Question ID = 672]
1. Bicapped prism [Option ID = 2688]
2. Square pyramidal [Option ID = 2686]
3. Trigonal planar [Option ID = 2685]
4. Bent [Option ID = 2687]
Correct Answer :-
Square pyramidal [Option ID = 2686]
[Question ID = 704]
1. [Option ID = 2816]
2. [Option ID = 2814]
3. [Option ID = 2813]
4. [Option ID = 2815]
Correct Answer :-
[Option ID = 2813]
57)
58)
[Question ID = 708]
1. [Option ID = 2829]
2. [Option ID = 2832]
3. [Option ID = 2831]
4. [Option ID = 2830]
Correct Answer :-
[Option ID = 2831]
[Question ID = 717]
1. [Option ID = 2866]
2. [Option ID = 2865]
3. [Option ID = 2867]
4. [Option ID = 2868]
Correct Answer :-
[Option ID = 2866]
59)
[Question ID = 711]
1. [Option ID = 2843]
2. [Option ID = 2842]
3. [Option ID = 2844]
4. [Option ID = 2841]
Correct Answer :-
[Option ID = 2842]
[Question ID = 710]
1. [Option ID = 2838]
2. [Option ID = 2839]
3. [Option ID = 2837]
4. [Option ID = 2840]
Correct Answer :-
60)
61)
62)
[Option ID = 2838]
[Question ID = 682]
1. 0.30 M [Option ID = 2728]
2. 0.25 M [Option ID = 2727]
3. 0.60 M [Option ID = 2725]
4. 0.10 M [Option ID = 2726]
Correct Answer :-
0.30 M [Option ID = 2728]
[Question ID = 772]
1. [Option ID = 3087]
2. [Option ID = 3088]
3. [Option ID = 3086]
4. [Option ID = 3085]
Correct Answer :-
[Option ID = 3086]
[Question ID = 733]
1. [Option ID = 2932]
63)
64)
65)
2. [Option ID = 2929]
3. [Option ID = 2930]
4. [Option ID = 2931]
Correct Answer :-
[Option ID = 2929]
[Question ID = 767]
1. 120 [Option ID = 3065]
2. 280 [Option ID = 3066]
3. 300 [Option ID = 3068]
4. 20 [Option ID = 3067]
Correct Answer :-
20 [Option ID = 3067]
[Question ID = 734]
1. [Option ID = 2936]
2. [Option ID = 2933]
3. [Option ID = 2934]
4. [Option ID = 2935]
Correct Answer :-
[Option ID = 2935]
[Question ID = 676]
66)
67)
68)
1. [Option ID = 2702]
2. [Option ID = 2703]
3. [Option ID = 2704]
4. [Option ID = 2701]
Correct Answer :-
[Option ID = 2703]
[Question ID = 729]
1. (2Z, 4Z)-3, 4-dibromo hepta-2, 4-diene [Option ID = 2914]
2. (2E, 4E)-3, 4-dibromo hepta-2, 4-diene [Option ID = 2915]
3. (2E, 4Z)-3, 4-dibromo hepta-2, 4-diene [Option ID = 2916]
4. (2Z, 4E)-3, 4-dibromo hepta-2, 4-diene [Option ID = 2913]
Correct Answer :-
(2E, 4Z)-3, 4-dibromo hepta-2, 4-diene [Option ID = 2916]
[Question ID = 765]
1. [Option ID = 3060]
2. [Option ID = 3057]
3. [Option ID = 3058]
4. [Option ID = 3059]
Correct Answer :-
[Option ID = 3059]
[Question ID = 724]
1. [Option ID = 2893]
69)
2. [Option ID = 2895]
3. [Option ID = 2896]
4. [Option ID = 2894]
Correct Answer :-
[Option ID = 2893]
[Question ID = 721]
1. [Option ID = 2881]
2. [Option ID = 2884]
3. [Option ID = 2882]
4. [Option ID = 2883]
Correct Answer :-
[Option ID = 2882]
70)
71)
72)
73)
[Question ID = 766]
1. 512 times [Option ID = 3064]
2. 256 times [Option ID = 3063]
3. 128 times [Option ID = 3062]
4. 64 times [Option ID = 3061]
Correct Answer :-
512 times [Option ID = 3064]
[Question ID = 670]
1. 2 [Option ID = 2678]
2. 5 [Option ID = 2680]
3. 1 [Option ID = 2677]
4. 3 [Option ID = 2679]
Correct Answer :-
5 [Option ID = 2680]
[Question ID = 731]
1. [Option ID = 2924]
2. [Option ID = 2923]
3. [Option ID = 2922]
4. [Option ID = 2921]
Correct Answer :-
[Option ID = 2922]
[Question ID = 690]
1. [Option ID = 2760]
2. [Option ID = 2759]
3. [Option ID = 2757]
4. [Option ID = 2758]
74)
75)
Correct Answer :-
[Option ID = 2758]
[Question ID = 720]
1. [Option ID = 2879]
2. [Option ID = 2878]
3. [Option ID = 2880]
4. [Option ID = 2877]
Correct Answer :-
[Option ID = 2877]
[Question ID = 722]
1. [Option ID = 2886]
2. [Option ID = 2888]
3. [Option ID = 2885]
4. [Option ID = 2887]
Correct Answer :-
76)
77)
78)
[Option ID = 2887]
[Question ID = 755]
1. [Option ID = 3019]
2. [Option ID = 3018]
3. [Option ID = 3020]
4. [Option ID = 3017]
Correct Answer :-
[Option ID = 3019]
[Question ID = 695]
1. Ba [Option ID = 2779]
2. BaO [Option ID = 2780]
3. BaOH [Option ID = 2778]
4. BaH
2
[Option ID = 2777]
Correct Answer :-
BaO [Option ID = 2780]
[Question ID = 703]
1. [Option ID = 2811]
2. [Option ID = 2809]
3. [Option ID = 2812]
4. [Option ID = 2810]
Correct Answer :-
[Option ID = 2811]
79)
80)
81)
82)
83)
[Option ID = 2810]
[Question ID = 732]
1. [Option ID = 2927]
2. [Option ID = 2928]
3. [Option ID = 2926]
4. [Option ID = 2925]
Correct Answer :-
[Option ID = 2926]
A monoatomic gas following Fermi-Dirac statistics begins to follow Maxwell-Boltzmann statistics at: [Question ID = 735]
1. Low Temperature and low density [Option ID = 2937]
2. High Temperature and high density [Option ID = 2940]
3. Low Temperature and high density [Option ID = 2938]
4. High Temperature and low density [Option ID = 2939]
Correct Answer :-
High Temperature and low density [Option ID = 2939]
The Dulong and Petit?s Law says that the molar heat capacity of elements is: [Question ID = 741]
1. [Option ID = 2964]
2. [Option ID = 2961]
3. [Option ID = 2963]
4. [Option ID = 2962]
Correct Answer :-
[Option ID = 2961]
What is the most common natural form in which fluorine is found on earth?
[Question ID = 684]
1. As a fluoride ion in various minerals [Option ID = 2735]
2. As weak acid HF (aq) [Option ID = 2734]
3. In various fluorocarbon compounds in the atmosphere. [Option ID = 2736]
4. As XeF
2
(s)

[Option ID = 2733]
Correct Answer :-
As a fluoride ion in various minerals [Option ID = 2735]
What is the correct form of Stirling?s approximation?
[Question ID = 738]
1. [Option ID = 2950]
2. [Option ID = 2951]
84)
85)
86)
87)
88)
3. [Option ID = 2949]
4. [Option ID = 2952]
Correct Answer :-
[Option ID = 2950]
What is the total energy of one mole of an ideal monoatomic gas in terms of Boltzmann?s Constant (k), Avogadro?s number (N) and
temperature (T)
[Question ID = 739]
1. 3 NkT [Option ID = 2953]
2. (3/2) NkT [Option ID = 2956]
3. (1/2) NkT [Option ID = 2955]
4. NkT [Option ID = 2954]
Correct Answer :-
(3/2) NkT [Option ID = 2956]
The following equation is associated with the relationship between the diffusion current and the concentration of the depolarizer
used in polarography: [Question ID = 753]
1. Debye-Huckel equation [Option ID = 3009]
2. Stern-Volmer equation [Option ID = 3010]
3. Nyquist equation [Option ID = 3012]
4. Ilkovic equation [Option ID = 3011]
Correct Answer :-
Ilkovic equation [Option ID = 3011]
Electronic transitions originating from the 1S energy level of the Hydrogen atom to higher levels belong to which series? [Question ID
= 747]
1. Lyman Series [Option ID = 2985]
2. Brackett Series [Option ID = 2987]
3. Balmer Series [Option ID = 2986]
4. Pfund Series [Option ID = 2988]
Correct Answer :-
Lyman Series [Option ID = 2985]
Which of the following reactions best classified as an oxidative addition? [Question ID = 701]
1. [Option ID = 2801]
2. [Option ID = 2803]
3. [Option ID = 2802]
4. [Option ID = 2804]
Correct Answer :-
[Option ID = 2804]
Which of the following is required for both paramagnetism and ferromagnetism? [Question ID = 698]
1. Super exchange [Option ID = 2791]
2. unpaired electrons [Option ID = 2792]
3. Low-spin electron configuration [Option ID = 2790]
4. Strong oxidizing conditions [Option ID = 2789]
Correct Answer :-
unpaired electrons [Option ID = 2792]
FirstRanker.com - FirstRanker's Choice
1)
2)
3)
4)
5)
DU PhD in Chemistry
Topic:- DU_J18_PHD_CHEM
Which of the following statements about sulfur dioxide is true?
[Question ID = 677]
1. It forms a S-S dimer in condensed phase [Option ID = 2707]
2. Its anhydride of sulfuric acid [Option ID = 2706]
3. Its O-S-O angle is 180
0
[Option ID = 2708]
4. It is a product of the combustion of fossil fuels that contain sulfur [Option ID = 2705]
Correct Answer :-
It is a product of the combustion of fossil fuels that contain sulfur [Option ID = 2705]
Which of the following is a strong acid in pure liquid HF
[Question ID = 683]
1. H
2
O [Option ID = 2731]
2. NaF [Option ID = 2729]
3. CH
3
COOH [Option ID = 2730]
4. SbF
5
[Option ID = 2732]
Correct Answer :-
SbF
5
[Option ID = 2732]
Each of the following molecules can act as a chelating agent EXCEPT [Question ID = 679]
1. [Option ID = 2716]
2. [Option ID = 2714]
3. [Option ID = 2713]
4. [Option ID = 2715]
Correct Answer :-
[Option ID = 2714]
What is correct about h-index?
[Question ID = 758]
1. Alternative of impact factor [Option ID = 3031]
2. Based on most quoted papers [Option ID = 3030]
3. Quantify scientific productivity [Option ID = 3029]
4. All of these [Option ID = 3032]
Correct Answer :-
All of these [Option ID = 3032]
The hyperfine electron spin resonance (e.s.r.) spectrum of the benzene radical has how many lines? [Question ID = 748]
1. 12 [Option ID = 2992]
2. 7 [Option ID = 2990]
3. 1 [Option ID = 2991]
4. 6 [Option ID = 2989]
Correct Answer :-
7 [Option ID = 2990]
6)
7)
8)
9)
10)
11)
12)
The energy changes involving the core electrons of an atom or molecule are expressed in which region of the electromagnetic
spectrum? [Question ID = 742]
1. Ultraviolet and Visible region [Option ID = 2967]
2. X-ray region [Option ID = 2968]
3. Radiofrequency region [Option ID = 2966]
4. Infra-red region [Option ID = 2965]
Correct Answer :-
X-ray region [Option ID = 2968]
Find out the expected intensity ratio of M and M+1 signal for the Naphthalene molecular ion [Question ID = 726]
1. 99:1.1 [Option ID = 2903]
2. 1.1:99 [Option ID = 2904]
3. 9:01 [Option ID = 2901]
4. 1:9 [Option ID = 2902]
Correct Answer :-
9:01 [Option ID = 2901]
Cobalt-60 is used in radiation therapy of cancer and can be produced by the bombardment of Cobalt-59 with [Question ID = 692]
1. Alpha particles [Option ID = 2765]
2. Beta particles [Option ID = 2767]
3. Neutrons [Option ID = 2766]
4. Gamma rays [Option ID = 2768]
Correct Answer :-
Neutrons [Option ID = 2766]
The standard emf of galvanic cell involving 3 moles of electrons in its redox reaction is 0.59 V. The equilibrium constant for the
reaction of the cell is- [Question ID = 763]
1. [Option ID = 3051]
2. [Option ID = 3052]
3. [Option ID = 3049]
4. [Option ID = 3050]
Correct Answer :-
[Option ID = 3052]
A characteristic common to polymers that can be made to conduct electricity such as polyacetylene, polypyrrole is: [Question ID =
685]
1. Conjugation throughout the polymeric chain. [Option ID = 2740]
2. A high degree of cross linking [Option ID = 2738]
3. A very low glass transition temperature [Option ID = 2737]
4. Presence of stereogenic centers of the same configuration [Option ID = 2739]
Correct Answer :-
Conjugation throughout the polymeric chain. [Option ID = 2740]
Impact factor is [Question ID = 768]
1. Ratio between citations and recent citable items publish [Option ID = 3071]
2. All of these [Option ID = 3072]
3. Addition of citations and recent citable items publish [Option ID = 3069]
4. Ratio between recent citable items publish and citations [Option ID = 3070]
Correct Answer :-
Ratio between recent citable items publish and citations [Option ID = 3070]
On the basis of oxidation-reduction potential, which of the following is most likely to occur? [Question ID = 693]
13)
14)
15)
16)
17)
1. [Option ID = 2770]
2. [Option ID = 2772]
3. [Option ID = 2771]
4. [Option ID = 2769]
Correct Answer :-
[Option ID = 2769]
How many diastereoisomers are possible for the compound 2, 4 ?diphenylcyclobutane-1, 3 di carboxylic acids. [Question ID = 725]
1. 6 [Option ID = 2899]
2. 5 [Option ID = 2898]
3. 8 [Option ID = 2900]
4. 4 [Option ID = 2897]
Correct Answer :-
5 [Option ID = 2898]
An increase in equivalent conductance of a strong electrolyte with dilution is mainly due to- [Question ID = 764]
1. increase in ionic mobility of ions [Option ID = 3055]
2. increase in number of ions [Option ID = 3054]
3. 100% ionization of electrolyte at normal dilution [Option ID = 3056]
4. increase in both i.e. number of ions and ionic mobility of ions. [Option ID = 3053]
Correct Answer :-
increase in ionic mobility of ions [Option ID = 3055]
The solid state structures of the principal allotropes of elemental boron are made up of which of the following structural units
[Question ID = 699]
1. [Option ID = 2796]
2. [Option ID = 2795]
3. [Option ID = 2794]
4. [Option ID = 2793]
Correct Answer :-
[Option ID = 2793]
The molecular geometry of thionyl chloride is best described as [Question ID = 688]
1. T-shaped [Option ID = 2752]
2. Tetrahedral [Option ID = 2751]
3. Trigonal pyramidal [Option ID = 2749]
4. Trigonal planar [Option ID = 2750]
Correct Answer :-
Trigonal pyramidal [Option ID = 2749]
In a face-center cubic (FCC) type of crystal lattice, the number of atoms belonging exclusively to each unit cell within the lattice
is/are: [Question ID = 754]
1. 2 [Option ID = 3014]
2. 1 [Option ID = 3013]
3. 3 [Option ID = 3015]
4. 4 [Option ID = 3016]
Correct Answer :-
4 [Option ID = 3016]
18)
19)
20)
21)
22)
23)
24)
Among the following, the weakest oxidizing agent is [Question ID = 675]
1. Mg (s) [Option ID = 2698]
2. [Option ID = 2699]
3. [Option ID = 2700]
4. [Option ID = 2697]
Correct Answer :-
Mg (s) [Option ID = 2698]
For a polymer, which of the following statement/s is/are true? [Question ID = 759]
1. Weight average molecular weight is almost always higher than the number average molecular weight [Option ID = 3035]
2. Formation of a polypeptide from its monomers (amino acids) is an example of addition polymerization [Option ID = 3034]
3. All of these [Option ID = 3036]
4. Vinyl polymerization is an example of condensation polymerization. [Option ID = 3033]
Correct Answer :-
Weight average molecular weight is almost always higher than the number average molecular weight [Option ID = 3035]
Quantum dots are [Question ID = 762]
1. Three dimensional [Option ID = 3048]
2. One dimensional [Option ID = 3046]
3. Two dimensional [Option ID = 3047]
4. Zero dimensional [Option ID = 3045]
Correct Answer :-
Zero dimensional [Option ID = 3045]
The unit of rate constant for a third order reaction is: [Question ID = 749]
1. [Option ID = 2993]
2. [Option ID = 2995]
3. [Option ID = 2996]
4. [Option ID = 2994]
Correct Answer :-
[Option ID = 2996]
All the following elements have at least one isotope that is not radioactive EXCEPT [Question ID = 673]
1. Pb [Option ID = 2690]
2. O [Option ID = 2689]
3. Sn [Option ID = 2691]
4. No [Option ID = 2692]
Correct Answer :-
No [Option ID = 2692]
The conditions for a species to follow Bose-Einstein statistics are; [Question ID = 736]
1. Particles are indistinguishable, with no restriction on filling up of energy levels [Option ID = 2944]
2. Particles are indistinguishable, with a restriction on filling up of energy levels [Option ID = 2943]
3. Particles are distinguishable, with a restriction on filling up of energy levels [Option ID = 2941]
4. Particles are distinguishable, with no restriction on filling up of energy levels [Option ID = 2942]
Correct Answer :-
Particles are indistinguishable, with no restriction on filling up of energy levels [Option ID = 2944]
In the kinetic theory of collisions, the SI unit of collision number, in terms of m (meter) and s (second), is:
[Question ID = 761]
25)
26)
27)
28)
29)
1. m-
2
s
-1
[Option ID = 3042]
2. m
4
s
-1
[Option ID = 3041]
3. m
2
s
-1
[Option ID = 3043]
4. None of these [Option ID = 3044]
Correct Answer :-
None of these [Option ID = 3044]
Correct characteristics of the functional groups of adenine in DNA base pair are [Question ID = 706]
1. [Option ID = 2824]
2. [Option ID = 2821]
3. [Option ID = 2823]
4. [Option ID = 2822]
Correct Answer :-
[Option ID = 2822]
The carbon monoxide molecule has an internuclear distance of 1.13 Angstroms. What is the moment of Inertia of this molecule?
[Question ID = 740]
1. [Option ID = 2960]
2. [Option ID = 2957]
3. [Option ID = 2958]
4. [Option ID = 2959]
Correct Answer :-
[Option ID = 2957]
Which of the following represent/s non-linear optical technique? [Question ID = 744]
1. Second Harmonic generation [Option ID = 2974]
2. Two-photon photoluminescence [Option ID = 2975]
3. Four-wave mixing [Option ID = 2973]
4. All of these [Option ID = 2976]
Correct Answer :-
All of these [Option ID = 2976]
. Which of the following does not affect the intensity of spectral lines of a sample? [Question ID = 743]
1. Path length of a sample [Option ID = 2972]
2. Population of energy states [Option ID = 2970]
3. Heisenberg?s Uncertainty principle [Option ID = 2971]
4. Concentration of a sample [Option ID = 2969]
Correct Answer :-
Heisenberg?s Uncertainty principle [Option ID = 2971]
[Question ID = 716]
30)
31)
1. [Option ID = 2862]
2. [Option ID = 2863]
3. [Option ID = 2861]
4. [Option ID = 2864]
Correct Answer :-
[Option ID = 2864]
[Question ID = 712]
1. [Option ID = 2845]
2. [Option ID = 2846]
3. [Option ID = 2848]
4. [Option ID = 2847]
Correct Answer :-
[Option ID = 2847]
[Question ID = 686]
1. [Option ID = 2743]
2. [Option ID = 2742]
3. [Option ID = 2741]
32)
33)
4. [Option ID = 2744]
Correct Answer :-
[Option ID = 2743]
[Question ID = 5633]
1. [Option ID = 22523]
2. [Option ID = 22524]
3. [Option ID = 22526]
4. [Option ID = 22525]
Correct Answer :-
[Option ID = 22523]
[Question ID = 730]
1. [Option ID = 2918]
2. [Option ID = 2920]
3. [Option ID = 2917]
4. [Option ID = 2919]
Correct Answer :-
34)
35)
36)
[Option ID = 2919]
[Question ID = 714]
1. [Option ID = 2856]
2. [Option ID = 2853]
3. [Option ID = 2855]
4. [Option ID = 2854]
Correct Answer :-
[Option ID = 2854]
[Question ID = 669]
1. 2 [Option ID = 2673]
2. 18 [Option ID = 2676]
3. 5 [Option ID = 2674]
4. 10 [Option ID = 2675]
Correct Answer :-
5 [Option ID = 2674]
[Question ID = 689]
1. 2 [Option ID = 2754]
2. 0 [Option ID = 2756]
3. 1 [Option ID = 2755]
37)
38)
4. 3 [Option ID = 2753]
Correct Answer :-
1 [Option ID = 2755]
[Question ID = 718]
1. C) [Option ID = 2871]
2. B) [Option ID = 2870]
3. A) [Option ID = 2869]
4. D) both (B) and (C) [Option ID = 2872]
Correct Answer :-
A) [Option ID = 2869]
[Question ID = 709]
1. [Option ID = 2836]
2. [Option ID = 2833]
3. [Option ID = 2835]
4. [Option ID = 2834]
39)
40)
41)
42)
Correct Answer :-
[Option ID = 2836]
[Question ID = 705]
1. (2Z, 4Z)-3-chlorohexa-2, 4-diene-1,6-diol. [Option ID = 2819]
2. (2E, 4E)-3-chlorohexa-2, 4-diene-1,6-diol. [Option ID = 2817]
3. (2Z, 4E)-3-chlorohexa-2, 4-diene-1,6-diol. [Option ID = 2818]
4. (2Z, 4E)-3-chlorohexa-2, 4-diene-1,6-diol. [Option ID = 2820]
Correct Answer :-
(2Z, 4E)-3-chlorohexa-2, 4-diene-1,6-diol. [Option ID = 2820]
[Question ID = 680]
1. The presence of other complexing ligands in solution affects the equilibrium concentration of metal-EDTA complexes [Option ID = 2719]
2. Metal-EDTA complexes have an equilibrium concentration independent of pH [Option ID = 2718]
3. Metal-EDTA complexes are often 2:1 in stoichiometry [Option ID = 2717]
4. Metal-EDTA complexes are less stable than the corresponding metal-ammine complexes [Option ID = 2720]
Correct Answer :-
The presence of other complexing ligands in solution affects the equilibrium concentration of metal-EDTA complexes [Option ID = 2719]
[Question ID = 671]
1. 150 g [Option ID = 2682]
2. 300 g [Option ID = 2684]
3. 120 g [Option ID = 2681]
4. 180 g [Option ID = 2683]
Correct Answer :-
300 g [Option ID = 2684]
[Question ID = 756]
1. [Option ID = 3021]
2. [Option ID = 3022]
3. [Option ID = 3024]
4. [Option ID = 3023]
Correct Answer :-
43)
44)
45)
[Option ID = 3024]
[Question ID = 750]
1. [Option ID = 2999]
2. [Option ID = 3000]
3. [Option ID = 2997]
4. [Option ID = 2998]
Correct Answer :-
[Option ID = 2997]
[Question ID = 715]
1. [Option ID = 2857]
2. [Option ID = 2860]
3. [Option ID = 2858]
4. [Option ID = 2859]
Correct Answer :-
[Option ID = 2860]
46)
47)
[Question ID = 719]
1. [Option ID = 2873]
2. [Option ID = 2876]
3. [Option ID = 2874]
4. [Option ID = 2875]
Correct Answer :-
[Question ID = 674]
1. Evaporating some water to decrease the volume of the solution. [Option ID = 2696]
2. [Option ID = 2695]
3. [Option ID = 2694]
4. [Option ID = 2693]
Correct Answer :-
[Option ID = 2693]
[Question ID = 728]
1. [Option ID = 2910]
2. [Option ID = 2911]
3. [Option ID = 2909]
4. [Option ID = 2912]
48)
49)
50)
51)
Correct Answer :-
[Option ID = 2912]
[Question ID = 697]
1. III only [Option ID = 2788]
2. II only [Option ID = 2787]
3. I only [Option ID = 2785]
4. I, II and III [Option ID = 2786]
Correct Answer :-
I, II and III [Option ID = 2786]
[Question ID = 700]
1. [Option ID = 2798]
2. [Option ID = 2797]
3. [Option ID = 2800]
4. [Option ID = 2799]
Correct Answer :-
[Option ID = 2799]
[Question ID = 678]
1. Cr
2+
has a low cationic charge [Option ID = 2711]
2. F has -1 anionic charge and highly electronegative [Option ID = 2709]
3. Spin-orbit coupling in Cr
2+
[Option ID = 2712]
4. The Jahn-Teller effect [Option ID = 2710]
Correct Answer :-
The Jahn-Teller effect [Option ID = 2710]
[Question ID = 723]
1. [Option ID = 2889]
52)
53)
2. [Option ID = 2890]
3. [Option ID = 2891]
4. [Option ID = 2892]
Correct Answer :-
[Option ID = 2889]
[Question ID = 727]
1. [Option ID = 2907]
2. [Option ID = 2906]
3. [Option ID = 2905]
4. [Option ID = 2908]
Correct Answer :-
[Option ID = 2905]
[Question ID = 707]
54)
55)
56)
1. N9 [Option ID = 2828]
2. N7 [Option ID = 2827]
3. N1 [Option ID = 2825]
4. N3 [Option ID = 2826]
Correct Answer :-
N7 [Option ID = 2827]
[Question ID = 672]
1. Bicapped prism [Option ID = 2688]
2. Square pyramidal [Option ID = 2686]
3. Trigonal planar [Option ID = 2685]
4. Bent [Option ID = 2687]
Correct Answer :-
Square pyramidal [Option ID = 2686]
[Question ID = 704]
1. [Option ID = 2816]
2. [Option ID = 2814]
3. [Option ID = 2813]
4. [Option ID = 2815]
Correct Answer :-
[Option ID = 2813]
57)
58)
[Question ID = 708]
1. [Option ID = 2829]
2. [Option ID = 2832]
3. [Option ID = 2831]
4. [Option ID = 2830]
Correct Answer :-
[Option ID = 2831]
[Question ID = 717]
1. [Option ID = 2866]
2. [Option ID = 2865]
3. [Option ID = 2867]
4. [Option ID = 2868]
Correct Answer :-
[Option ID = 2866]
59)
[Question ID = 711]
1. [Option ID = 2843]
2. [Option ID = 2842]
3. [Option ID = 2844]
4. [Option ID = 2841]
Correct Answer :-
[Option ID = 2842]
[Question ID = 710]
1. [Option ID = 2838]
2. [Option ID = 2839]
3. [Option ID = 2837]
4. [Option ID = 2840]
Correct Answer :-
60)
61)
62)
[Option ID = 2838]
[Question ID = 682]
1. 0.30 M [Option ID = 2728]
2. 0.25 M [Option ID = 2727]
3. 0.60 M [Option ID = 2725]
4. 0.10 M [Option ID = 2726]
Correct Answer :-
0.30 M [Option ID = 2728]
[Question ID = 772]
1. [Option ID = 3087]
2. [Option ID = 3088]
3. [Option ID = 3086]
4. [Option ID = 3085]
Correct Answer :-
[Option ID = 3086]
[Question ID = 733]
1. [Option ID = 2932]
63)
64)
65)
2. [Option ID = 2929]
3. [Option ID = 2930]
4. [Option ID = 2931]
Correct Answer :-
[Option ID = 2929]
[Question ID = 767]
1. 120 [Option ID = 3065]
2. 280 [Option ID = 3066]
3. 300 [Option ID = 3068]
4. 20 [Option ID = 3067]
Correct Answer :-
20 [Option ID = 3067]
[Question ID = 734]
1. [Option ID = 2936]
2. [Option ID = 2933]
3. [Option ID = 2934]
4. [Option ID = 2935]
Correct Answer :-
[Option ID = 2935]
[Question ID = 676]
66)
67)
68)
1. [Option ID = 2702]
2. [Option ID = 2703]
3. [Option ID = 2704]
4. [Option ID = 2701]
Correct Answer :-
[Option ID = 2703]
[Question ID = 729]
1. (2Z, 4Z)-3, 4-dibromo hepta-2, 4-diene [Option ID = 2914]
2. (2E, 4E)-3, 4-dibromo hepta-2, 4-diene [Option ID = 2915]
3. (2E, 4Z)-3, 4-dibromo hepta-2, 4-diene [Option ID = 2916]
4. (2Z, 4E)-3, 4-dibromo hepta-2, 4-diene [Option ID = 2913]
Correct Answer :-
(2E, 4Z)-3, 4-dibromo hepta-2, 4-diene [Option ID = 2916]
[Question ID = 765]
1. [Option ID = 3060]
2. [Option ID = 3057]
3. [Option ID = 3058]
4. [Option ID = 3059]
Correct Answer :-
[Option ID = 3059]
[Question ID = 724]
1. [Option ID = 2893]
69)
2. [Option ID = 2895]
3. [Option ID = 2896]
4. [Option ID = 2894]
Correct Answer :-
[Option ID = 2893]
[Question ID = 721]
1. [Option ID = 2881]
2. [Option ID = 2884]
3. [Option ID = 2882]
4. [Option ID = 2883]
Correct Answer :-
[Option ID = 2882]
70)
71)
72)
73)
[Question ID = 766]
1. 512 times [Option ID = 3064]
2. 256 times [Option ID = 3063]
3. 128 times [Option ID = 3062]
4. 64 times [Option ID = 3061]
Correct Answer :-
512 times [Option ID = 3064]
[Question ID = 670]
1. 2 [Option ID = 2678]
2. 5 [Option ID = 2680]
3. 1 [Option ID = 2677]
4. 3 [Option ID = 2679]
Correct Answer :-
5 [Option ID = 2680]
[Question ID = 731]
1. [Option ID = 2924]
2. [Option ID = 2923]
3. [Option ID = 2922]
4. [Option ID = 2921]
Correct Answer :-
[Option ID = 2922]
[Question ID = 690]
1. [Option ID = 2760]
2. [Option ID = 2759]
3. [Option ID = 2757]
4. [Option ID = 2758]
74)
75)
Correct Answer :-
[Option ID = 2758]
[Question ID = 720]
1. [Option ID = 2879]
2. [Option ID = 2878]
3. [Option ID = 2880]
4. [Option ID = 2877]
Correct Answer :-
[Option ID = 2877]
[Question ID = 722]
1. [Option ID = 2886]
2. [Option ID = 2888]
3. [Option ID = 2885]
4. [Option ID = 2887]
Correct Answer :-
76)
77)
78)
[Option ID = 2887]
[Question ID = 755]
1. [Option ID = 3019]
2. [Option ID = 3018]
3. [Option ID = 3020]
4. [Option ID = 3017]
Correct Answer :-
[Option ID = 3019]
[Question ID = 695]
1. Ba [Option ID = 2779]
2. BaO [Option ID = 2780]
3. BaOH [Option ID = 2778]
4. BaH
2
[Option ID = 2777]
Correct Answer :-
BaO [Option ID = 2780]
[Question ID = 703]
1. [Option ID = 2811]
2. [Option ID = 2809]
3. [Option ID = 2812]
4. [Option ID = 2810]
Correct Answer :-
[Option ID = 2811]
79)
80)
81)
82)
83)
[Option ID = 2810]
[Question ID = 732]
1. [Option ID = 2927]
2. [Option ID = 2928]
3. [Option ID = 2926]
4. [Option ID = 2925]
Correct Answer :-
[Option ID = 2926]
A monoatomic gas following Fermi-Dirac statistics begins to follow Maxwell-Boltzmann statistics at: [Question ID = 735]
1. Low Temperature and low density [Option ID = 2937]
2. High Temperature and high density [Option ID = 2940]
3. Low Temperature and high density [Option ID = 2938]
4. High Temperature and low density [Option ID = 2939]
Correct Answer :-
High Temperature and low density [Option ID = 2939]
The Dulong and Petit?s Law says that the molar heat capacity of elements is: [Question ID = 741]
1. [Option ID = 2964]
2. [Option ID = 2961]
3. [Option ID = 2963]
4. [Option ID = 2962]
Correct Answer :-
[Option ID = 2961]
What is the most common natural form in which fluorine is found on earth?
[Question ID = 684]
1. As a fluoride ion in various minerals [Option ID = 2735]
2. As weak acid HF (aq) [Option ID = 2734]
3. In various fluorocarbon compounds in the atmosphere. [Option ID = 2736]
4. As XeF
2
(s)

[Option ID = 2733]
Correct Answer :-
As a fluoride ion in various minerals [Option ID = 2735]
What is the correct form of Stirling?s approximation?
[Question ID = 738]
1. [Option ID = 2950]
2. [Option ID = 2951]
84)
85)
86)
87)
88)
3. [Option ID = 2949]
4. [Option ID = 2952]
Correct Answer :-
[Option ID = 2950]
What is the total energy of one mole of an ideal monoatomic gas in terms of Boltzmann?s Constant (k), Avogadro?s number (N) and
temperature (T)
[Question ID = 739]
1. 3 NkT [Option ID = 2953]
2. (3/2) NkT [Option ID = 2956]
3. (1/2) NkT [Option ID = 2955]
4. NkT [Option ID = 2954]
Correct Answer :-
(3/2) NkT [Option ID = 2956]
The following equation is associated with the relationship between the diffusion current and the concentration of the depolarizer
used in polarography: [Question ID = 753]
1. Debye-Huckel equation [Option ID = 3009]
2. Stern-Volmer equation [Option ID = 3010]
3. Nyquist equation [Option ID = 3012]
4. Ilkovic equation [Option ID = 3011]
Correct Answer :-
Ilkovic equation [Option ID = 3011]
Electronic transitions originating from the 1S energy level of the Hydrogen atom to higher levels belong to which series? [Question ID
= 747]
1. Lyman Series [Option ID = 2985]
2. Brackett Series [Option ID = 2987]
3. Balmer Series [Option ID = 2986]
4. Pfund Series [Option ID = 2988]
Correct Answer :-
Lyman Series [Option ID = 2985]
Which of the following reactions best classified as an oxidative addition? [Question ID = 701]
1. [Option ID = 2801]
2. [Option ID = 2803]
3. [Option ID = 2802]
4. [Option ID = 2804]
Correct Answer :-
[Option ID = 2804]
Which of the following is required for both paramagnetism and ferromagnetism? [Question ID = 698]
1. Super exchange [Option ID = 2791]
2. unpaired electrons [Option ID = 2792]
3. Low-spin electron configuration [Option ID = 2790]
4. Strong oxidizing conditions [Option ID = 2789]
Correct Answer :-
unpaired electrons [Option ID = 2792]
89)
90)
91)
92)
93)
94)
Which of the following experimental techniques is not used to determine the average molecular weight of a polymer? [Question ID =
760]
1. Transmission electron microscopy [Option ID = 3039]
2. Equilibrium sedimentation [Option ID = 3038]
3. Intrinsic viscosity measurement [Option ID = 3040]
4. Dynamic light scattering [Option ID = 3037]
Correct Answer :-
Transmission electron microscopy [Option ID = 3039]
Which of the following is NOT a known relatively stable compound of uranium? [Question ID = 687]
1. [Option ID = 2745]
2. [Option ID = 2748]
3. [Option ID = 2747]
4. [Option ID = 2746]
Correct Answer :-
[Option ID = 2746]
Which of the following compounds exist in stereoisomeric form? [Question ID = 681]
1. [Option ID = 2721]
2. [Option ID = 2724]
3. [Option ID = 2722]
4. [Option ID = 2723]
Correct Answer :-
[Option ID = 2724]
Which of the following statement is not true? [Question ID = 745]
1. Methane is a spherical top molecule [Option ID = 2978]
2. Chloroform is a symmetric top molecule [Option ID = 2980]
3. Vinyl chloride is a symmetric top molecule [Option ID = 2979]
4. Water is an asymmetric top molecule [Option ID = 2977]
Correct Answer :-
Vinyl chloride is a symmetric top molecule [Option ID = 2979]
Which of the following is a n-type semiconductor? [Question ID = 696]
1. Silicon carbide [Option ID = 2784]
2. Silicon [Option ID = 2781]
3. Arsenic doped silicon [Option ID = 2783]
4. Gallium doped silicon [Option ID = 2782]
Correct Answer :-
Arsenic doped silicon [Option ID = 2783]
Which of the statement is not true? [Question ID = 746]
1. Franck Condon Principle states that during electronic transition the internuclear distance of a molecule does not change [Option ID = 2983]
2. The intensity of a fundamental vibrational transition is higher than that of a first overtone transition. [Option ID = 2984]
3. Morse equation represents the energy expression of a simple harmonic oscillator [Option ID = 2982]
4. The energy spacing between various vibrational levels are the same in a simple harmonic oscillator [Option ID = 2981]
Correct Answer :-
Franck Condon Principle states that during electronic transition the internuclear distance of a molecule does not change [Option ID = 2983]
FirstRanker.com - FirstRanker's Choice
1)
2)
3)
4)
5)
DU PhD in Chemistry
Topic:- DU_J18_PHD_CHEM
Which of the following statements about sulfur dioxide is true?
[Question ID = 677]
1. It forms a S-S dimer in condensed phase [Option ID = 2707]
2. Its anhydride of sulfuric acid [Option ID = 2706]
3. Its O-S-O angle is 180
0
[Option ID = 2708]
4. It is a product of the combustion of fossil fuels that contain sulfur [Option ID = 2705]
Correct Answer :-
It is a product of the combustion of fossil fuels that contain sulfur [Option ID = 2705]
Which of the following is a strong acid in pure liquid HF
[Question ID = 683]
1. H
2
O [Option ID = 2731]
2. NaF [Option ID = 2729]
3. CH
3
COOH [Option ID = 2730]
4. SbF
5
[Option ID = 2732]
Correct Answer :-
SbF
5
[Option ID = 2732]
Each of the following molecules can act as a chelating agent EXCEPT [Question ID = 679]
1. [Option ID = 2716]
2. [Option ID = 2714]
3. [Option ID = 2713]
4. [Option ID = 2715]
Correct Answer :-
[Option ID = 2714]
What is correct about h-index?
[Question ID = 758]
1. Alternative of impact factor [Option ID = 3031]
2. Based on most quoted papers [Option ID = 3030]
3. Quantify scientific productivity [Option ID = 3029]
4. All of these [Option ID = 3032]
Correct Answer :-
All of these [Option ID = 3032]
The hyperfine electron spin resonance (e.s.r.) spectrum of the benzene radical has how many lines? [Question ID = 748]
1. 12 [Option ID = 2992]
2. 7 [Option ID = 2990]
3. 1 [Option ID = 2991]
4. 6 [Option ID = 2989]
Correct Answer :-
7 [Option ID = 2990]
6)
7)
8)
9)
10)
11)
12)
The energy changes involving the core electrons of an atom or molecule are expressed in which region of the electromagnetic
spectrum? [Question ID = 742]
1. Ultraviolet and Visible region [Option ID = 2967]
2. X-ray region [Option ID = 2968]
3. Radiofrequency region [Option ID = 2966]
4. Infra-red region [Option ID = 2965]
Correct Answer :-
X-ray region [Option ID = 2968]
Find out the expected intensity ratio of M and M+1 signal for the Naphthalene molecular ion [Question ID = 726]
1. 99:1.1 [Option ID = 2903]
2. 1.1:99 [Option ID = 2904]
3. 9:01 [Option ID = 2901]
4. 1:9 [Option ID = 2902]
Correct Answer :-
9:01 [Option ID = 2901]
Cobalt-60 is used in radiation therapy of cancer and can be produced by the bombardment of Cobalt-59 with [Question ID = 692]
1. Alpha particles [Option ID = 2765]
2. Beta particles [Option ID = 2767]
3. Neutrons [Option ID = 2766]
4. Gamma rays [Option ID = 2768]
Correct Answer :-
Neutrons [Option ID = 2766]
The standard emf of galvanic cell involving 3 moles of electrons in its redox reaction is 0.59 V. The equilibrium constant for the
reaction of the cell is- [Question ID = 763]
1. [Option ID = 3051]
2. [Option ID = 3052]
3. [Option ID = 3049]
4. [Option ID = 3050]
Correct Answer :-
[Option ID = 3052]
A characteristic common to polymers that can be made to conduct electricity such as polyacetylene, polypyrrole is: [Question ID =
685]
1. Conjugation throughout the polymeric chain. [Option ID = 2740]
2. A high degree of cross linking [Option ID = 2738]
3. A very low glass transition temperature [Option ID = 2737]
4. Presence of stereogenic centers of the same configuration [Option ID = 2739]
Correct Answer :-
Conjugation throughout the polymeric chain. [Option ID = 2740]
Impact factor is [Question ID = 768]
1. Ratio between citations and recent citable items publish [Option ID = 3071]
2. All of these [Option ID = 3072]
3. Addition of citations and recent citable items publish [Option ID = 3069]
4. Ratio between recent citable items publish and citations [Option ID = 3070]
Correct Answer :-
Ratio between recent citable items publish and citations [Option ID = 3070]
On the basis of oxidation-reduction potential, which of the following is most likely to occur? [Question ID = 693]
13)
14)
15)
16)
17)
1. [Option ID = 2770]
2. [Option ID = 2772]
3. [Option ID = 2771]
4. [Option ID = 2769]
Correct Answer :-
[Option ID = 2769]
How many diastereoisomers are possible for the compound 2, 4 ?diphenylcyclobutane-1, 3 di carboxylic acids. [Question ID = 725]
1. 6 [Option ID = 2899]
2. 5 [Option ID = 2898]
3. 8 [Option ID = 2900]
4. 4 [Option ID = 2897]
Correct Answer :-
5 [Option ID = 2898]
An increase in equivalent conductance of a strong electrolyte with dilution is mainly due to- [Question ID = 764]
1. increase in ionic mobility of ions [Option ID = 3055]
2. increase in number of ions [Option ID = 3054]
3. 100% ionization of electrolyte at normal dilution [Option ID = 3056]
4. increase in both i.e. number of ions and ionic mobility of ions. [Option ID = 3053]
Correct Answer :-
increase in ionic mobility of ions [Option ID = 3055]
The solid state structures of the principal allotropes of elemental boron are made up of which of the following structural units
[Question ID = 699]
1. [Option ID = 2796]
2. [Option ID = 2795]
3. [Option ID = 2794]
4. [Option ID = 2793]
Correct Answer :-
[Option ID = 2793]
The molecular geometry of thionyl chloride is best described as [Question ID = 688]
1. T-shaped [Option ID = 2752]
2. Tetrahedral [Option ID = 2751]
3. Trigonal pyramidal [Option ID = 2749]
4. Trigonal planar [Option ID = 2750]
Correct Answer :-
Trigonal pyramidal [Option ID = 2749]
In a face-center cubic (FCC) type of crystal lattice, the number of atoms belonging exclusively to each unit cell within the lattice
is/are: [Question ID = 754]
1. 2 [Option ID = 3014]
2. 1 [Option ID = 3013]
3. 3 [Option ID = 3015]
4. 4 [Option ID = 3016]
Correct Answer :-
4 [Option ID = 3016]
18)
19)
20)
21)
22)
23)
24)
Among the following, the weakest oxidizing agent is [Question ID = 675]
1. Mg (s) [Option ID = 2698]
2. [Option ID = 2699]
3. [Option ID = 2700]
4. [Option ID = 2697]
Correct Answer :-
Mg (s) [Option ID = 2698]
For a polymer, which of the following statement/s is/are true? [Question ID = 759]
1. Weight average molecular weight is almost always higher than the number average molecular weight [Option ID = 3035]
2. Formation of a polypeptide from its monomers (amino acids) is an example of addition polymerization [Option ID = 3034]
3. All of these [Option ID = 3036]
4. Vinyl polymerization is an example of condensation polymerization. [Option ID = 3033]
Correct Answer :-
Weight average molecular weight is almost always higher than the number average molecular weight [Option ID = 3035]
Quantum dots are [Question ID = 762]
1. Three dimensional [Option ID = 3048]
2. One dimensional [Option ID = 3046]
3. Two dimensional [Option ID = 3047]
4. Zero dimensional [Option ID = 3045]
Correct Answer :-
Zero dimensional [Option ID = 3045]
The unit of rate constant for a third order reaction is: [Question ID = 749]
1. [Option ID = 2993]
2. [Option ID = 2995]
3. [Option ID = 2996]
4. [Option ID = 2994]
Correct Answer :-
[Option ID = 2996]
All the following elements have at least one isotope that is not radioactive EXCEPT [Question ID = 673]
1. Pb [Option ID = 2690]
2. O [Option ID = 2689]
3. Sn [Option ID = 2691]
4. No [Option ID = 2692]
Correct Answer :-
No [Option ID = 2692]
The conditions for a species to follow Bose-Einstein statistics are; [Question ID = 736]
1. Particles are indistinguishable, with no restriction on filling up of energy levels [Option ID = 2944]
2. Particles are indistinguishable, with a restriction on filling up of energy levels [Option ID = 2943]
3. Particles are distinguishable, with a restriction on filling up of energy levels [Option ID = 2941]
4. Particles are distinguishable, with no restriction on filling up of energy levels [Option ID = 2942]
Correct Answer :-
Particles are indistinguishable, with no restriction on filling up of energy levels [Option ID = 2944]
In the kinetic theory of collisions, the SI unit of collision number, in terms of m (meter) and s (second), is:
[Question ID = 761]
25)
26)
27)
28)
29)
1. m-
2
s
-1
[Option ID = 3042]
2. m
4
s
-1
[Option ID = 3041]
3. m
2
s
-1
[Option ID = 3043]
4. None of these [Option ID = 3044]
Correct Answer :-
None of these [Option ID = 3044]
Correct characteristics of the functional groups of adenine in DNA base pair are [Question ID = 706]
1. [Option ID = 2824]
2. [Option ID = 2821]
3. [Option ID = 2823]
4. [Option ID = 2822]
Correct Answer :-
[Option ID = 2822]
The carbon monoxide molecule has an internuclear distance of 1.13 Angstroms. What is the moment of Inertia of this molecule?
[Question ID = 740]
1. [Option ID = 2960]
2. [Option ID = 2957]
3. [Option ID = 2958]
4. [Option ID = 2959]
Correct Answer :-
[Option ID = 2957]
Which of the following represent/s non-linear optical technique? [Question ID = 744]
1. Second Harmonic generation [Option ID = 2974]
2. Two-photon photoluminescence [Option ID = 2975]
3. Four-wave mixing [Option ID = 2973]
4. All of these [Option ID = 2976]
Correct Answer :-
All of these [Option ID = 2976]
. Which of the following does not affect the intensity of spectral lines of a sample? [Question ID = 743]
1. Path length of a sample [Option ID = 2972]
2. Population of energy states [Option ID = 2970]
3. Heisenberg?s Uncertainty principle [Option ID = 2971]
4. Concentration of a sample [Option ID = 2969]
Correct Answer :-
Heisenberg?s Uncertainty principle [Option ID = 2971]
[Question ID = 716]
30)
31)
1. [Option ID = 2862]
2. [Option ID = 2863]
3. [Option ID = 2861]
4. [Option ID = 2864]
Correct Answer :-
[Option ID = 2864]
[Question ID = 712]
1. [Option ID = 2845]
2. [Option ID = 2846]
3. [Option ID = 2848]
4. [Option ID = 2847]
Correct Answer :-
[Option ID = 2847]
[Question ID = 686]
1. [Option ID = 2743]
2. [Option ID = 2742]
3. [Option ID = 2741]
32)
33)
4. [Option ID = 2744]
Correct Answer :-
[Option ID = 2743]
[Question ID = 5633]
1. [Option ID = 22523]
2. [Option ID = 22524]
3. [Option ID = 22526]
4. [Option ID = 22525]
Correct Answer :-
[Option ID = 22523]
[Question ID = 730]
1. [Option ID = 2918]
2. [Option ID = 2920]
3. [Option ID = 2917]
4. [Option ID = 2919]
Correct Answer :-
34)
35)
36)
[Option ID = 2919]
[Question ID = 714]
1. [Option ID = 2856]
2. [Option ID = 2853]
3. [Option ID = 2855]
4. [Option ID = 2854]
Correct Answer :-
[Option ID = 2854]
[Question ID = 669]
1. 2 [Option ID = 2673]
2. 18 [Option ID = 2676]
3. 5 [Option ID = 2674]
4. 10 [Option ID = 2675]
Correct Answer :-
5 [Option ID = 2674]
[Question ID = 689]
1. 2 [Option ID = 2754]
2. 0 [Option ID = 2756]
3. 1 [Option ID = 2755]
37)
38)
4. 3 [Option ID = 2753]
Correct Answer :-
1 [Option ID = 2755]
[Question ID = 718]
1. C) [Option ID = 2871]
2. B) [Option ID = 2870]
3. A) [Option ID = 2869]
4. D) both (B) and (C) [Option ID = 2872]
Correct Answer :-
A) [Option ID = 2869]
[Question ID = 709]
1. [Option ID = 2836]
2. [Option ID = 2833]
3. [Option ID = 2835]
4. [Option ID = 2834]
39)
40)
41)
42)
Correct Answer :-
[Option ID = 2836]
[Question ID = 705]
1. (2Z, 4Z)-3-chlorohexa-2, 4-diene-1,6-diol. [Option ID = 2819]
2. (2E, 4E)-3-chlorohexa-2, 4-diene-1,6-diol. [Option ID = 2817]
3. (2Z, 4E)-3-chlorohexa-2, 4-diene-1,6-diol. [Option ID = 2818]
4. (2Z, 4E)-3-chlorohexa-2, 4-diene-1,6-diol. [Option ID = 2820]
Correct Answer :-
(2Z, 4E)-3-chlorohexa-2, 4-diene-1,6-diol. [Option ID = 2820]
[Question ID = 680]
1. The presence of other complexing ligands in solution affects the equilibrium concentration of metal-EDTA complexes [Option ID = 2719]
2. Metal-EDTA complexes have an equilibrium concentration independent of pH [Option ID = 2718]
3. Metal-EDTA complexes are often 2:1 in stoichiometry [Option ID = 2717]
4. Metal-EDTA complexes are less stable than the corresponding metal-ammine complexes [Option ID = 2720]
Correct Answer :-
The presence of other complexing ligands in solution affects the equilibrium concentration of metal-EDTA complexes [Option ID = 2719]
[Question ID = 671]
1. 150 g [Option ID = 2682]
2. 300 g [Option ID = 2684]
3. 120 g [Option ID = 2681]
4. 180 g [Option ID = 2683]
Correct Answer :-
300 g [Option ID = 2684]
[Question ID = 756]
1. [Option ID = 3021]
2. [Option ID = 3022]
3. [Option ID = 3024]
4. [Option ID = 3023]
Correct Answer :-
43)
44)
45)
[Option ID = 3024]
[Question ID = 750]
1. [Option ID = 2999]
2. [Option ID = 3000]
3. [Option ID = 2997]
4. [Option ID = 2998]
Correct Answer :-
[Option ID = 2997]
[Question ID = 715]
1. [Option ID = 2857]
2. [Option ID = 2860]
3. [Option ID = 2858]
4. [Option ID = 2859]
Correct Answer :-
[Option ID = 2860]
46)
47)
[Question ID = 719]
1. [Option ID = 2873]
2. [Option ID = 2876]
3. [Option ID = 2874]
4. [Option ID = 2875]
Correct Answer :-
[Question ID = 674]
1. Evaporating some water to decrease the volume of the solution. [Option ID = 2696]
2. [Option ID = 2695]
3. [Option ID = 2694]
4. [Option ID = 2693]
Correct Answer :-
[Option ID = 2693]
[Question ID = 728]
1. [Option ID = 2910]
2. [Option ID = 2911]
3. [Option ID = 2909]
4. [Option ID = 2912]
48)
49)
50)
51)
Correct Answer :-
[Option ID = 2912]
[Question ID = 697]
1. III only [Option ID = 2788]
2. II only [Option ID = 2787]
3. I only [Option ID = 2785]
4. I, II and III [Option ID = 2786]
Correct Answer :-
I, II and III [Option ID = 2786]
[Question ID = 700]
1. [Option ID = 2798]
2. [Option ID = 2797]
3. [Option ID = 2800]
4. [Option ID = 2799]
Correct Answer :-
[Option ID = 2799]
[Question ID = 678]
1. Cr
2+
has a low cationic charge [Option ID = 2711]
2. F has -1 anionic charge and highly electronegative [Option ID = 2709]
3. Spin-orbit coupling in Cr
2+
[Option ID = 2712]
4. The Jahn-Teller effect [Option ID = 2710]
Correct Answer :-
The Jahn-Teller effect [Option ID = 2710]
[Question ID = 723]
1. [Option ID = 2889]
52)
53)
2. [Option ID = 2890]
3. [Option ID = 2891]
4. [Option ID = 2892]
Correct Answer :-
[Option ID = 2889]
[Question ID = 727]
1. [Option ID = 2907]
2. [Option ID = 2906]
3. [Option ID = 2905]
4. [Option ID = 2908]
Correct Answer :-
[Option ID = 2905]
[Question ID = 707]
54)
55)
56)
1. N9 [Option ID = 2828]
2. N7 [Option ID = 2827]
3. N1 [Option ID = 2825]
4. N3 [Option ID = 2826]
Correct Answer :-
N7 [Option ID = 2827]
[Question ID = 672]
1. Bicapped prism [Option ID = 2688]
2. Square pyramidal [Option ID = 2686]
3. Trigonal planar [Option ID = 2685]
4. Bent [Option ID = 2687]
Correct Answer :-
Square pyramidal [Option ID = 2686]
[Question ID = 704]
1. [Option ID = 2816]
2. [Option ID = 2814]
3. [Option ID = 2813]
4. [Option ID = 2815]
Correct Answer :-
[Option ID = 2813]
57)
58)
[Question ID = 708]
1. [Option ID = 2829]
2. [Option ID = 2832]
3. [Option ID = 2831]
4. [Option ID = 2830]
Correct Answer :-
[Option ID = 2831]
[Question ID = 717]
1. [Option ID = 2866]
2. [Option ID = 2865]
3. [Option ID = 2867]
4. [Option ID = 2868]
Correct Answer :-
[Option ID = 2866]
59)
[Question ID = 711]
1. [Option ID = 2843]
2. [Option ID = 2842]
3. [Option ID = 2844]
4. [Option ID = 2841]
Correct Answer :-
[Option ID = 2842]
[Question ID = 710]
1. [Option ID = 2838]
2. [Option ID = 2839]
3. [Option ID = 2837]
4. [Option ID = 2840]
Correct Answer :-
60)
61)
62)
[Option ID = 2838]
[Question ID = 682]
1. 0.30 M [Option ID = 2728]
2. 0.25 M [Option ID = 2727]
3. 0.60 M [Option ID = 2725]
4. 0.10 M [Option ID = 2726]
Correct Answer :-
0.30 M [Option ID = 2728]
[Question ID = 772]
1. [Option ID = 3087]
2. [Option ID = 3088]
3. [Option ID = 3086]
4. [Option ID = 3085]
Correct Answer :-
[Option ID = 3086]
[Question ID = 733]
1. [Option ID = 2932]
63)
64)
65)
2. [Option ID = 2929]
3. [Option ID = 2930]
4. [Option ID = 2931]
Correct Answer :-
[Option ID = 2929]
[Question ID = 767]
1. 120 [Option ID = 3065]
2. 280 [Option ID = 3066]
3. 300 [Option ID = 3068]
4. 20 [Option ID = 3067]
Correct Answer :-
20 [Option ID = 3067]
[Question ID = 734]
1. [Option ID = 2936]
2. [Option ID = 2933]
3. [Option ID = 2934]
4. [Option ID = 2935]
Correct Answer :-
[Option ID = 2935]
[Question ID = 676]
66)
67)
68)
1. [Option ID = 2702]
2. [Option ID = 2703]
3. [Option ID = 2704]
4. [Option ID = 2701]
Correct Answer :-
[Option ID = 2703]
[Question ID = 729]
1. (2Z, 4Z)-3, 4-dibromo hepta-2, 4-diene [Option ID = 2914]
2. (2E, 4E)-3, 4-dibromo hepta-2, 4-diene [Option ID = 2915]
3. (2E, 4Z)-3, 4-dibromo hepta-2, 4-diene [Option ID = 2916]
4. (2Z, 4E)-3, 4-dibromo hepta-2, 4-diene [Option ID = 2913]
Correct Answer :-
(2E, 4Z)-3, 4-dibromo hepta-2, 4-diene [Option ID = 2916]
[Question ID = 765]
1. [Option ID = 3060]
2. [Option ID = 3057]
3. [Option ID = 3058]
4. [Option ID = 3059]
Correct Answer :-
[Option ID = 3059]
[Question ID = 724]
1. [Option ID = 2893]
69)
2. [Option ID = 2895]
3. [Option ID = 2896]
4. [Option ID = 2894]
Correct Answer :-
[Option ID = 2893]
[Question ID = 721]
1. [Option ID = 2881]
2. [Option ID = 2884]
3. [Option ID = 2882]
4. [Option ID = 2883]
Correct Answer :-
[Option ID = 2882]
70)
71)
72)
73)
[Question ID = 766]
1. 512 times [Option ID = 3064]
2. 256 times [Option ID = 3063]
3. 128 times [Option ID = 3062]
4. 64 times [Option ID = 3061]
Correct Answer :-
512 times [Option ID = 3064]
[Question ID = 670]
1. 2 [Option ID = 2678]
2. 5 [Option ID = 2680]
3. 1 [Option ID = 2677]
4. 3 [Option ID = 2679]
Correct Answer :-
5 [Option ID = 2680]
[Question ID = 731]
1. [Option ID = 2924]
2. [Option ID = 2923]
3. [Option ID = 2922]
4. [Option ID = 2921]
Correct Answer :-
[Option ID = 2922]
[Question ID = 690]
1. [Option ID = 2760]
2. [Option ID = 2759]
3. [Option ID = 2757]
4. [Option ID = 2758]
74)
75)
Correct Answer :-
[Option ID = 2758]
[Question ID = 720]
1. [Option ID = 2879]
2. [Option ID = 2878]
3. [Option ID = 2880]
4. [Option ID = 2877]
Correct Answer :-
[Option ID = 2877]
[Question ID = 722]
1. [Option ID = 2886]
2. [Option ID = 2888]
3. [Option ID = 2885]
4. [Option ID = 2887]
Correct Answer :-
76)
77)
78)
[Option ID = 2887]
[Question ID = 755]
1. [Option ID = 3019]
2. [Option ID = 3018]
3. [Option ID = 3020]
4. [Option ID = 3017]
Correct Answer :-
[Option ID = 3019]
[Question ID = 695]
1. Ba [Option ID = 2779]
2. BaO [Option ID = 2780]
3. BaOH [Option ID = 2778]
4. BaH
2
[Option ID = 2777]
Correct Answer :-
BaO [Option ID = 2780]
[Question ID = 703]
1. [Option ID = 2811]
2. [Option ID = 2809]
3. [Option ID = 2812]
4. [Option ID = 2810]
Correct Answer :-
[Option ID = 2811]
79)
80)
81)
82)
83)
[Option ID = 2810]
[Question ID = 732]
1. [Option ID = 2927]
2. [Option ID = 2928]
3. [Option ID = 2926]
4. [Option ID = 2925]
Correct Answer :-
[Option ID = 2926]
A monoatomic gas following Fermi-Dirac statistics begins to follow Maxwell-Boltzmann statistics at: [Question ID = 735]
1. Low Temperature and low density [Option ID = 2937]
2. High Temperature and high density [Option ID = 2940]
3. Low Temperature and high density [Option ID = 2938]
4. High Temperature and low density [Option ID = 2939]
Correct Answer :-
High Temperature and low density [Option ID = 2939]
The Dulong and Petit?s Law says that the molar heat capacity of elements is: [Question ID = 741]
1. [Option ID = 2964]
2. [Option ID = 2961]
3. [Option ID = 2963]
4. [Option ID = 2962]
Correct Answer :-
[Option ID = 2961]
What is the most common natural form in which fluorine is found on earth?
[Question ID = 684]
1. As a fluoride ion in various minerals [Option ID = 2735]
2. As weak acid HF (aq) [Option ID = 2734]
3. In various fluorocarbon compounds in the atmosphere. [Option ID = 2736]
4. As XeF
2
(s)

[Option ID = 2733]
Correct Answer :-
As a fluoride ion in various minerals [Option ID = 2735]
What is the correct form of Stirling?s approximation?
[Question ID = 738]
1. [Option ID = 2950]
2. [Option ID = 2951]
84)
85)
86)
87)
88)
3. [Option ID = 2949]
4. [Option ID = 2952]
Correct Answer :-
[Option ID = 2950]
What is the total energy of one mole of an ideal monoatomic gas in terms of Boltzmann?s Constant (k), Avogadro?s number (N) and
temperature (T)
[Question ID = 739]
1. 3 NkT [Option ID = 2953]
2. (3/2) NkT [Option ID = 2956]
3. (1/2) NkT [Option ID = 2955]
4. NkT [Option ID = 2954]
Correct Answer :-
(3/2) NkT [Option ID = 2956]
The following equation is associated with the relationship between the diffusion current and the concentration of the depolarizer
used in polarography: [Question ID = 753]
1. Debye-Huckel equation [Option ID = 3009]
2. Stern-Volmer equation [Option ID = 3010]
3. Nyquist equation [Option ID = 3012]
4. Ilkovic equation [Option ID = 3011]
Correct Answer :-
Ilkovic equation [Option ID = 3011]
Electronic transitions originating from the 1S energy level of the Hydrogen atom to higher levels belong to which series? [Question ID
= 747]
1. Lyman Series [Option ID = 2985]
2. Brackett Series [Option ID = 2987]
3. Balmer Series [Option ID = 2986]
4. Pfund Series [Option ID = 2988]
Correct Answer :-
Lyman Series [Option ID = 2985]
Which of the following reactions best classified as an oxidative addition? [Question ID = 701]
1. [Option ID = 2801]
2. [Option ID = 2803]
3. [Option ID = 2802]
4. [Option ID = 2804]
Correct Answer :-
[Option ID = 2804]
Which of the following is required for both paramagnetism and ferromagnetism? [Question ID = 698]
1. Super exchange [Option ID = 2791]
2. unpaired electrons [Option ID = 2792]
3. Low-spin electron configuration [Option ID = 2790]
4. Strong oxidizing conditions [Option ID = 2789]
Correct Answer :-
unpaired electrons [Option ID = 2792]
89)
90)
91)
92)
93)
94)
Which of the following experimental techniques is not used to determine the average molecular weight of a polymer? [Question ID =
760]
1. Transmission electron microscopy [Option ID = 3039]
2. Equilibrium sedimentation [Option ID = 3038]
3. Intrinsic viscosity measurement [Option ID = 3040]
4. Dynamic light scattering [Option ID = 3037]
Correct Answer :-
Transmission electron microscopy [Option ID = 3039]
Which of the following is NOT a known relatively stable compound of uranium? [Question ID = 687]
1. [Option ID = 2745]
2. [Option ID = 2748]
3. [Option ID = 2747]
4. [Option ID = 2746]
Correct Answer :-
[Option ID = 2746]
Which of the following compounds exist in stereoisomeric form? [Question ID = 681]
1. [Option ID = 2721]
2. [Option ID = 2724]
3. [Option ID = 2722]
4. [Option ID = 2723]
Correct Answer :-
[Option ID = 2724]
Which of the following statement is not true? [Question ID = 745]
1. Methane is a spherical top molecule [Option ID = 2978]
2. Chloroform is a symmetric top molecule [Option ID = 2980]
3. Vinyl chloride is a symmetric top molecule [Option ID = 2979]
4. Water is an asymmetric top molecule [Option ID = 2977]
Correct Answer :-
Vinyl chloride is a symmetric top molecule [Option ID = 2979]
Which of the following is a n-type semiconductor? [Question ID = 696]
1. Silicon carbide [Option ID = 2784]
2. Silicon [Option ID = 2781]
3. Arsenic doped silicon [Option ID = 2783]
4. Gallium doped silicon [Option ID = 2782]
Correct Answer :-
Arsenic doped silicon [Option ID = 2783]
Which of the statement is not true? [Question ID = 746]
1. Franck Condon Principle states that during electronic transition the internuclear distance of a molecule does not change [Option ID = 2983]
2. The intensity of a fundamental vibrational transition is higher than that of a first overtone transition. [Option ID = 2984]
3. Morse equation represents the energy expression of a simple harmonic oscillator [Option ID = 2982]
4. The energy spacing between various vibrational levels are the same in a simple harmonic oscillator [Option ID = 2981]
Correct Answer :-
Franck Condon Principle states that during electronic transition the internuclear distance of a molecule does not change [Option ID = 2983]
95)
96)
97)
98)
99)
100)
Which of the statement is true? [Question ID = 752]
1. The mean ionic activity coefficients of aqueous NaCl solution and aqueous KBr solution, both at low concentrations, are independent of their respective
ionic strengths [Option ID = 3008]
2. The mean ionic activity coefficient of aqueous NaCl solution at low concentration decreases with increase in its ionic strength [Option ID = 3006]
3. The mean ionic activity coefficients of aqueous NaCl solution and aqueous KBr solution, both at low concentrations, vary differently upon increase of
their respective ionic strengths [Option ID = 3007]
4. The mean ionic activity coefficient of aqueous NaCl solution at low concentration increases with increase in its ionic strength [Option ID = 3005]
Correct Answer :-
The mean ionic activity coefficient of aqueous NaCl solution at low concentration decreases with increase in its ionic strength [Option ID = 3006]
The highest temperature that can be achieved due to a single normal mode of vibration in a solid crystal is known as: [Question ID =
757]
1. Debye Temperature [Option ID = 3026]
2. Theta Temperature [Option ID = 3027]
3. Curie Temperature [Option ID = 3025]
4. Flory Temperature [Option ID = 3028]
Correct Answer :-
Debye Temperature [Option ID = 3026]
Which is not a scientific site? [Question ID = 691]
1. Research Gate [Option ID = 2763]
2. Scopus [Option ID = 2761]
3. Web of Science [Option ID = 2762]
4. Google Plus [Option ID = 2764]
Correct Answer :-
Google Plus [Option ID = 2764]
According to the Michaelis Menten equation for unimolecular reactions: [Question ID = 751]
1. The rate is first order at low pressure, but becomes zero order at high pressure [Option ID = 3003]
2. The rate is zero order at both low and high pressures [Option ID = 3002]
3. The rate is zero order at low pressure, but becomes first order at high pressure [Option ID = 3004]
4. The rate is first order at both low and high pressures [Option ID = 3001]
Correct Answer :-
The rate is first order at low pressure, but becomes zero order at high pressure [Option ID = 3003]
The +1 oxidation state is more stable than +3 oxidation state for which of the following Group 13 element [Question ID = 694]
1. In [Option ID = 2775]
2. B [Option ID = 2773]
3. Al [Option ID = 2774]
4. Tl [Option ID = 2776]
Correct Answer :-
Tl [Option ID = 2776]
In how many ways can 10 distinguishable particles be placed in 3 boxes, so that there are 3 particles in first box, 5 in second and 2
in third? [Question ID = 737]
1. None of these [Option ID = 2948]
2. 1520 ways [Option ID = 2946]
3. 3260 ways [Option ID = 2947]
4. 2520 ways [Option ID = 2945]
Correct Answer :-
2520 ways [Option ID = 2945]
FirstRanker.com - FirstRanker's Choice

This post was last modified on 29 January 2020